Download as pdf or txt
Download as pdf or txt
You are on page 1of 175

NBME CBSE ACTUAL TEST QUESTIONS AND

ANSWERS 2023-2024
62 year old woman - osteoporosis - a bisphosphonate is prescribed. The expected beneficial
effect of the drug is due to which of the following? -Answer- Decreased Osteoclast Activity

Cohot Study of elderly women - relative risk ratio for hip fractures among those who exercise
regularly is 1.2 (95% confidence interval of 1.1 to 1.8). Which of the following is the conclusion
about the effect of exercise on the risk of hip fracture? -Answer-
Statistically Significant Overall Increase Risk

52 year old man goes to ER with chest pain radiating to his jaw while shoveling snow.
Pulse is 80/min and blood pressure is 130/70. The most immediate treatment
mechanism of action? -Answer-
24 year old woman - spilled hot grease on her left leg while working at a fast-food restaurant.
Exam of leg shows 7cm pink, soft, granular, edematous wound. The formation of this tissue
was most likely caused by increased activity of which? -Answera. Vascular Endothelial Growth
Factor VEGF - stimulates angiogenesiss.
TYPE III Collagen = Blood Vessels - early wound repair

27 year old man- MVC - skull x-ray shows a linear, nondepressed basal skull fracture increased
serum osm and decreased urin osm. Following desmopressin urine osm increases.
Desmopressin's effect is due to the activation of which of the following? - Answer- a. Adenylyl
Cyclase
Adenylate Cyclase - ATP - CAMP -- PROTEKINASE A -- Ca v2
A 10 month old boy - 4 day history of fever and cough. He attends day care center.
Chest exam shows intercostal retractions along with bilateral, diffuse wheezes and
expiratory rhonchi. The infectious agent most likely has which of the following
properties? -Answer- Mediation of Cell Entry via a fusion protein

A 17 year old girl in ED - 15 minutes after being stung by a bee. Mild light headedness
but no difficulty swallowing- Bilateral wheezing - Which is most appropriate
pharmacotherapy for this patient? -Answer- B2- Agonist
14 year old boy - 2 day history of sore throat and fever that peaks in late afternoon. 1 week
of fatigue. He recentaly had sex with one partner. Physical exam show cervical
lymphadenopathy and pharyngeal erythema with a creamy exudate. DX?
-Answer- Infectious Mononucleosis
57 year old man - radiation therapy for squamous cell carcinoma of the lung. Despite
therapy, tumor increases in size and he dies 6 months later. The progressive tumor
growth is due to a defect in cell cycle arrest in which of the following phases of cell
cycle? -Answer- G1
28 year old - lived in sub-Saharan Africa - until he came to the US. Temp of 100.4 - imaging
shows bilateral hydroureter and hydronephrosis. Biopsy shows marked fibrosis and scattered
granulomas. DX? -Answer- Schistosomiasis

A couple with a family history of a-thalassemia. Woman has one gene deletion and man has two
gene deletion. If the two gene is trans - what percentage of offspring will have a two gene
deletion? -Answer- 50%
previously healthy 40 year brought to emergency department by her husband - 2 day history
of fever, lethargy, confusion. PE shows scattered petechiae and ecchymoses over the lower
extremities - 3+ polychromasia and 3+ schistocytes and Low platelets
Dx? -Answer- Thrombotic Thrombocytopenia Purpura
16 year old boy - is admitted to the ER because of a knife wound to the left side of his chest.
An X-ray of the chest shows an air-fluid level in the left side of the chest, partial collapse of the
left lung, and the elevation of the stomach bubble. The mediastinum is midline. DX? -Answer-
Hemopneumpothorax under tension
49 year old woman - coronary artery disease - BP 140/90 - High Cholesterol, High LDL
(190), High triglycerides (350) - TX with atorvastatin and losartan. What are the effects
on HDL and Triglycerides? -Answer- HDL increased
Triglycerides Decreased
73 yeare old - diffuse weakness and tingling of her arms and legs. Sensation and vibration and
position is decreased in all extremities. What vitamin deficiency? -AnswerVitamin B12 -
(cyanocobalamin)
Tea and Toast - low B12 in diet
15 year old girl - 3 month history of acne - which is the underlying cause of the patients
acne? -Answer- Stimulation of Sebaceous Glands by androgens
b. ACNE = Propionibacterium ACNE
4 year old from Brazil - PE shows single 12x10cm lesion in the right side of jaw with diffuse
regular edges. Photomicrographs of an incisional biopsy (looks like Burkitts Lymphoma/ Starry
night) - which of the processes most likely to occur in the region indicated by the arrow? -
Answer- Apoptosis
b. Endemic Burkitt lymphoma can happen in Brazil as well as Africa (jaw lesion, puffy face).
51 year old - lump on tongue - 1 pack smoking history for 30 years. 1.5 cm mass on apex of
tongue. It is most appropriate to evaluate which lymph nodes first for evidence of metastasis? -
Answer- Submental

15 year old boy - ER - 2 hour history of confusion and agitation - fever, headache, stiff neck, and
vomiting - since returned from summer camp - patient is hallucinating - lumbar puncture -
shows cysts and trophozoites- most likely pathogen? -Answer- a. Olfactory Nerve
Naegleria fowleri

17 year old - ED - 30 minutes after being found with a blank stare. Physical exam shows rigidity.
During exam he becomes hostile and assaults physician - Pt ingested which drug? -Answer- PCP
Placebo controlled clinical trial - 5000 pts with essential hypertension. 2500 patients receive
new drug and 2500 patients receive placebo. If alpha is set at 0.01 instead of 0.05, which of the
following is most likely result? -Answer- Significant findings can be reported with greater
confidence

17 year old - gymnast - comes to hospital because of lack of menstrual period for 6 months.
BMI 15 Which is the cause of the amenorrhea? -Answer- Hypogonadotropic Hypogonadism
A male stillborn is delivered at 32 weeks - Oligohydramnios - absence of a urethral
opening. Which is most likely finding? -Answer- Pulmonary Hypoplasia
A 6 day old - breast fed boy in ED - poor weight gain and irritability since delivery - Physical
exam shows jaundice and hepatomegaly. The concentration of which of the following
metabolites is most likely increased? -Answer- a. Galactose - 1 - phosphate b. Congenital
intolerance to breast milk

A 25-year-old man - comes to ED - severe muscle pain, diffuse, painful swelling of his neck,
underarms, and groin after camping in New Mexico Generalized scattered black maculae.
Examination of the right upper extremity shows erythematous, solid, tender mass. Mass is
draining blood and necrotic material. The most effective antibiotic for patient disorder will
interfere with which of the following processes? -Answer- a. Ribosomal Assembly b. Yersenia
Pestis
45 year old - progressive weakness - muscle fasciculations of the upper extremities and
weakness of the lower extremity - What additional findings? -Answer- Atrophy
b. ALS = Lou Gherig

A new severe respiratory illness - Why use a killed vaccine vs a live vaccine? -Answer-
Avoids Concerns of reversion to virulence

b. Killed vaccines - avoids reversion to virulence


c. live vaccines - can (but rarely do) cause the disease they're designed to prevent

A 33 year old - keratinizing squamous cell carcinoma of cervix. Which of the following describes
pathogenesis of this patient's disease? -Answer- Inactivation of Cellular P53 b. p53 protein =
tumor suppressor (it activates apoptosis) - most human cancer

A 54 year old - 40 year history of T1DM - receiving hemodialysis for end stage renal disease
while awaiting a kidney transplant. Receives a drug that induces reticulocyte release from bone
marrow and stimulates a cytokine receptor that signals Jak/Stat pathway? -Answer-
Erythropoietin
Jak Stat - erythropoietin
During a clinical study examining the effects of exercise. The average pulse is 175/min.
Compared with measurement before the session, which is most likely decreased? -
Answer- Total Peripheral Resistance
An 8-year-old boy - 3-day history of fever, sore throat, and itchy eyes. Returned from week long
summer camp that includes hiking trips and swimming - PE shows conjunctival injection and
oropharyngeal edema - Outbreak among other campers.
Which is most likely cause of this patient's symptoms? -Answer- Adenovirus
Fever+ Sore Throat + Itchy Eyes transmitted via
swimming pools Conjunctivitis Viral =
adenovirus
44-year-old woman - 10 month history of wide red streaks over her lower trunk (striae) and
weight gain in face (moonface). Which additional findings? -Answer- Hypertension
and muscle weakness

b. Cushing - hypertension and muscle weakness

12 year old boy - pain below left knee -unable to play soccer - An x-ray shown - Which
structures attached to the abnormal anterior tibial area? -Answer- patellar ligament
b. Osgood-Schlatter
A 65-year-old health maintenance exams - He lives is a single-family home with his cat and dog.
He spend much of his time in his basement woodworking shop. This patient is increased risk for
lung cancer due to which of the following environmental exposures? - Answer- Radon

54-year-old man - intense overwhelming fear. Which portion of brain stimulated? -


Answer- Amygdala b.
Fear = amygdala

30 year old woman - recurrent URI - Sweat is Salty- Genetic testing for 36 most common
mutations shows detectable G551D in one allele of CFTR - What is patients clinical phenotype? -
Answer- The Second CFTR Gene was not detected by the testing obtained
74 year old - COPD - "I enjoy coming to see you because you remind me of my daughter. She
died 35 years ago". Which is most appropriate response by physician? Answer- You must miss
your daughter very much. Tell me about her. b. PT = Transference

9 month old boy - awakens and cries at least once nightly and settles back to sleep after
drinking a bottle of formula - Explanation for patient's sleep pattern? -Answer- Normal
Development
Pubertal gynecomastia in males is norma
A 32 year old - abnormal movement in hands and worse when he feels angry - Unable to fix his
gaze on one point or protrude his tongue for more than 30 sec - Pt most likely has anatomic
abnormalities in which of the following locations? -Answer- Basal
Ganglia
A 14-year-old boy - recurrent otitis media since infancy - underwent tonsillectomy for
obstructive sleep apnea - Cardiac auscultation exam listen - shows S3 - which is most likely
finding? -Answer- Normal Findings b. normal physiologic splitting
37 year old woman - right lower extremity edema - sudden SOB+, If present on physical
exam which sign would be the most specific indicator or pulmonary arterial
hypertension? -Answer- P2 louder than A2
b. Pulmonary Hypertension - Split S2

43 year old - with t2DM - ER - nausea and vomiting - after drinking champagne. Treated
for Trichomonas Vaginalis Infection - TX with metronidazole. What is cause of nausea
and vomiting? -Answer- Accumulation of Acetaldehyde
b. Metronidazole - disulfiram reaction
45 year old man - right shoulder pain - patient unable to externally rotate the shoulder against
resistance - Which of the following tendons is inflamed? -Answer- Infraspinatus b. SITS Muscles
c. Teres Minor and Infraspinatus external rotation
d. Innervated by Suprascapular Nerve
A 44 year old man - 6 week feet numbness - chemotherapy with vincristine - MOA of
Vincristine? -Answer- Depolymerization of Microtubules
10 year old - 95th percent height and 25th weight - Nurse thinks she has marfans - Molecular
testing for FBN1 Gene shows single nucleotide change does not change amino acid at the locus
- Explain the single nucleotide change in the patient and the mother? -Answer- It is a
polymorphism
26 year old woman - ED - 2 day history of runny nose - Patient has to wait 6 hours before she is
seen. In addition to apologizing, which of the following is the most appropriate remark? -
Answer- Thank you for waiting. How can I help you today?

A 26 year old man - ER -m 30 minutes after being shot in the leg - Pulse is 120/min,
respirations are 16 /min - blood pressure is 80/60. Findings compared with healthy adult? -
Answer- Baroreceptor Firing decreased
SVR increased
Pulmonary Vascular Resistance Increased
Absorption Increased
e. BARORECPTOR - stretch receptors - mirror BP - decreased in hemorrhagic shock
A 36 year old man - fluid is presenting adbomen in which of the following areas as
indicated by the arrow? -Answer- Omental Bursa (lesser sac)
What is the positive predictive value (refer to chart)? -Answer- .67
Probability that a patient with a positive test has a disease
(A+B) A/
A 75 year old woman with T2DM and HTN - Emigrated from Argentina - Which of the
following is the most appropriate person to serve as an interpreter for this patient
encounter? -Answer- Telephone Interpreter

During a study of renal glomeruli - if efferent arteriole is constricted - which starling


forces are likely to change in the glomeruli? -Answer- Increased Hydrostatic Pressure
2 year old with neurocognitive dysfunction - fine during birth - both parents have learning
disabilities - maternal uncle with cognitive disabilities - Irritable making eye contact with
flapping his hands? -Answer- Trinucleotide Repeat Expansion b. Fragile X - intellectual
disability - trinucleotide repeat in FMR1 gene
A 33 year old man - undergoes radical thyroidectomy - postop serum calcium is 7.5 and
albumin concentration of 4g/dl - PTH concentration of 200 - damage to which vessel
causes finding? -Answer- Branch of Thyrocervical Trunk

b. inferior thyroid arteries, which arise from the thyrocervical trunk.


c. External carotid - superior thyroid
46 year old woman - 2 month history of fatigue and muscle weakness - 10 year history of
hypertension - treated with thiazide diurectics - measured hypokalemia, hypotension and
decreased BP- brisk reflexes. Likely cause? -Answer- Adverse Drug
Effect
b. Treated with a thiazide diuretic for HTN with measured hypokalemia, hypotension and
decreased Hg

55-year old man - nice to nurses - mean to youngest child- Which is the most likely
explanation of patient's behavior? -Answer- Splitting b. Cluster B - borderline
personality (BANH)
A Study designed to evaluate the feasibility of acupuncture in children with chronic headache -
60 children recruited - In addition to their usual therapy - all children are treated with
acupuncture three times per week for 2 months - Describe findings? -
Answer- Case-Series
6 year old girl - 1 week history - lots of infections -raised neutrophil count- but cells show
a delay in bactericidal activity against Staph Aureus- Most likely cause? -Answer-
Myeloperoxidase
2 year old boy - severe pain, swelling, and redness of his left thumb (image). PE shows
oral vesicle, cervical lymphadenopathy - infectious agent causing findings in thumb? -
Answer- DNA Virus Herpetic Whitlow

Herpetic Whitlow -Answer- vesicles on fingers

7 year old - lives in Kentucky - 2 week history of cramping abdominal pain and diarrhea
- mother looked in underpants and saw earthworm - no travel history- Transmission via?
-Answer- Ingestion of Soil
b. Ascaris - fecal oral transmission

23 year old man with multiple sex partners - dysuria and yellow urethral exudate - Gram Stain -
numerous neutrophils - many contain gram negative diplococci. He had 3 episodes over the
past 2 years. Which property of the organism - explains reinfection? -
Answer- Antigenic Variation
b. Gram negative Diplococci
c. N. gonorrhea can change its pilus,
23 year old woman - genetic counseling - brother and maternal uncle had Duchene
Muscular Dystrophy - Serum CK of 120 - and mother serum CK 300- What is assessment of
carrier status? -Answer- The patients DMD carrier status is uncertain because of random X
Inactivation b. Barr Body
c. DMD is X-linked.
20 year old woman - heavy bleeding - Platelet Aggregation is normal - PTT elevated -
no easy bruising - Which hemorrhagic disorder cause of patients menorrhagia? -
Answer- Von Willebrand Disease

B. Platelet aggregation time being normal, ok fine I can see that.


32 year old man - ER - for MVC - urethral injury - blood noted at the uretheral meatus. Which
portion of the urethra would be at greatest risk of injury? -Answer- Membranous a. Pelvic
fracture - MVC
b. Pelvic Straddle injury = Bulbar (spongy) urethera
A 63 year old - ED - unresponsive - Crackles heard over left upper and entire right lung. Pt dies.
Photograph of section of right lung obtain - Which is likely cause of the position indicated by the
arrow? -Answer- C5a
Complement system calls in neutrophils
C5a = neutrophil chemotaxis

A 22 year old woman - 6 months sever lower backpain - endometrial biopsy shows abundant
mitotic figures in the endometrial glands and stroma. Which protein or enzymes regulate the
progression of this phase of the menstrual cycle? -Answer- Cyclin Dependent Kinase.

29 year old woman - prescribed carbamazepine for trigeminal neuralgia- family history of
osteoarthritis - the most likely reason for this recommendation is that carbamazepine may
affect which of the following processes? -Answer- Metabolism

An 18 year old woman - fever, dizziness, weakness, and vomiting (toxic shock) - using synthethic
pads and tampons - Physical exam shows injected conjunctiva and rash- Pts use of which
increased risk? -Answer- Tampons
A 53 year old man - - 6 month history of blood in stool - 1 cm anal mass - If mass is
malignant, evaluate which lymph nodes? -Answer- Superficial Inguinal
1 cm mass below dentate line
24 year old - brought to ED - MVC - face struck steering wheel - facial pain and double vision -
Exam of left eye shows infraorbital edema - ocular movement restricted vertically - CT scan
shown. Which of the following impaired on left side?
-Answer- Sensation over the upper lip
V2 = maxillary nerve

23 year old - collapsed at finish line - no loss of consciousness Associated with prolonged
use of vaginal tampons or nasal packing She is placed on a cot - feet elevated - 30
minutes later her blood pressure is 110/70 mm Hg. Which described patients condition?
-Answer- Orthostatic Hypotension .decrease in systolic blood pressure of 20 mm Hg
(110-85)
e. decrease in diastolic blood pressure of 10 mm Hg. (70-50)

27 year old - ER - pain in navel - nausea and one episode of vomiting 1 hour ago - underwent
appendectomy 2 years ago- CT scan of abdomen shows fat stranding in small bowel with areas
of inflammation in terminal illeum - 2 cm bulge on antimesenteric border of the inflamed
segment. Histological specimen - most likely to show which of the following? -Answer- Gastric
Mucosa
b. Crohns with the multiple episodes over the last year, fat stranding, terminal ileum
involvement
14 year old girl -parents divorced - starts acting sullen, defiant, and truant at school, sexual
intercourse with new partners and no condoms, previously honor studenty- which defense
mechanisms? -Answer- Acting Out

38 year old - pregnant woman - ultrasound shows oligohydramnios and a full term fetus with a
large kidney and empty right renal fossa. Need history on maternal use of which during
pregnancy? -Answer- ACE Inhibitors
A 65 year old - bilateral wheezing - accumulation of which of the following? -Answer-
Leukotrienes
19 year old - with asthma - comes to ER multiple times for acute asthma exacerbations - "All
doctors don'g understand me" - they keep prescribing steroid inhalers. What is the most
appropriate statement by the physician? -Answer- Tell me more about what you know about
steroid inhalers and how are they supposed to work?

A healthy 22 year old man - in study for glucose metabolism - He consumes an 800 calories
meal consisting - 12 hours later serum glucose is within reference range -
Which is involved in maintaining serum glucose? -Answer- Glycogenolysis of Liver
30 year old man - 6 week history of colicky abdominal pain and diarrhea with occasional blood -
on autopsy - small bowel is seen in the wall of a thickend loop of small intestine -
Photomicrographs (granulomas)- DX? -Answer- Crohns Disease
A 3 month old boy - 2 hour respiratory distress - Physical exam shows cyanosis - blood sample
appears brown - most likely cause of patients condition" -Answer- Decreased activity in the
enzyme that reduced Fe3+ to Fe 2
a. the hypoxia and the "blood sample appears brown" should point you towards
methemoglobinemia.
b. if you want a fun video from "9-1-1" that actually explains a lot of it:
c. methemoglobinemia = cyanosis and chocolate colored blood.
68 year old - 1 day history of fever and swelling in left leg - cellulitis - gram stain - gram
positive, catalase negative cocci - clear zones of blood agar - Which species of
Streptococcus? -Answer- Strep Pyogenes
67 year old woman - bicuspid aortic valve - admitted to the hospital - 2 day history of fever and
chills. Blood culture grows viridans streptococci - In addition to penicillin, which other drug is
administered to shorten the duration of treatment? -Answer- Binding to 30S protein
12-year-old girl - 2 month history of intermittent yellowing of the eyes and skin - PE - no
abnormalities except jaundice. Total bilirubin 3mg/dl unconjugated bilirubin > Direct -
Findings in patient? -Answer- Decreased activity of UDP glucoronylytransferase

During an experiment, drug X is added to a muscle bath containing a strip of guinea pig
intestinal smooth muscle. Agonists are added to the bath, and the resultant effects on
muscle tension are shown in table. Acetylcholine increased. Which of the following
drugs most likely to produce effects most similar to those of drug X? -Answer-
Cholinesterase Inhibitor

30 year old women - 28 week gestation - Fetal ultrasound normal - Father's blood group
in B, Rh Positive - Physician recommends administration of Rh0D immune globulin -
This treatment is most likely to prevent? -Answer- Formation of antibodies to RhD
55- year -old -man - hypertension and chronic kidney disease - ECG shows low voltage with
electrical alternans. Physical exam is most likely to show which of the following? - Answer- BP
85/60
Pulse 120
JVP Increased
Pulse Paradoxus Increased
a. Electrical Alternans = Cardiac Tamponade = pleural effusion
b. Beck Triad - Hypotension, JVP, muffled heart sounds

52-year-old - newly diagnosed with Type 2 diabetes mellitus. Four weeks later, her hepatic
glucose out is decreased and target tissue glucose uptake are increased. Which agent? -Answer-
Metformin

An asymptomatic 44-year-old man - with HIV infection during routine screening to donating
blood. A complete blood count- platelets are low - which of the following finds are most likely?
-Answer- Petechiae
Platelet problem -Answer- epistaxis, ecchymoses, petechia, bleeding from superficial
scratches

Coagulation problem -Answer- late re-bleed, Menorrhagia, GI bleeds, hemarthroses

23 year old - bone marrow failure - given a large dose of rabbit antimyocyte globulin - 10 days
later - develops fever, lymphadenopathy, arthralgias, and erythema on her hands and feet.
Which is the most likely cause of these symptoms? -Answer- Immune complex deposition in
tissues
b. Serum sickness - TYPE 3 hypersensitivity

42 year old - gunshot wound - 36 hours post-operatively she is receiving morphine. she needs
morphine - she has expressed concern she is becoming addicted. Which of the following initial
actions by the physician is most appropriate? -Answer- Reassure the patient has a miniscule
chance of becoming addicted to narcotics
A 22 y/o woman -- itchy, red rash on her arm - new hair dye - picture (contact dermatitis) -
Which ligand pairs play a role in the proliferation of the T Lymphocytes Answer- CD28 on T
Lymphocytes and CD80 on epidermal Langerhans cells
6 healthy subjects in a study of muscle metabolism - Increased malonyl-CoA directly inhibits
which of the following processes in these subjects -Answer- Fatty Acid Oxidation b. Malonyl-
CoA inhibits the rate limiting step in beta-oxidation of fatty acid.

Over 1 year - study is conducted to assess the antileukemic activity of a new tyrosine kinase
inhibitorwith CML in blast crisis - All patients in the study had SML and are informed that they
would be treated with tyrosinase inhibitor. What type of study design?
-Answer- Open Label Clinical Trial
A 63 year old man - 4 day history of increasingly severe left leg pain and swelling on his
left calf - CT scan shows no abnormalities - A CT scan of the abdomen shows a 3 cm
mass in the body of pancreas - Ultrasound shows a femoropopliteal venous clot - Which
of the following is the most likely cause? -Answer- Hypercoagulability of advanced
malignancy
A 40 year - comes to physician because of 6-month history of facial hair growth - temporal
balding and coarse hair on the upper lip chin - ultrasound shows a 12 cm ovarian mass -
Describe mass? -Answer- Sertoli - Leydig Cell Tumor

35 year old man - pain and swelling in right arm when scraped a tree branch - Temp 101F-
Examination of the right arm shows edema around a lesion - primary mechanism of edema? -
Answer- Separation of endothelial Junction

12-year-old - 2 month history of headaches - 6 day history of nausea and vomiting - bilateral
papilledema - broad based gait An MRI - shows tumor in pineal region. Pt most likely
oculuomotor impairment? -Answer- Upward Gaze Parinaud Syndrome due to compression of
tectumvertical gaze palsy occurs with pineolmas

52 year old man - 1 day history of nausea, vomiting, and right sided abdominal pain - radiates to
his back - CT scan - shows enlarged right kidney and wedge shaped areas of hypodensity - no
nephrolithiasis - strongest predisposing factor? -Answer- Atrial Fibrillation

Wedge Shaped renal infarct -Answer- coagulative necrosis


24 year old - rash from hot tub- the infectious agent causing these findings most likely
began to proliferate in which of the following locations? -Answer- Hair Follicle
b. Hot Tub Folliculitis = Pseudomonas

45-year-old - picture of H. Pylori - epigastric heartburn and weight loss - Which process
likely involved? -Answer- Elaboration of proteases and urease with local support tissue
A 14-year-old - hit with baseball bat - unable to dorsiflex his foot. Which of the following
nerves is most likely injured? -Answer- Common Fibular Nerve (peroneal nerve)
26 year old woman - brought to ED - 8 hour history - severe back and abdominal pain -
persistent vaginal bleeding - Ultrasonography shows 2 cm ectopic pregnancy in the
ampulla - Ampulla ruptured into surrounding tissue. Fluid will most likely be found in th
following locations? -Answer- Pouch of Douglas
e

46 year od with ankylosing spondylitis - treated with NSAIDs drugs - Sulfasalazine treatment has
not resulted in improvement - most appropriate next step in treatment is administration of a
drug that inhibits which of the following? -Answer- Tumor Necrosis Factor
55-year-old with blisters that do not break easily, and no oral lesions - these blisters are most
likely the result of adhesions failure of which of the following? -Answer- Basement
Membrane
47-year-old with organophosphate poisoning - blurred vision, difficult breathing, diarrhea,
muscle weakness, fatigue - Most appropriate immediate pharmacotherapy? -
Answer- Atropine
b. Organophosphate poisoning - DUMMBELLS
c. Crosses blood brain barrier - reverses muscarinic effects in CNS

72 year old - difficulty swallowing, chest pain, and cough- tachypnea and equal pulses
bilaterally- percussion shows dullness over right lung - chest X-ray shows area of opacification
in the lower region of the right lower region of lung - Which is most likely cause of patients
condition? -Answer- Aspiration

62 year old man -abdominal pain - interviewed by physician - used single word and sarcastic
answers. No eye contact - FROWNS - tells physician third time he has been asked question-
Appropriate response? -Answer- You Sound Upset. Tell me a little more about that.
54-year-old - ECG shows P wave and R wave no relation between the two. Which is the
most likely diagnosis? -Answer- Third Degree AV Block
42 year old - constipation, abdominal discomfort, mild fatigue, CA increased, P04 decreased,
treated for renal calculus - PE shows now abnormalities - Most likely cause of patients condition
is a small well defined nodule in which location? -Answer- Parathyroid Gland

72 year old - ER - stroke symptoms - slurred speech and difficulty walking - left sided
hemiparesis - Tongue deviates to right when protruded - proprioception and sensation to light
touch are absent over the left upper and lower extremity - Which site in brain stem is damaged
-Answer- =Area Labeled C
68 year old woman - end stage renal disease - initially she did well - within 3 months
she has been admitted for fluid overload - poor adherence to fluid and salt restrictions -
Which is most appropriate initial response? -Answer- It is tough to change your diet and
fluid intake, but what sorts of things were you doing first when you were following
recommendations
During an experiment - southern blot analysis - Figure showed resulting pattern with DNA
samples isolated from different organs. Southern blot demonstrates? -Answer-
Gene rearrangement
17 year old girl - physical exam shows normal female body habitus - normal breast
development - refuses to have a pelvic or rectal examination.- no axillary or pubic hair most
likely clinical presentation? -Answer- Androgen Insensitivity
16 year old boy - 3 day abdominal pain and vomiting - CT scan shows perforated
appendix - Exam of peritoneal fluid shows which organism? -Answer- Escherichia Coli
45 year old woman 6 month history of hot flashes, night sweats, and insomnia- Physician
informed consent on new treatment - most appropriate next step? -AnswerCould you tell me
the results of the hormone treatment options we have discussed?

A randomized control trial - assess risk of GI adverse risk using azithromycin compared to
erythromycin in the pertussis - 100 children with pertussis enrolled - 50 azithromycin and 50
erythromycin - vomiting among 5 pts in azithromycin group and 15 patients in erythromycin
group - Which represents absolute risk reduction? -Answer- a. 0.2 . 15/50 - 5/50 = .2

34 - year -old woman with a 10 year history of Hep C - progressive fatigue - vitals normal. - liver
biopsy shows hepatocellular injury - infiltration and early fibrosis - which mechanism is most
likely to cause hepatocyte injury? -Answer- Foreign Peptides bound to Class I MHC Molecules
are recognized by CD8+ T Lymphocytes
A 50 year old - high blood pressure - If left untreated, which of the following is most
likely to decrease in this patient? -Answer- Baroreceptor Firing

A 62-year-old with interstitial pneumonitis - compared with a healthy man, analysis of this
patients biopsy is most likely to show the following patterns in the cell population
(Type 1, Type II, and Fibroblast)? -Answer- Type I pneumocytes decreased
Type II pneumocytes increased
Fibroblasts increased
31 year old - with T2DM - comes to physician because of an oozing, foul smelling wound -
patient has crepitant bullae - Gram Statin shows gram-positive rods - The causal organism has
which of the following virulence factors? -Answer- A -toxin
4 month year old with SCID received a bone marrow transplant. - six days later - he
develops a widespread, erythematous, maculopapular rash - Exam of skin shows
diffuse degeneration of basal epidermal cells with mononuclear inflammatory cell
infiltrate- Which is cause of rash? -Answer- Graft vs Host Disease

1 -Answer-

1. 36-year-old F with 2 week history of fatigue, bleeding of the gums, and bone pain. Physical
examination shows pallor, hepatosplenomegaly, and ecchymotic lesions over extremities.
Labs:Hb 8g/dlHt 25%Leukocytes: 36,000segm neutr 4%eosinophils 4%lymph 6%mono 6%
promyelo 80%platelets 25,000Polymerase chain reaction test shows an mRNA corresponding to
the retinoic acid receptor- alfa/promyelocytic leukemia fusion gene resulting from a reciprocal
translocation of chromosomes 15 and 17. Treatment w/ all-trans retinoic acid is started. In
response to the therapy, the fusion protein will most likely attract which of the following
proteins to form a pre-transcriptional complex? -Answer- a. Histone acetylase
b. Histone acetylation allows for relaxation of the DNA
e. Histone Acetylation makes DNA Active
f. Histone Methylation Mostly Makes DNA Mute.

68-year-old man with 3 days of increasingly severe chest pain, shortness of breath, stridor,
hoarseness, difficulty swallowing and nonproductive cough. Long-standing hx of hypertension.
Smokes 2 packs of cigarettes for 45 years. T 99 F, pulse 80, rr 15, bp 160/94. PE shows visible
pulsation above the manubrium of the sternum and displacement of the trachea to the right.
Murmur second right intercostal space. Dx? Answer- Aortic aneurysm male smoker with HTN +
deep chest pain and pulsatile mass = AA all day long

1. 35-year old man w several episodes of squeezing chest pain gets angiogram... gets IV NE.
Question shows a graph of coronary blood flow with a drop after the NE and then a rise.
Which substance causes increased total coronary blood flow 1-2 mins after NE? -
Answer- Adenosine

Adenosine and Nitric oxide (NO) increases coronary blood flow


4. Man comes to doc for cast removal. Fracture of left humerus that required open reduction,
internal fixation, cast immobilization. Muscle strength is 2/5 with extension of elbow and 1/5
with extension of wrist and fingers. Patient most likely sustained a fracture at (which location in
humerus)? -Answer- Radial groove (radial N. is the extensors of arm, wrist and fingers)

Prevalence of high-grade cervical intraepithelial neoplasia in unscreened population i


5%. Prevalence in population with negative Pap smear results is as high 0.2%.

Prevalence decreases, which also decreases? -Answer- Predictive value of a positive


test result

Higher prevalence - higher predictive vales - positive, predictive, prevalance


s

45-year-old woman with joint pain due to rheumatoid arthritis comes for infective treatment
with over-the-counter agents. Initiate disease-modifying antirheumatic drug
(DMARD). Delayed onset of action of DMARD, so physician prescribes another until
DMARD is effective. Drug? -Answer- Prednisone
You can use three classes of medication to treat RA: NSAIDs, corticosteroids, and DMARDs.
NSAIDs and corticosteroids have a short onset of action, while DMARDs take a while to take
effect.
28-year-old man infertility, weight lifter and takes anabolic steroids. Mechanism of
infertility? -Answer- Suppression of Gonadotropins

Anabolic steroid use suppresses the Hypothalamic-Pituitary-Gonadal axis which will


down regulate GnRH.
FA 2020 p636

62-year-old woman with 3-day hx of fever, shaking chills, and left flank pain. Dx acute
pyelonephritis and treated with ciprofloxacin. Five days after, sudden onset watery diarrhea
and lower abdominal cramps. T 100.9 F, pulse 80, rr 18, bp 124/88. PE moderate tenderness
to palpation in lower quadrants especially on right and increased bowel sounds. Stool is
brown and occult blood negative. Next step? -Answer- Test of the stool for Clostridium
difficile toxin b. Pseudomembranous colitis
c. Couple days after Ciprofloxin

72-year-old woman comes to the physician because of a 6-month history of increased


bruising on her forearms.

She appears alert and well nourished. Physical examination shows extensive wrinkling, scaly
erythematous patches on the face, and irregularly shaped brown macules on the face and
forearms.
There are ecchymoses in various stages of healing on both forearms; the ecchymoses
are more numerous on the right side.
Laboratory studies, including a complete blood count and coagulation studies, are within the
reference ranges. S

he has noticed no bleeding from her gums after brushing her teeth. Which of the following is
the most likely cause of the ecchymoses in this patient? -Answer- Extensive solar elastosis
Actinic elastosis, also known as solar elastosis, is an accumulation of abnormal elastin (elastic
tissue) in the dermis of the skin, or in the conjunctiva of the eye, which occurs as a result of the
cumulative effects of prolonged and excessive sun exposure, a process known as photo=aging
]21-year-old woman with 2-days history of urinary frequency and pain with urination. T
39 C (102.2 F), pulse 125, bp 96/60. Urine grows gram-negative bacteria. Virulence
factor for adherence to bladder? -Answer- Fimbriae
Ecoli
73-year-old man has poor appetite and lost 25 lb over 4 months. Labs show normochromatic
normocytic anemia. Xray of chest shows 2-cm perihilar mass. Biopsy shows small cell carcinoma
of lung. Which is responsible for weight loss? -Answer-
Cytokine effect
Cachexia = increased pro-inflammatory cytokines
Mediated by TNF-α, IFN-γ, IL-1, and IL-6

34-year-old man with herniated lumbar intervertebral disc. Laminectomy and removal of hernia
scheduled. CT scan of vertebrae shown, which is surgical entrance location into neural canal? -
Answer- Area labeled D
24-year-old woman overdose on drug X and has serum concentration of 32. Drug X follows first-
order kinetics. 6 hours later, the serum concentration is 16. Cannot be moved from intensive
care unit until concentration 1.1 or lower. Which is minimum number of hours from first blood
sample (32) that patient must remain in icu? -Answer-
30 hours (5 half lives)
b. First Order - Constant Fraction
c. Zero Order - Constant Amount
45-year-old woman dx with cholelithiasis. She asks why abdominal pain is intermittent
and not constant. Pain is produced when gallbladder contracts against gallstone
obstructing cystic duct. Where is hormone released from that causes gallbladder
contraction? -Answer- Enteroendocrine cells of the small intestine (CCK)

CCK = produced by the I cells in the duodenum and jejunum in response to fatty acids
and amino acids.
CCK - cause gallbladder to contract

. 35-year-old with Chagas disease and 2-hour history of moderate chest pain. 4.4 lb weight loss
in 2 months. Vitals stable. Barium swallow shows dilated esophagus with beak-like narrowing at
lower esophageal sphincter (LES). Biopsy shows? -Answer- Loss of neurons in myenteric plexus
failure of LES relaxation, causing distal stenosis of the LES ("bird's beak" sign) proximal dilation
indicative or achalasia

45-year-old man comes to physician for follow up after appendectomy. There is mild scleral
icterus and well healing surgical incision.
Lab values show
Total bilirubin 3.2
Direct: 0.2
Indirect 3 mg/dL
Dx? -Answer- a. Gilbert
Stress related - Mild jaundice with increased unconjugated in an older fellow is decreased UDP-
glucuronyltransferase activity.

35yo F has congestive cardiomyopathy and pitting edema. Her serum urea nitrogen
concentration is 25mg/dl and serum creatinine is 1.8mg/dl. Furosemide therapy is started.
After 5 days, labs show:
Na 130K 4.5Cl 90HCO 30
BUN 85 Creatinine 2.2
Albumin 3Urine specific gravity 1.023, rbc 0
wbc0 sediment none
Urinary fractional excretion of sodium is less than 1%.
Explanation? -Answer- Prerenal azotemia
After the addition of furosemide, her BUN/Cr ratio went up to 38 and Fraction Excretion of Na <
1
33-year-old woman who three weeks ago, underwent oophorectomy for epithelial ovarian
cancer. Recommends adjuvant chemotherapy with paclitaxel. Mechanism? Answer- a. Inhibits
microtubule disassembly
"taxes STABILIZE the economy"

4-year-old boy with fatigue and irritability for 2 months. Family visited rural Louisiana 5
months ago and ran around barefoot. Conjunctivae are pale. Labs shows normal wbc with
15% eosinophils. Stool prep shows parasite egg (picture). Cause of fatigue? - Answer-
Microcytic anemia

Necator or Ancylostoma = running around barefoot (entrance site for laravae

hookworms suck blood from the intestines leading to iron deficiency anemia.

Parasite in US - unable to absorb nutrient like iron


Heart of 76-year-old woman shows concentric enlargement of left ventricle. Micro exam
shows enlarged myocardial cells with large nuclei. Dx that causes this cardiac
enlargement? -Answer- Hypertension

b. concentric ventricular hypertrophy = increased afterload


c. eccentric ventricular hypertrophy = increased preload
d. chronic HTN, Friedreich ataxia
23-year-old woman g1p1, not felt well since delivery 2 weeks ago; ongoing fatigue, inability to
breast-feed, light-headedness in upright position. Pregnancy complicated by preeclampsia and
required cesarean delivery, after she required blood transfusions because of hypotension. PE
appears lethargic. HR 80, BP 100/50, HR standing 85,
BP standing 86/44. Hb 11.6, hct 36%. Serum hormone concentrations? -Answer-
Prolactin decreased
ACTH decreased
TSH decreased
Aldosterone increased
Sheehan syndrome
c. severe bleeding requiring transfusions + inability to breastfeed (decreased prolactin
d. All hormones from pituitary - destroyed

50-year-old man with pulmonary embolus. Treated with intravenous heparin. 24 hours later,
warfarin added. Day 2, partial thromboplastin time is 52 seconds (control 26 sec), and
prothrombin time is 12 seconds (control 12.1 sec; INR = 1). Best explanation for normal
prothrombin time and INR? -Answer- Long half-life of factor II (prothrombin)
Factor II (prothrombin) has the longest half-life
Warfarin acts on Factor II, VII, IX, X

Heparin = factor IIa inhibited by ATIII - thus


low
PTT is prolonged
8-year-old boy with persistent disruptive behavior. Teacher says he's in "constant
motion" and never completes assignments. Treated in ER several times because of
skateboarding injuries. Tx? -Answer- Methylphenidate

sxs in 2+ settings--> school, and ER for skateboarding injuries Tx= methylphenidate


MOA = stimulates NE and DA
65-year-old man with cancer of cecum found to have metastatic lesion to right lobe liver.
Venous route of mets? -Answer- Ileocolic --> superior mesenteric --> portal --> right
hepatic branch of portal

right lobe of the liver = all drain into the Superior Mesenteric Vein.

left sided structures drain into the Inferior Mesenteric Vein.

48yo man with possible hypertension. based on 20 measurements, his average diastolic
pressure is 94mmHg, SD is 8mmHg. If only four measurements were made rather than 20,
which of the following statements would best describe the width of the
95% CI with regard the mean blood pressure -Answer- Larger
Width of confidence interval larger with fewer cases
Width will become smaller - with more cases
98-year-old woman who lives with daughter and grandson comes for routine exam with
type 2 diabetes and hypertension but no complaints. She has had 14-lb weight loss
since last visit 3 months ago. BMI 15. PE shows multiple ecchymoses in various stages
of healing upper extremities and torso. Physician should ask? -Answer- Are you safe at
home?
A transcription factor that activates expression of HMG coA reductase is identified, this TF is
initially synthetized as a large precursor protein, with 2 membrane spanning domains. The
transcriptional domain is released by proteolysis when the membrane has a decreased
cholesterol concentration. The precursor protein is most likely initially produced in which of the
following labeled area on this labeled diagram of a hepatocyte? -Answer- Area labeled F

Endoplasmic Reticulum

18-year-old woman comes 12 hours after ingesting 100 aspirin tablets in suicide
attempt. PE shows tachypnea. Labs?pH/pCO2/HCO3- -Answer- pH 7.32 pCO2 15 HCO3
8

aspirin overdose (early) = Resp Alkalosis

aspirin overdose (Later) = Metabolic acidosis

HCO3- should also be less than normal (22-28)

12 hours = Late Stages


10-day-old male newborn 1-day history red eyes with discharge. Pregnancy uncomplicated
but no prenatal care in third trimester. PE bilateral conjunctival injection with water
discharge. Cause? -Answer- Chlamydia trachomatis b. chlamydia occurs 1-2 weeks after birth
c. conjunctivitis from gonorrhea occurs 2-5 days after birth

54-year-old man with normal renal functions gets a heart transplantation. One year later,
bp 170/110 and serum creatinine 2.1. Which immunosuppressive drug caused these
findings? -Answer- Cyclosporine ADR: hypertension and nephrotoxicity

2-year-old girl with febrile seizure. PE shows nuchal rigidity and bacterial meningitis suspected.
LP and immediate abx therapy planned, but parents unavailable for consent. Next step? -
Answer- Initiate the procedure and treatment without consent

Implied consent in emergency

14-year-old girl with 4 months of heavy menstrual flow. Menarche at 13 with regular intervals.
History of frequent nosebleeds and easy bruisability. Father has problems with nosebleeds and
clotting. PE shows mild gum bleeding and ecchymoses. Labs: hb 8.2,
hct 24.6, platelet 250,000, bleeding time 10, PT 14 (INR 1.5), PTT 60. Dx? -Answer-
von Willebrand disease

prolonged bleeding time + prolonged PTT =


A study is conducted to compare the incidence of myocardial infarction in patients
undergoing two different types of angioplasty or an operative procedure to manage single-
vessel coronary artery disease. A total of 1000 patients are enrolled. Through a chance
process, 500 are assigned to undergo the operative procedure, 250 are assigned to undergo
one type of angioplasty, and 250 are assigned to undergo a second type of angioplasty. All
patients are followed for 3 years to determine the incidence of myocardial infarction. Which
of the following best describes this study design? -Answer- Randomized clinical trial
Study design with randomly assigned experimental and control groups is still a randomized
clinical trial

In cohort study, you don't assign the exposure, you just simply follow them (risk factors).

51-year-old with Graves disease develops ulcerating pharyngitis after 6 months of


propylthiouracil. What's the underlying cause of ulcerative pharyngeal disease? -
Answer- Agranulocytosis
(rare side effect of PTU) commonly presents with acute pharyngitis.

. 22-year-old nulligravid woman has menses that occur at irregular 26- to 32-day intervals.
Height 5 ft 4 in and weighs 187 lb. BMI 32. PE shows mild hirsutism and velvety brown,
thickened skin at the base of the neck and around axillae. Patient has which condition? -
Answer- Hyperinsulinemia b. PCOS
c. Acanthosis Nigrans
3-year-old boy 7 days after dx with severe chronic diarrhea due to Giardia.
Immunological studies show peripheral leukocytes that express both CD3 and CD4 and
fail to express CD40 ligand. Based on this, the immunoglobulin isotype that predominates has
which biological properties? -Answer- Complement activation
Hyper IgM syndrome; without CD40 ligand, T cells are unable to provide the secondary
signal to activate B cell class switching.

prevents IgM from switching into IgD, G, E and A, and thus an accumulation of IgM -
hence the name "hyper IgM" - and deficiency in the rest
Complement activation= IgM and IgG
67-year-old woman comes to the physician because of a 1-month history of low back
pain. She has hypertension well controlled with a thiazide diuretic. Her pulse is 140/min,
and blood pressure is 140/85 mm Hg. Physical examination shows tenderness to
palpation over the L2-3 vertebrae. Laboratory studies show:Hemoglobin 13.5 g/dLHematocrit
41%Leukocyte count 10,500/mm3Segmented neutrophils 65%Eosinophils 1%Basophils
1%Lymphocytes 30%Monocytes 3%Platelet count
250,000/mm3
Serum electrophoresis shows a monoclonal spike of IgG kappa. A chest x-ray shows
cardiomegaly with bilateral pleural and pericardial effusions.
An x-ray of the spine shows a lytic lesion. Echocardiography shows an echodense thickened left
ventricle and poor diastolic compliance. A photomicrograph of a specimen obtained on
myocardial biopsy is shown. Which of the following is the most likely cause of the cardiac
findings in this patient? -Answer- Amyloid infiltration
Multiple Myleoma = monoclonal IgG kappa (light chain) spike on electrophoresis, lytic bone
lesions in the spine, cardiomegaly all point to multiple myeloma.

Amyloid infiltration- Causes a primary amyloidosis that can deposit in multiple tissues,
Investigator studying immune response to fungi. Fungi express beta-glucans on cell
surface and that triggers innate immune response. Which tx decreases glucan
expression? -Answer- Caspofungin

Inhibit cell wall synthesis by inhibiting synthesis of β-glucan


6-year-old boy with 5-day history of intermittent vomiting and 3-month hx of progressive
clumsiness. Can no longer ride bicycle and difficulty getting out of car. Funduscopic exam shows
bilateral papilledema. Neuro exam shows impaired upward gaze and pupil response to light.
Walks shuffling gait. CT shows enlarged lateral and third ventricles and a 2-cm mass. Location of
mass? -Answer- Pineal gland
Pinealoma
Parinaud syndrome (compression of the tectum --> vertical gaze palsy) Obstructive
Hydrocephalus
.34-year-old man burned hands firing pots 3 months ago. He has no pain during or after
burn. PE shows mild atrophy of arms and hands, absence of deep tendon reflexes in
upper extremities, and decreased pain and temperature sensations in C4 to T1
dermatomes. Touch is preserved. Dx? -Answer- Syringomyelia
Cape like distribution of loss of pain and temperature
Touch preserved - lack of degeneration in the DCML.
57-year-old man with alcoholism dies of klebsiella pneumonia. Abscess cavities filled
with purulent exudate on autopsy. Pattern of necrosis in lung tissue? -Answer-
Liquefaction

Abscess (proteolytic enzymes from neutrophils liquify tissue

An investigator conducts a meta-analysis of three genome-wide association studies of


Crohn Disease. The studies encompassed 3200 cases and 4800 controls, all of
European descent. The initial studies identified 11 significant loci with odds ratios above
1.3 and 1.5; the combined meta-analysis identified an additional 21 loci with odd ratio of 1.1 to
1.3. It is estimated that the 32 loci identified explain about 10% of the variance in disease risk
with 2 loci accounting for 2% of the variance. Previous studies of twins indicated a 50%
heritability rate for Crohn's disease. Which of the following best explains these results? -
Answer- The identified loci account for a relatively small part of the variance

5-year-old girls with severe leg pain 1 day after fell off sled. Immigrate to USA from Iceland.
Has had three bone fractures since birth. Parents are vegan and don't give dairy products. PE
shows tenderness, swelling, and limited range of motion of left lower extremity. Xray shows
fracture of fibula. Pt has vitamin deficiency that affects which of the following? -Answer-
Intestinal calcium absorption Vegan
vitamin D deficiency

36-year-old woman diagnosed with HIV. Two months ago, started antiretroviral therapy with
efavirenz, emtricitabine, and tenofovir. Labs: CD4 352 and undetectable HIV viral load. Two
months ago, CD4 count was 158 and plasma HIV viral load was 5500. Next step? -Answer-
Continue efavirenz, emtricitabine, and tenofovir with no changes

If it ain't broke don't fix it.


Typical antiretroviral HIV therapy regime is:

3 NRTIs OR 2 NRTIs AND 1 NNRTI OR 1 Protease inhibitor OR 1 Integrase inhibitor


18-year-old woman get general anesthesia for wisdom teeth extraction. After 5 minutes, gets
hypertonicity of skeletal muscles and increased body temperature. The treatment is a drug that
decreases? -Answer- Sarcoplasmic Ca release
Malignant hyperthermia- rare, life-threatening condition in which inhaled anesthetics or
succinylcholine induce severe muscle contractions and hyperthermia

Dantrolene - TX- Prevents release of Ca2+from sarcoplasmic reticulum of skeletal


muscle by inhibiting the ryanodine receptor
48. 66-year-old man with 1-month hx of nonproductive cough and 6 months shortness of
breath. Bilateral end-inspiratory crackles heard. CT chest shows diffuse reticular opacities at
periphery and bases of lungs. Micro exam of biopsy shows patchy interstitial fibrosis, several
fibroblastic foci and no granulomas. Dx? -Answer- Idiopathic pulmonary fibrosis honeycomb
lung

35-year-old man with several episodes of dizziness and fainting during the past 2 months.
Father and several paternal uncles died suddenly. PE and lipid studies normal.
Angiography shows no coronary artery blockage. ECG shows prolonged QT.
Decreased activity in which of following causes this? -Answer- Outward (delayed)
rectifying potassium channel

QT is correlated with the repolarization (Phase III),


4-month-old boy with 1-day respiratory distress. Has progressive weakness and difficulty
feeding for a month. RR 50/min. PE shows enlarged tongue and generalized hypotonia.
Increased lactate, pyruvate, glucose, and uric acid. CXR shows cardiomegaly. Biopsy of skeletal
muscle shows increased glycogen. Enzyme impaired?
-Answer- alpha-1,4-Glucosidase
Pompe Disease (Type 2)
Lack of - Lysosomal Debranching Enzyme (α-1,6 Glucosidase)

58-year-old woman with 6-month shortness of breath and chronic nonproductive cough. 2-year
history difficulty swallowing, joint stiffness, diffuse tightening of skin on face, neck, shoulders,
arms, fingers. Sensitivity to cold weather, fingers turn white. Hx of esophageal reflux. Biopsy
showed atrophy of epidermis and deposition of collagen throughout dermis with loss of dermal
appendages. PE cutaneous ulceration, clawlike flexion deformity, decreased joint mobility. At
risk for which pulmonary disorder? -
Answer- Pulmonary hypertension systemic
sclerosis/CREST

complications = interstitial fibrosis, pulmonary HTN

CREST -
Calcinosis, anti-Centromere antibody,
Raynaud phenomenon,
Esophageal dysmotility,
Sclerodactyly
Telangiectasia

2-year-old woman has operation for hyperparathyroidism. Three parathyroid glands found but
one does not appear in normal superior location on right side. Embryologic event that led to
this? -Answer- Abnormal migration of endoderm from fourth pharyngeal pouch

4th pharyngeal pouch gives rise to superior parathyroid


EAR Tonsils - Bottom to Top

46-year-old woman with 1-week low-grade fever and joint pain. Has chronic headaches and takes
ibuprofen several times. PE diffuse maculopapular rash. UA 2+ protein, 10-20 WBC and
eosinophils. Renal biopsy would show what? -Answer- Inflammatory infiltrates in the interstitium
acute interstitial nephritis urine eosinophils + NSAID use + Rash
47-year-old woman with irregular, raised, multicolored dark lesion on left forearm with
frequent sunlight exposure. Biopsy shows malignant pigmented cells. Worst prognosis with
involvement of which layer? -Answer- Subcutaneous tissue
highly vascularized and has larger blood vessels than the dermis, leading to a higher risk of
metastasis and worse prognosis (TNM staging system)

Deeper than basement membrane - no longer carcinoma in situ

33-year-old man dx with epilepsy age 10 years. Most recent generalized tonic-clonic was 5
years ago. Medication was adjusted. Current meds include carbamazepine. He's never had any
collisions while driving his motor vehicle. Patient's status with respect to driving? -Answer- He is
medically qualified to drive
most of the states a 1-2-year free seizure period is required for epileptic patients to get their
license

35-year-old from group home comes for worsening behavior for 2 weeks. He believes CIA is
spying on him through television set. Reports hearing voices in hall outside and that CIA now
plans to kill him. Appears disheveled with unkempt hair and poor hygiene, difficulty answering
questions because listening to internal stimuli. Mental status exam will show which? -Answer-
Flattened affect
Schizophrenia
Positive—hallucinations, delusions, unusual thought processes, disorganized speech,
bizarre behavior
Negative—flat or blunted affect, apathy, anhedonia, alogia, socialwithdrawal
45 yr old man with SOB on exertion x 6 months, nosebleeds since adolescence, 2 pics:
clubbing + hemorrhagic lesions in tongue (IMAGE), inhaled albuterol doesn't improve
his symptoms. Cause of symptoms? -Answer- Pulmonary AV shunting
osler weber rendua

hereditary hemorrhagic telangiectasia


55-year-old man northern European descent with 2-month weakness, altered skin color,
bilateral knee pain. Siblings have type 2 diabetes and cirrhosis. PE bronzed skin, hepatomegaly,
arthritis. Increased saturation of transferrin and ferritin. Liver biopsy increased iron content and
cirrhosis Greatest risk? -Answer- Hepatocellular carcinoma hemochromatosis (bronzed skin,
hepatomegaly, arthritis, increased iron content and cirrhosis)
Cirrhosis -> hepatocellular carcinoma

52-year-old female with a history of breast cancer who received 4week course of radiations 6
months ago comes for follow-up. Exam shows no cancer recurrence. Respiratory Rate 26/min.
CT chest shows b/l atelectasis in upper lung fields. Primary pathophysiological cause? -Answer-
Contraction
radiation therapy caused fibrosis and the myofibroblasts "contract" during fibrosis
19-year-old woman 2-day history of pain in left index finger. Injured it when catching a
ball. PE shows erythema of left index finger. Unable to flex the distal phalanx when
proximal interphalangeal joint metacarpophalangeal joints restrained. Xray normal.
Injured structure? -Answer- Flexor digitorum profundus tendon
profound go far - profundus to distal/furthest phalynx
21-year-old woman with 10-days difficulty walking. Two years ago had loss of vision in left
eye which improved. Neuro exam shows decreased visual acuity in left eye with pallor of
optic disc. Has past-pointing on a finger-nose test. Broad-based gait. MRI shows brain lesions
in white matter of cerebellum. Pathogenesis? -Answer- CD4+ T lymphocytes are activated by
myelin basic protein Multiple Sclerosis
increased IgG level and myelin basic protein in CSF
Oligoclonal Bands

69-year-old woman with 3-week history of muscle cramps, weakness, abdominal pain, and
constipation. Hypertension treated with metoprolol and hydrochlorothiazide for past
4 months. Labs show hypokalemia. Which drug should be added? -Answer-
Triamterene
(potassium-sparing) = Keep your SEAT
Spirolactone, Eplenone Amiloride, Triametrene
Work in Distal Tubule Collecting Duct

82-year-old woman comes to the physician because of constant severe lower abdominal pain
and fever for 24 hours. Laparoscopic examination shows severe diverticulosis and perforated
diverticulitis. In spite of appropriate therapy, she dies 2 days later. Liver autopsy shown. Which
of the following is the primary component of the
material shown on the hepatic surface (IMAGE)? -Answer- Fibrin
Fibrin activated - perforated - for coagulation
Fibrin deposition local defense mechanism

58-year-old man comes to physician for benzodiazepine prescription for situation at work.
Feels anxious and thinks he can complete last 2 years of work if anxiety decreases. Gives 1-
month regimen of benzodiazepine. Wife calls, "My husband got fired! I know it was because
he was taking too much of that drug you gave him. Didn't you know he had history of
alcoholism?" Initial action? -Answer- Contact the patient to discuss the situation Reach out to
patient

4-year-old boy (pedigree shown) has clumsy gait for past year. Exam calf hypertrophy and
proximal muscle weakness. Creatine kinase increased. Muscle biopsy shows loss of muscle
tissue, regenerating muscles fibers and fibrosis. Maternal uncle had similar findings and died at
15 years. Patient's sister is pregnant. Ultrasound identifies male fetus. Probability fetus has
disorder? -Answer- ¼
boy you only look at half, so the chance of him having it is 1/2.so 1/2 * 1/2= 1/4 2-year-old boy
2-day history of fever, sore throat, rash. T 101.7 F (38.7 C). Widespread, red, sandpaper-like
rash on extremities. Purulent exudate over tonsils. Dx streptococcus pyogenes (group A)
infection. Cause of skin findings? -Answer- Erythrogenic toxin causes a Toxic shock-like
syndrome Fever, Rash, Shock, Scarlet Fever.
Streptococcus Pyogens (group A strep)
35-year-old woman pain in left leg 2 days. PE shows deep venous thrombosis. Labs:
platelet 200,000, PT 12 (INR 1), PTT 37. Heparin started. Five days later, platelet
120,000.

Reason for decreased platelets? -Answer- Drug-related antibodies


Heparin induced thrombocytopenia:
IgG against platelet factor 4
FA 2020
1-year-old has numerous infections of skin and oral mucosa since birth. Infections slow to
respond to antibiotic therapy. T 100.4F. PE multiple erythematous lesions of skin some with
superficial ulceration. WBC 21,000 77% segmented neutrophils, 6% bands, 14% lymphocytes
and 3% monocytes. Biopsy shows no neutrophils in dermis or epidermis. Culture of lesion grows
Staphylococcus aureus. Defective? -Answer- An integrin
Leukocyte adhesion deficiency = integrin defect
Neutrophils are unable to adhere and migrate to infection sites, instead accumulating in
blood.
A 25-year-old woman comes to the physician because of a 2-year history of intermittent,
diffuse, cramping lower abdominal pain. The pain is usually associated with 2 to 6 days of loose,
watery stools, and is typically relieved with defecation. Between these episodes, her stools are
normal. Her vital signs are within normal limits. Physical examination shows no abnormalities.
Laboratory studies, including complete blood count, metabolic panel, and thyroid function tests
show no abnormalities. A drug targeting which of the following mechanisms of action is most
appropriate for this
patient? -Answer- Accentuation of μ-opioid myenteric plexus receptor Irritable
Bowel Syndrome.
Loperamide = opioid anti-diarrheal

35-year-old woman with infertile, receive injection of contrast material into cervix. On
hysterosalpingogram (shown), contrast material (indicated by arrows) also seen in peritoneal
cavity, which explain this finding? -Answer- Spillage of contrast which normal fallopian tubes -
leak contrast - a gap between fallopian tubes and ovary.

no spillage, that means there is a blockage = infertility

27-year-old primigravid woman at 18 weeks gestation comes to physician for routine prenatal
examination. The uterus consistent in size with 18-week gestation.
Ultrasonography shows a male fetus. The collecting system and pelvis of the left kidney is
dilated and the renal cortex appears compressed. The left and right ureters are not dilated.
The right kidney appears normal. Amniotic fluid volume is normal. Which causing renal finding
in this fetus? -Answer- Incomplete recanalization of proximal ureter ureteropelvic junction, -
LAST to canalize and can lead to congenital obstruction.

25-year-old-man who has a history of facial flushing and rapid heart rate each time he
consumes a small amount of alcohol participates in study of alcohol intolerance. Molecular
analysis shows presence of a lysine (K487, oriental variant) for glutamate (E487, native variant)
substitution in aldehyde dehydrogenase. Kinetic characteristics of the enzyme variants
shown:(NAD+))E487 Km 37 KCat 180K487 Km 5600 kCat
9.5based on these findings, which of the following is the most likely cause of this man's
condition? -Answer- Decreased catalytic efficiency of K487 Catalytic
efficiency is defined as Kcat/KM

21-year-old man loses 15% total blood volume 2 minute after motor vehicle collision. Finding
most likely? -Answer- Increased sympathetic nerve traffic to sinoatrial node

48-year-old woman with gradual onset back pain past 2 weeks. No trauma. Doesn't smoke
cigarettes, drink alcohol or use drugs. Hemogram, serologic studies and urinalysis
unremarkable. X-ray of spine shows two lytic lesions, in T-10 and L-1. Dx? Answer- Metastatic
carcinoma of the breast
Prostate, Breast >Kidney, Thyroid, Lung
Workbench wiped down with alcohol, successfully inactivates viruses with which
characteristic? -Answer- Enveloped virion

Alcohol-based products disrupt the lipid membrane envelope

52-year-old with chronic pancreatitis is deficient in which enzymes that causes inability
to digest triglycerides? -Answer- Colipase
cofactor for pancreatic lipase,

35-year-old primigravid woman 36 weeks' gestation with 6-hour history of heavy vaginal
bleeding. No prenatal care. Ultrasound shows placenta over cervical os. Can't stop
bleeding and has cesarean. Dx? -Answer- Placenta previa
Opening of Cervical OS - preview of Placement
45-year-old woman with 4-month joint pain, muscles aches and fatigue. Had a renal calculus 6
months ago and had cholecystectomy 1 year ago. Appears fatigue, depressed, there is mucosal
pallor. No mass45-year-old woman with 4-month joint pain, muscles aches and fatigue. Had a
renal calculus 6 months ago and had cholecystectomy 1 year ago. Appears fatigue, depressed,
there is mucosal pallor.
No masses palpated in neck.
Labs: erythrocyte count 3 million, K+ 4, Cl- 106, Ca2+ 13, phosphorous 2.7, alk phos
125.
Technetium-99m scan shows 1.2 cm nodule in neck. Mutation?s palpated in neck.
Labs: erythrocyte count 3 million, K+ 4, Cl- 106, Ca2+ 13, phosphorous 2.7, alk phos
125. Technetium-99m scan shows 1.2 cm nodule in neck. Mutation? -Answer- MEN1 Pituitary,
Pancreatic, Parathyroid - 3ps = MEN 1

Parathyroid tumor releasing PTH, increasing Ca and ALP.

6-year-old girl with 15-minute history of severe shortness of breath. Diagnosed with throat
tumor 3 years ago. RR 32. PE nasal flaring. Laryngoscopy shows multiple raised, finger-shaped
lesions from vocal cords and epiglottis. Lesions excised and shows finger-shaped fibrovascular
cores lined with benign squamous epithelium. Causal virus?
-Answer- Human papillomavirus, type 6
Laryngopapiloma

A 68-year-old man comes to the physician because of a 1-month history of light- headedness
and tightness in his chest with exertion. He adds that the pain is worse after arguing with his
wife, and the symptoms resolve with rest. He has a past history of lower gastrointestinal
bleeding; evaluation at that time was negative on upper endoscopy and colonoscopy. His
temperature is 37°C (98.6°F), pulse is 85/min, respirations are 15/min, and blood pressure is
110/75 mm Hg. Physical examination shows no abnormalities. His hemoglobin concentration is
8.2 g/dL, and hematocrit is 24%. Test of the stool for occult blood is positive. An ECG shows no
abnormalities. Repeat colonoscopy shows no abnormalities. Which of the following is the most
likely cause of this patient's gastrointestinal symptoms? -Answer- Angiodysplasia
Hematochezia is the passage of fresh blood per anus
Tortuous dilation of vessels hematochezia

24-year-old man with 3-day progressive numbness of both feet ascended to thighs. Last 24
hours, numbness and tingling of hands. PE ataxic gait. Deep tendon reflexes diminished in
upper extremities and absent in knees and ankles. Vibration and joint position absent in
fingertips and feet bilaterally. Mild weakness distal upper extremities ad moderate weakness of
lower extremities. Structure involved? -Answer- Myelinated primary afferents
Most common subtype of Guillain-Barré syndrome

27-year-old man for psychiatric eval sent by employer because he is "very odd." Is a
computer repair specialist and lives alone, Refuses to socialize and has no friends.
Extremely preoccupied with science fiction, occult, afterlife. Personality disorder? - Answer-
Schizotypal
Eccentric appearance, odd beliefs or magical thinking
Cluster A
45-year-old man bmi 26, total cholesterol 200, HDL 50, triglycerides 550. Which drug to
prescribe? -Answer- Fenofibrate
Triglycerides 550

MOA = Activates PPAR-αto induce HDL synthesis


60-year-old man for routine health examination. Has had normal blood pressure
measurements. BP today 170/95mmHg. Physical examination shows no other abnormalities.
Serum show hypokalemia and metabolic alkalosis. Plasma renin activity and serum aldosterone
concentrations are increased. Following the administration of captopril, there is a marked
increase in plasma renin activity. Which of the following is the most likely cause of the findings
in this patient? -Answer- Renal artery stenosis
Decreased renal perfusion (one or both kidneys)
Increased renin
Increased angiotensin ---HTN

Mismatching of ventilation and perfusio

insufficient ventilation to provide the oxygen needed to oxygenate the blood


After operation, 65-year-old patient has lung region that is under ventilated but well
perfused. Which increases? -Answer- Physiologic shunt
-
-

Physiologic shunt -Answer- -Mismatching of ventilation and perfusion with the lung -Results in
insufficient ventilation to provide the oxygen needed to oxygenate the blood flowing through
the alveolar capillaries
Physiologic dead space -Answer- portion of the tracheobronchial tree that does not
participate in gas exchange
63-year-old man 2-week sensation of fullness in left upper quadrant. Has lethargy and
shortness of breath. 20-lb weight loss during 3 months. PE shows pallor. Spleen tip palpated.
Labs: hb 9, hct 27%, wbc 4000, serum uric acid 15. Peripheral blood smear numerous
erythrocytes with abnormal shapes and sizes, nucleated erythrocytes and myelocytes.
Aspiration of bone marrow dry tap. Biopsy shows thickened bony trabeculae with increased
reticulum. Dx? -Answer- Myelofibrosis "Bone marrow cries because it's fibrosed and is a dry
tap"

17-year-old boy brought by mother with concern that puberty is delayed. When mother leaves
room, patient states, "I'm fine. I don't know what's the matter with her. She wants me to be tall
like my dad." Patient is 175 cm (5 ft 9 in) tall and weighs 70 kg (155 lb); BMI 23. Sex
development is Tanner stage 4. After reassurance, Most appropriate next statement to mother
-Answer- Tell me more about your concerns about your son's height

1-year-old boy with rash for 2 weeks. 10th percentile for height and weight. PE scaly, seborrheic
eruption over scalp, palms, back, diaper region and soles of feet.
Generalized lymphadenopathy and hepatosplenomegaly. Xray of skull shows osteolytic lesions.
EM biopsy of skin shows tennis racket-shaped bilamellar granule in cytoplasm. Immuno studies
show CD1a antigen expression. Abnormal cells in patient are derived from which cell? -Answer-
Dendritic cells
Langerhans cells (LC) are tissue-resident dendritic cells of the skin, and contain organelles

2-month-old boy well child exam, mother with no concerns. 25th percentile for length and
30th percentile for weight. Cardiac exam shows blowing holosystolic murmur best heard at
lower left sternal border. Cause? -Answer- Ventricular septal defect Blowing Holosystolic
Murmur at Lower left sternal border

Study designed to test the effectiveness of a new drug in the treatment of endometriosis, 100
women randomly assigned to one of two groups. 48 of women receive new drug, 52 receive
standard therapy. The primary purpose of this method of assigning patients to different groups
is to create which of the following? -Answer- Two groups with similar underlying characteristics

A 70-year-old woman comes to the physician for a routine pelvic examination. During speculum
examination of the vagina and cervix, the Valsalva maneuver causes a bulge of the anterior
vaginal wall. Which of the following is the most likely cause of this finding? -Answer- Cystocele
prolapsed bladder, is a medical condition in which a woman's bladder bulges into her vagina.
Some may have no symptoms

32-year-old man with HIV infection follow up examination, has been treated with HAART for
the past 6 years HIV plasma viral load has been undetectable. HIV viral load now increase,
antiretroviral resistance suspected. HIV genotype analysis confirms that the virus has resistance
mutations, which of the following most likely mutated? -Answer-
Reverse transcriptase and protease
HAART therapy = reverse transcriptase and protease enzymes.
32-year-old man with X-linked recessive disease has deafness, hematuria and
progressive renal failure. Protein abnormality? -Answer- Type IV collagen
Alport Syndrome
Can't See, Can't pee, Can't hear a Bee
65-year-old woman with 6 hour Hx fever and shaking chills. 4 hours ago, she took 325
mg aspirine tablets. Temp 39.4 C, pulse 96/m, respiration 18/m and blood pressure
102/60 mmHg. Physical examination shows marked tenderness bilaterally in the
costovertebral areas.
Lab studies show:
Hb 13 g/dlHt 39%
Leukocyte count 32,.000/mm3
Urine ph 6
Nitrites 4+
Leukocyte esterase 4+
A photograph of wright stained peripheral blood smear, which the most likely cause this
patient leukocytosis? -Answer- Leukemoid reaction
25-year-old man just returned from work as worker from Africa begins oral chloroquine therapy
for malaria caused by Plasmodium vivax. His initial therapeutic response is good, but he
develops recurrent parasitemia 2 months later. Which of the following best explains the
recurrence? -Answer- Chloroquine is ineffective on the exoerythrocytic malaria tissue stages
P vivax/ovale, add primaquine for hypnozoite
Hypnozoite = inactive dormant - only in P. Vivax and Ovale
28-year-old woman comes to the physician because of a 4-day history of palpitations, severe
neck pain, fatigue, and malaise. Her pulse is 120/min and regular. Physical examination shows a
diffusely tender, mildly enlarged thyroid gland. There is no exophthalmos. Serum studies show
a thyroid-stimulating hormone concentration of 0.01 μU/mL. Which of the following is the most
likely diagnosis? -Answer- Subacute granulomatous thyroiditis de quervain's = pain

68-year-old man with a 10-month history of shortness of breath and swelling of his feet, family
history of cardiovascular disease. He smoked 2 packs of cigarettes daily for 50 years. Pulse
80/min, rr 24/min, BP 150/80. PE: 3+ pitting edema of lower extremities. Diffuse, scattered
wheezes are heard bilaterally on auscultation of the chest. Grade 2/6 pansystolic mumur heard
best at lower left sternal border, which increases on inspiration. Maximal impulse palpated in
sub-xiphoid area. S1 and S2 sounds are distant. Liver span 14 cm. Diagnosis? -Answer- Cor
pulmonale
Backfilling of blood from the lungs into the R ventricle is stretching out the R side
(dilation) and also remodeling the heart via hypertrophy
cor pulmonale -Answer- right ventricular hypertrophy and heart failure due to pulmonary
hypertension
17-year-old boy in septic shock unresponsive to ADH (vasopressin). Treat is discontinued,
and high-dose dopamine in started. Which receptors are stimulated? Answer- alpha1-
adrenoreceptors low doses d1 = dilates renal vessels medium b1 agonist = ionotropic and
chonotropic high doses a1 agonist = vasoconstriction

8-year-old girl is brought to the physician by her mother because of a 3-week history of poor
feeding chronic diarrhea and pale foul-smelling stools. Mother says the symptoms began with
colicky abdominal pain following introduction of solid food to the infant's diet.
History of gluten sensitivity. 60th percentile for length and 25th percentile for weight.
Which cell is dysfunctional? -Answer- Enterocyte
Celiac disease, an autoimmune intolerance to gliadin leading to enterocyte damage and
subsequent malabsorption

25-year-old woman comes for counseling prior to conception. Has seizure disorder on
valproic acid. Fetus at greatest risk for drug-related adverse effect during which
pregnancy stage? -Answer- 3 to 8 weeks

Neural tube defects Neuropores fail to fuse (4th week)

16-year-old boy who frequently thinks about sex, daydreams about girls before going to
sleep, and masturbates one to two times daily. After counseling about safe sex, best
next step? -Answer- Schedule next routine examination
Normal behavior for a teen boy
35-year-old woman with 3-month progressive shortness of breath with exertion. RR 26
Physical shows jvd and prominent a wave. Lungs clear. Cardiac exam shows loud
pulmonic component of S2 and right-sided S4 gallop. Increased pulmonary expression
of what? -Answer- Endothelin-1

pulmonary hypertension = imbalance between vasodilation substances and


vasoconstrictors.
.

7-year-old boy is about undergo an appendectomy. An intravenous catheter needs to be


inserted, but the patient is fearful of being stuck with needle. The most appropriate anesthesia
administered by mask to anesthetize this patient quickly would have which of the following
characteristics? -Answer- Low blood solubility
--> faster onset --> faster recovery
Lipid solubility determines potency

66-year-old man dies 7 days after myocardial infarction. Gross of heart shown (perforated
interventricular wall). Histology? -Answer- Erythrocytes, cellular debris, macrophages, and early
granulation tissue
0-24 hours Dark Mottling
3-14 days - Macrophages, then granulation tissue at margins
68-year-old man with creatinine 2.3 due to chronically increased hydrostatic pressure in
Bowman space. Cause? -Answer- Benign prostatic hyperplasia

A sexually active 32-year-old woman has vaginal pain with urination. Pelvic examination
show bilateral vesicoulcerative lesions of introitus. Tx? -Answer- Acyclovir
inhibit viral DNA polymerase by chain termination
26-year-old man comes in 2 hours after injuring arm skiing. Sensation to pinprick absent over
lateral aspect of shoulder. Xray of right shoulder shown (fracture of surgical neck of humerus).
Nerve damaged? -Answer- Axillary fracture of the surgical neck of the humerus. axillary nerve
and the posterior circumflex humeral artery
Radial does lateral
elbow
musculocutaneous does
lateral forearm
Patient making sexual advances towards
physician. Appropriate measure? -Answer-
Have a chaperone join them for the
remainder of the examination
Inappropriate behavior - sexual advances towards physician

A 70-year-old man comes with skin blistering for 1 week. No oral lesions. Physical shows tense
bullae in joint folds of upper and lower extremities. Biopsy shows subepidermal blister
formation. Immunofluorescence microscopy shows antibodies against proteins at the dermal-
epidermal junction. Target by antibodies? -Answer-
Hemidesmosome Subepidermal blister formation
+ Tense Bullae

An experimental animal is created that has a defect in an innate gastrointestinal defense


mechanism. Organism is found to have decreased HCl prod. After 2 months on biopsy gastric
fundus and body show decreased mucosal thickness and hyperplasia of enterochromaffin like
cells. This resembles? -Answer- Chronic Gastritis mucosal inflammation leading to atrophy
56-year-old man comes with vague right-sided flank pain and 7-lb weight loss over past
6 weeks. No recent trauma or back strain. Urine is darker than usual over 2 weeks.

Renal ultrasound shows 1.5-cm solid mass in upper portion of right kidney. Urine shows
50 erythrocytes, 6 leukocytes, no bacteria. Dx? -Answer- Renal Cell Carcinoma
hematuria, palpable masses, 2° polycythemia, flank pain, fever, weight loss
67-year-old woman with atrial fibrillation with sudden onset severe abdominal pain. Ex- lap
shows embolus in superior mesenteric artery with complete occlusion of middle colic artery.
Ischemic changes where? -Answer- Small intestine, ascending colon, and part of the transverse
colon
Midgut = major duodenal papilla (of the duodenum) to the proximal 2/3 of the transverse
colon.
Persons are at risk of influenza are vaccination every year because of antigenic variation,
which can be drift (minor) or shift (major). Mechanism of antigenic shift? Answer-
Reassortment viruses with segmented genomes (eg, influenza virus) exchange genetic
material. 2 viruses making a combo virus - 2 genomes

34-year-old man with a 3-month history of a progressive rash on his feet. Rash is not itchy or
painful. A photograph of feet is shown = Kaposi Sarcoma. HIV+. In addition to
HAART, which of the following pharmacotherapy is most appropriate? -Answer- Antineoplastic
Endothelial malignancy most commonly affecting the skin, mouth, GI tract, respiratory
tract.
Associated with HHV-8 and HIV

Kaposi typically presents in HIV pts earlier, under a 500 CD4 tcell count,

bartonella angiomatosis presents <100


70-year-old man with early morning awakening, decreased energy, difficulty concentrating,
anhedonia, psychomotor retardation, depressed mood for 3 months. He had myocardial
infarction and non sustained ventricular tachycardia 2 years ago.
Tx? - Answer- Paroxetine
SSRIs are first line for depression.
Avoid TCAs in elderly = anticholinergic side effects.

29-year-old woman comes to the physician because of irregular menstrual periods since
menarche at the age of 12. She is 5.3 Ft and weighs 86kgs. BMI 34. She is evaluated and a
diagnosis of PCOS is made. After explaining the diagnosis, the physician discusses behavioral
changes, including dietary modification and exercise as part of her treatment. Which of the
following will ensure adherence? -Answer- Provide follow ups to monitor progress in attaining
her goal

Improve Doctor patient relationship - add next appointment

52 yo man is brought to er 30min after the onset of chest pain and shortness of breath. He had
played tennis all day and he does not remember how much fluid he had consumed. His
temperature is 36.7 oC, pulse 122min, respirations 28min and BP 90/50 mmHg. PE shows dry
skin and decreased capillary refill. An ECG and evaluation of cardiac enzymes show no
abnormalities. Which of the following findings in the nephron best describes the tubular
osmolarity, compared with serum in this patient? -Answer- PCT Isotonic,
Macula Densa hypotonic
Medullary Collecting Duct hypertonic
3-week-old male newborn with recurrent vomiting after feeding since birth, and eager to feed
after vomiting. Abdominal exam shows firm, mobile mass in epigastrium to right of
midline. Dx description? -Answer- Single Primary Development Defect
pyloric stenosis - olive shaped mass - stand-alone defect
VACTERL -Answer- vertebral defects, heart defects, renal and limb abnormalities

65-year-old with sudden onset generalized tonic-clonic seizures. Personality change last 6
months; used to be mild mannered and now verbally abusive. CT shows single mass enhances
with contrast in right frontal lobe and crosses to left hemisphere through corpus callosum.
Dx? -Answer- High-grade fibrillary astrocytoma
Glioblastoma multiforme- "crosses hemisphere"
GFAP +
A new antiplatelet agent is developed for the prevention of recurrence of stroke. In a large
randomized clinical trial with equal numbers of men and women, the rates of stroke are lower
in patients receiving the new agent than in patients receiving the standard treatment. Results
are shown:Recurrent Stroke Rates per 1000 Person-
YearsStandard Treatment vs. New Antiplatelet Drug
Women .12 .04 Men
.24 .08
Overall .18. .06
Based on these results, which of the following is the relative risk reduction in women? Answer-
67%
.12-.04= .08
.18-.06 = .12
.08/.12 = 67%

22-year-old man who is a professional cyclist undergoes extensive physiologic testing as part of
his training regimen. His resting pulse is 33/min, and blood pressure is110/62 mm Hg.
Echocardiography shows dilated ventricles with normal function and a left ventricular ejection
fraction of 75%. Which of the following best describes the findings in this patient? -Answer-
Eccentric Hypertrophy
ECCentric - ECCpands ventricle
CONcentric - CONstricts ventricle

68-year-old man with alcoholism comes for fever, chills and productive cough of purulent
sputum for 3 days. Blood cultures positive. Gram stain: gram-positive, lancetshaped
diplococci. Vaccine is against which bacterial component? -Answer- Capsular polysaccharide
Strep pneumonia

Child with XXY karyotype, genetic studies showed he received the extra "x" from his father. An
error of chromosome segregation occurred during anaphase at which of the following stages of
spermatogenesis in the patient's father? -Answer- Primary Spermatocyte
Most problems with cell division occur in anaphase of Meiosis I.
Meiosis 1⇒from primary to secondary spermatocyte Meiosis 2 → Secondary to spermatid'
28-year-old man with ulcerative colitis, history of partial colectomy, and improved rectal
bleeding and diarrhea since then. Meds include mesalamine, hydrocodone and
acetaminophen. He says, "I've been taking so much of my opiate prescription that I ran
out and stole some from my job." Next step? -Answer- Discuss treatment options and a
referral for detoxification program

67-year-old man with 1 year progressive difficulty writing and walking. Pt is stooped and
talks slowly. PE shows bland facial expression, fine resting tremor in both hands, no
tremor when moves, walks with difficulty starting and stopping, cogwheel rigidity. Brain
tissue histology shown as well as gross cross sections of midbrain both normal and
diseased. Substance referred to by arrow. Which Substance? -Answer- Alpha-synuclein
8-year-old boy with pain on back and head since he fell off swing. Tender 2.5cm swollen mass
over right occiput. CT shows osteolytic and soft-tissue mass in skull with inward displacement
of dura. Biopsy shows sheet-like infiltrate of pale eosinophilic cells with bean-shaped nuclei.
Cells positive for CD1a. Electron microscopy of cells shows
Birbeck granules. Abnormal cell type? -Answer- Langerhans cells
30-year-old man in bicycle collision and hits right shoulder forcefully. Unable to flex right
elbow with decreased sensation to pinprick over right lateral forearm. Brachial plexus
lesion? -Answer- E (musculocutaneous nerve)
8-year-old man comes with cracked lips and peeling sunburned skin. Works as lifeguard. PE
shows desquamation of sunburned skin. Lips are dry and cracked. Petrolatum to lips may
reduce lip symptoms by which of following effects of the compound? -Answer- Barrier

During a clinical study of calcium and phosphorus metabolism, a 50-year-old man undergoes
series of lab studies. His serum Ca, PO4 and PTH are normal. He is given infusion 2 g Calcium
chloride over 2 hours. His serum Ca concentration now is
11.5mg/dl. Compared with pre infusion levels, the serum concentration of which of the
following substances is likely to be increased at this time? -Answer-
24,25Dihydroxycholecalciferol inactive metabolite preferentially synthesized
when normal

30. 35-year-old woman with 2-day history of blistering lesions on sun-exposed face, arms and
hands. Recurrent episodes of skin lesions over several years. Taking oral contraceptives for 15y.
PE shows fluid-filled vesicles and bullae. Labs: AST increased, ALT increased, total porphyrin
increased, urine uroporphyrin III increased. Precursor to uroporphyrin? -Answer- Succinyl CoA

32-year-old woman brought in after 10-foot fall from ladder. PE suggests severe hemorrhage
and shock. Xray shows fracture of left ninth and tenth ribs. Organ injured? Answer- Spleen

4-day-old boy with vomiting after breastfeeding. PE shows lethargy and dry mucous
membranes. Labs: Na 139, Cl 90, K 7, HCO3 17, Glucose 42, BUN 25, Cr 0.4, 17-
hydroxyprogesterone increased. Enzyme deficiency? -Answer- 21-hydroxylase infancy (salt
wasting) or childhood(precocious puberty

70-year-old woman comes to the physician because of increasingly severe pain in the right knee
over the past 3 months. She has fallen repeatedly. She has a 10-year history of symptomatic
osteitis deformans. She has bowed tibia and tenderness of the proximal right tibia. an X ray
shows a fracture of the proximal tibia with elevated periosteum and sunburst pattern. an x-ray
of lungs shows pulmonary nodules of various sizes. which of the following findings is most likely
on biopsy? -Answer- Pleomorphic neoplastic cells producing new woven bone
Osteosarcoma histological appearance.
Paget disease is one of risk factor

Cholera toxin catalyzes transfer of ADP-ribose to an arginine residue in stimulatory G protein


resulting in inhibition of GTPase activity. Which increases in cells as a result? Answer-
Concentration of cAMP
10-year-old boy with 4 days nosebleeds and easy bruising. Three weeks ago, upper respiratory
tract infection. PE ecchymoses on upper and lower extremities. Labs shows hemoglobin and
leukocyte count normal, and decreased platelet count. Bone marrow smear shows increased
megakaryocytes. Mechanism of dz? -Answer- Antibodies directed against glycoprotein (Gp)
IIb/IIIa complex Immune thrombocytopenia

60-year-old man 1 day of fever, chills, confusion and memory loss. Returned from Gulf coast
where he walked barefoot. Hx of severe cirrhosis and portal hypertension. T 39C (102.2 F), RR
22, bp 90/48. Physical shows early blister formation on right lower extremity. Blood culture:
gram-negative, lactose-fermenting organism. Bug? -Answer- Vibrio vulnificus

45-year-old man with hypertension not compliant with medications. bp 160/100. Cardiac exam
shows apical impulse displaced laterally, loud S2 and S4 gallop. Echo shows thickening of left
ventricular wall. Mechanism of change in cardiac muscle? -Answer-
Transcription factor c-Jun: increased
beta-myosin heave chain: increased
endothelin: increased
Newborn is found to have a cervical rib. Transformation of seventh cervical segment to thoracic
identity. Which is true of HOX gene alteration? -Answer- Expression of a HOX
gene normally expressed only caudal to C7
30-year-old woman with a long-standing history of pelvic inflammatory disease has surgical
resection of a scarred segment of a fallopian tube. Which of the following inflammatory cells is
most likely to be found in the resected specimen? -Answer- Macrophages

39-year-old man with 1-week of red spots on shins, joint pain and fatigue. PE shows purpura
over lower extremities. Liver palpated 4 cm below costal margin. Labs: WBC 10,000, AST
142, ALT 154, hepatitis C virus RNA positive, anti-hepatitis C virus antibody positive,
cryoglobulins positive, C4 120 (N=350-600), urine protein 4+, urine
RBC numerous. Hypersensitivity reaction? -Answer- Type III (immune complex-
mediated)
25-year-old woman with polycystic kidneys and 3-month history of weakness, fatigue,
headaches, hypertension, loss of appetite and itching. Cr 4. Labs -Answer- HCO3- decreased -
no resorption
PO4 increased,
PTH increased

5-year-old boy with motion sickness. Planning vacation to Australia, and wants
diphenhydramine for motion sickness. Mechanism of action for motion sickness? -
Answer- Antagonist at muscarinic-3 (M3) receptors
63-year-old woman with 2-week history of daily episodes of severe, lancinating, left-
sided facial pain. Pain lasts 30 to 60 seconds and shoots down ear along jawline.
Precipitated by chewing or brushing teeth. Dx? -Answer- Trigeminal neuralgia
45-year-old homeless man found unconscious. Breath smells of alcohol. Vitals stable. PE shows
bronzed skin and spider angiomata on chest. Labs: hemoglobin 10, hematocrit 30%, MCV 110,
WBC 9000, platelets 160,000, ferritin 200, b12 500, folate 20. Blood smear shows
hypersegmented neutrophils and 3+ oval macrocytes. Labs? -
Answer- Methylmalonic acid: normal
Homocysteine: increased
55-year-old man with elevated LDL cholesterol is prescribed lovastatin. This treatment
would result in which adaptive responses at the cellular level? -Answer- Increased
transcription of HMG-CoA reductase
63 yo man with mild emphysema has smoked 1 pack for 45 years sudden headache, myalgia
and rising temperature. A dry cough develops, with chill and chest pain. Examination of gram
stain of sputum disclose neutrophils, but bacteria are not evident. An X-ray of the chest is
consistent with severe pneumonia. A culture of sputum is negative, but culture of
bronchoscopy specimen on a highly specialized bacteriologic medium yields gram negative
rods. The identity of these bacteria is confirmed by staining with specific fluorescent antibodies.
The patient respond therapy with macrolide
Ab. Mechanism? -Answer- Inhalation of aerosol from an environmental source (Legionella) -
Aerosol transmission

50-year-old woman with COPD comes with 3 months of progressive shortness of breath.
Physical shows JVD, loud pulmonary component of S2. Pulmonary function tests show FEV1:FVC
ratio of 20% and decreased diffusing capacity for carbon monoxide. Which is decreased in
pulmonary vascular smooth muscle? -Answer- Endothelial nitric oxide synthase production
63-year-old man with abnormal blood pressure. 6 months ago, bp was 135/85. Today bp
170/98. Bruit heard over left renal artery. CT angios hows left arterial stenosis. Labs?
-Answer- Total peripheral resistance increased,
Plasma renin activity increased,
Serum aldosterone concentration increased

renal artery stenosis.

20-year-old woman at 27 weeks' gestation is admitted to the hospital because of a 12hour


history of intense uterine contractions occurring every 8 minutes. Her membranes ruptured 32
hours ago. Her temperature is 39.1°C(102.4°F), and pulse is 115/min.
Physical examination shows tenderness of the uterus. Pelvic examination shows a closed cervix
that is not effaced. The fetal heart rate is 210/min. Which of the following is the primary
stimulus for her uterine muscle contractions? -Answer- Inflamed maternal decidua release of
prostaglandin Preterm Premature Rupture Of Membrane

30-year-old woman with 2-years of numbness, blanching, and bluish color to ears, fingers and
toes after emotional upset or cold exposure. Vitals and PE normal. Avoid taking which drug? -
Answer- Phenylephrine
18-month-old girl with 2 day progressive cough and hoarseness. T 102.2F, HR 88, RR
24, bp 100/70. Oxygen saturation 95%. PE mild erythema of oropharyngeal and
laryngeal mucosa no exudate, harsh, barking cough heard. Improves within 4 days.
Cause? -Answer- Parainfluenza
Patient with orthostatic hypotension, loose stools for 1 year, and history of type 1 DM.
Stool studies are normal. What is the pathophysiological mechanism of the diarrhea? Answer-
Motility disorder
39-year-old man with polycystic kidney disease has 6-month history intermittent blood in
urine. T 98.6 F HR 100 RR 24 BP 160/90. Urea nitrogen 100, creatinine 8. UA shows
blood. Arterial blood gas shows? -Answer- pH 7.22
pCO2. 28
HCO3- 11
Uremia: part of MUDPILES

Bad kidneys = Can't reabsorb HCO3 "base" >


Metabolic Acidic with respiratory compensation
20-year-old woman with 2-week fever, shaking chills, headaches, fatigue, and joint and
muscle pain. Spent summer workin as lifeguard in Long Island, New York. Has
splenectomy for motor vehicle collision 6 years ago. T 102.4 F. Peripheral blood smear
shows small intraerythrocytic rings. Polymerase chain reaction for Plasmodium is
negative. Causal organism from bite of? -Answer- Tick

Male newborn delivered at 28 weeks, neonatal respiratory distress syndrome, ABG shows
decreased pH, decreased Po2 increased PCO2. A deficiency in which of
the following most likely caused the disorder? -Answer-
Dipalmitoylphosphatidylcholine
55-year-old woman who is a physician has fractured femur during motor vehicle collision. 2
days after admission, has tachycardia, restlessness, diaphoresis, anxiety, seeing "vague shapes"
on walls. Dx? -Answer- Alcohol withdrawal

Full-term newborn in respiratory distress. Imaging shows abdominal contents in left pleural
cavity. Maldevelopment of which structure led to diaphragm defect? -AnswerLeft
pleuroperitoneal membrane
Diaphragmatic hernia

To decrease risk for cv disease 24 yo man begins diet. 1.53 95 kg BMI 32, Intends to lose 16 kg
by limiting caloric intake to 2000 cal. to maintain the recommended protein intake (56g day); a
balanced decreased in carbs and fat is required (caloric radio of fat and carbs is 30:55). which
best describes number of calories that should be provided by fat in this its diet each day? -
Answer- 630
0.3 fat x 2000 = 600 calories (fat)
(Fat 30 + Carbs 55+15 = 100)
35-year-old woman passed out while washing dishes. Began to feel weak and dizzy, husband
shares with nursing staff history of bulimia nervosa, and has been binging more frequently.
Suspected she has been vomiting. Labs of patient? -Answer- K+ decreased HCO3- increased/
Anion Gap normal pH increased'
Metabolic Alkalosis > loss of HCL
Anion Gap Normal = NOT a MUDPILE
HCL, NA+ = out the mouth
Retrospective study of incidence and outcome of subarachnoid hemorrhage. Incidence
is 15 per 100,000 people, mortality is 6 per 100,000 people. Incidence remains
constant, case fatality rate? -Answer- 40%

16-year-old girl calls the physician on a Friday night 2 hours after a condom broke
during sexual intercourse with her boyfriend. She asks the physician to prescribe an
emergency oral contraceptive. The physician on call is not the patient's regular
physician and does not dispense emergency contraception for moral reasons. After the
physician respectfully informs the patient that he does not prescribe this contraceptive, it
is most appropriate for the physician to state which of the following? -Answer- "I can
have one of my colleagues call you back to further discuss your concerns"
58-year-old man comes to the physician because of a 4-year history of recurrent cough
productive of increased sputum. Use of over-the-counter cough suppressants has not resolved
his symptoms. He has smoked 2 packs of cigarettes daily for 35 years. He has no family history
of lung disease. His temperature is 37°C (98.6°F), pulse is 72/min, and respirations are 18/min.
Physical examination shows cyanosis. Diffuse wheezing is heard on auscultation. Which of the
following pulmonary cell types is most likely to be
abnormal in this patient? -Answer- Pseudostratified columnar epithelial cells
Smoker, Cyanosis - chronic bronchitis squamous
metaplasia

64-year-old with non-Hodgkin lymphoma and 3-day history of abdominal pain and nausea. T
99.7F, HR 100, bp 130/80. Abdominal exam tenderness of flanks and lower quadrants. BUN 34
and creatinine 3.8. CT shows bilateral hydronephrosis and lymphadenopathy compressing
ureters. Tx to improve renal function? -Answer- Bilateral stents in the ureters

34-year old man is brought to the ER semiconscious and combative. In addition to sedation, a
short-acting neuromuscular blocking agent is administered for intubation to prevent
aspiration. Within a few seconds after admin of the drug, he has transient muscle
fasciculations in his face; he develops generalized paralysis within 1 minute. Forty five
minutes after completion of the procedure, he is still paralyzed. A genetic abnormality of
which of the following enzymes is the most likely cause of his unusually slow recovery from
paralysis? -Answer- Pseudocholinesterase Succinycholine - Acetylcholine Agonist increased in
system --> longer anesthesia recovery

Gy67-year-old man who eight months ago dx with primary lung carcinoma involving adrenal
glands, liver, and bone. Had 17.6-lb weight loss during 3 months. PE shows cachexia and
significant muscle wasting. Intracellular components increased in patient's muscle cells? -
Answer- Autophagic vacuoles

48-year-old man 2-month increasing abdominal girth and inability to achieve an erection.
Smoked 1 pack cigarettes for 20 years and drank 1 pint of liquor daily. Vitals normal. PE shows
scleral icterus, spider angiomata, gynecomastia, ascites, and prominent umbilical venous
pattern. Testis small. Cause of gynecomastia? -Answer- Failure of liver to degrade estrogen
Alcoholic - cirrhosis - Liver injury

Investigator studying vancomycin-resistant strain of Enterococcus faecalis. Ten generations


created. Culture inoculated and resulting bacterial colonies are screened for vancomycin
resistance. Vancomycin-sensitive colonies observed at frequency of one per 100 cells.
Mechanism of decreased vancomycin resistance? -Answer- Plasmid loss 20-year-old woman with
palpable lump in right breast 4 months, no pain, swelling or nipple discharge. Lump is smooth,
firm, round, mobile, nontender, well delineated. No skin change. Dx? -Answer- Fibroadenoma
Researching new cancer drug, effective at killing rapidly dividing cells, in mice caused
profound myelosuppression.

In patients, most appropriate to follow which when at risk for infectious complications? -
Answer- Neutrophil counts
35-year-old woman with fever and sharp chest pain for 3 days. T 101.3F.
Friction rub heard. All causes of secondary pericarditis ruled out. Cause of primary
pericarditis? -Answer- Virus
65-year-old woman comes to the physician because of a 1-week history of a swollen, painful
left knee. Her temperature is 38°C (100.4°F), and blood pressure is 110/65 mm Hg.
Examination of the left knee shows erythema and swelling of the joint and decreased range of
motion. A photomicrograph of synovial fluid obtained by arthrocentesis is shown. This
patient's synovial fluid most likely contains which of the following? -Answer- Uric acid needle
shape right in the middle was a uric acid crystal

17-year-old boy brought by mother with concern that puberty is delayed. When mother leaves
room, patient states, "I'm fine. I don't know what's the matter with her. She wants me to be tall
like my dad." Patient is 175 cm (5 ft 9 in) tall and weighs 70 kg (155 lb); BMI 23. Sex
development is Tanner stage 4. After reassuring mom, Most appropriate next statement to
mother? -Answer- "Tell me more about your concerns about your son's height."

\19-year-old man with gastrointestinal bleeding. Laparotomy done and 5-cm blind outpouching
on antimesenteric side of terminal ileum 15 cm from ileocecal valve resected. 99mTc-
pertechnetate scan Pathology shows? -Answer- Heterotopic gastric mucosa

99mTc-pertechnetate scan = Meckel Diverticula

19-year-old woman is admitted to the hospital for antibiotic treatment of meningococcal


meningitis. She is stabilized. Three days later, her pulse is 120/min, and blood pressure is 60/30
mm Hg. Physical examination shows bilateral flank tenderness. Serum studies show a sodium
concentration of 128 mEq/L, potassium of 5.4 mEq/L, and bicarbonate of 20 mEq/L. Which of
the following is the most appropriate next step to determine the cause of this patient's
hypotension? -Answer- Adrenocorticotropic hormone stimulation test
25-year-old woman with fatigue for 3 wks and intermittent fever for 7 days, had teeth
cleaned a month ago, no abx for prophylaxis, and had rheumatic fever as child and
endocarditis 4 y ago. PE shows 2/6 murmur, ultrasound shows abnormal mitral valve. Photo
of growth from blood cultures shown (GP cocci in chains). Characteristic of causal organism?
-Answer- ]Greening reaction on blood agar alpha hemolysis (green)

83-year-old with arteriosclerosis undergoes repair of infrarenal abdominal aortic aneurysm.


Graft extends just below the renal arteries to the bifurcation of the aorta. Which organ will lose
primary blood supply and rely on collateral circulation? -AnswerDescending colon
In a 25-year-old patient who underwent splenectomy, target cells are seen on peripheral
smear. These cells are seen due to loss of function of which portion of spleen? -
Answer- Red pulp

Red pulp -Answer- the region of the spleen composed of blood-filled venous sinuses and thin
plates of splenic tissue (cords), which consist of red blood cells and various
white blood cells
Filter the blood of antigens, microorganisms, and defective or worn-out red blood cell
56-year-old exposed to possible chemical attack. Respirations labored, diaphoresis,
excessive lacrimation, increased salivation, muscle strength 2/5, urinary and fecal
incontinence. Besides atropine, another tx? -Answer- Pralidoxime
Studying epithelial repair of small intestine in experimental animal. Wants to identify most
active cell division location. Where is this cell activity found? -Answer- Base of the crypt

48-year-old recently diagnosed with lupus and difficulty working as schoolteacher because of
severe pain and swelling of joints. Current meds include prednisone, hydroxychloroquine, and
oxycodone for pain. Best action to encourage patient to adapt to illness? -Answer- Encourage
the patient to participate in a support group for persons with her condition

A 25-year-old man is admitted to the hospital because of severe crush injuries to the chest and
extensive burns over 30% of his body surface area. Three hours later, he develops tachypnea
and dyspnea. Arterial blood gas analysis on room air shows a decreased Po2 and Pco2. A chest
x-ray shows bilateral interstitial and alveolar infiltrates. The patient is intubated and
mechanically ventilated. Damage to what will preclude restoration? -Answer- Basement
membranes

12-year-old boy with mother concerned about enlarged left breast and family history of breast
cancer. Left breast is slightly larger than right, nipple mildly tender. Penis slightly
enlarged and pubic hair curling and beginning to darken at base. Best next step? -
Answer- Reassure the mother that physical findings are not uncommon for his
age
17-year-old boy with syncopal episode, fever, nausea, muscle aches, progressive confusion. T
103.5F, palpable bp 80. PE rash on lower extremities. WBC 26,000. Blood and CSF cultures grow
oxidase positive, gram negative diplococcus. Brother with similar infection at same age.
Immune disorder? -Answer- Late component of complement deficiency

75-year-old man comes to the physician because of an enlarging face shoulders and trunk and
thinning of his arms and legs. Physical examination shows a large plethoric face, fat pad over
the upper thoracic spine and purple striae on the abdomen. Serum studies show undetectable
ACTH and an increased cortisol concentration.
Administration of low dose dexamethasone would most likely result in which of the
following sets of serum findings? -Answer- ACTH no change cortisol no change

71-year old woman with coronary disease and well-controlled hypertension is brought to the
physician 2 hours after sudden onset of weakness of her left leg. Her BP 145/85 mmHg.
Neurologic exam: weakness and decreased sensation over the left lower extremity. There are
no other sensory or motor deficits. Which labeled structure is site of injury? -Answer- A
(Pre/postcentral gyrus; motor/sensory cortex)

71-year-old man admitted for prostatic resection. Normal mental status. 2 days after
surgery, confused and restless. Cannot sleep and restless, seeing little men coming through
window. Dx? -Answer- Delirium
2-month-old female with T 102F, vomiting, diarrhea, dehydration. Exam of stool shows
viral particles with wheel-like shape. Properties of virus?

Type of nucleic acid/envelop/capsid symmetry -Answer- Double-stranded RNA,


segmented/no/icosahedral
Double-stranded RNA,
segmented/no/icosahedral - "wheel-like shape"
Answer- rotavirus

62-year-old with angina pectoris is referred for cardiac catheterization. Patient worried about
hospital-associated infection. Studies show 30% of patients require admission after procedure,
and 2 percent of admitted patients acquire hospital infection. Patient's risk for infection
overall? -Answer- 6/1000

30% * 2% = .0006
Randomized controlled trial evaluating tx of acute otitis media. No statistically significant
differences found between infants receiving the antibiotic and those with standard tx.
Which aspect results in type II error? -Answer- Number of subjects in the study
.66-year-old woman is brought to the emergency department by her daughter because of a 2-
day history of fever, flank pain, pain with urination, and nausea. Ten days ago, she was
admitted to the hospital for similar symptoms and was diagnosed with acute pyelonephritis.
She was discharged with instructions to take oral ciprofloxacin after a 3day course of
intravenous ciprofloxacin resulted in improvement. She also has hypertension, hyperlipidemia,
and osteoporosis. Current medications also include alendronate, calcium carbonate,
ezetimibe, hydrochlorothiazide, and simvastatin. Her temperature is 39.1°C (102.4°F), and
blood pressure is 130/80 mm Hg. The most likely cause of this patient's current condition is an
interaction between her current oral antibiotic and which of the following medications? -
Answer- Calcium carbonate antacid - chelate tetracyclines and fluoroquinolones

35-year-old man with severe back pain, gained 8 kg in the past 6 months, decreased peripheral
vision, compression fracture spine at T10 and L1, healing left rib fracture at T6, imaging shows
pituitary macroadenoma. This tumor produces which? -Answer-
ACTH
67-year-old man with 3 months fatigue and shortness of breath. Vitals HR 90, RR 15, bp
150/98. PE conjunctival pallor. Labs: hb 8.5, hct 26%, MCV 90, RDW 14.4% (N=13-
15%), Cr 2.9, Ferritin 144, Iron 24, Transferrin saturation 23% (N=20-50%). Besides iron
supplementation, most appropriate tx? -Answer- Erythropoietin
14-year-old girl with type 1 diabetes mellitus and 4-hour history of lethargy, confusion,
disorientation. Symptoms gradually developed and she did not take her usual insulin dose
during a sleepover. HR 110, RR 24 deep and rapid, bp 95/75. Labs: glucose 450, arterial pH 7.15.
ABG? -Answer- pCO decreased
HCO3- decreased
Anion gap increase

DKA

38-year-old man 1-week shortness of breath with exertion, light palpation of carotid artery
shows upstroke is abnormally brisk and downstroke falls precipitously. Cause of finding? -
Answer- Aortic Regurgitation
water hammer pulse = brisk upstroke and downstroke in the carotid
65-year-old man with 4-hour history of intermittent severe pain in flank area radiating to
genital region. History of hypercalciuria and renal calculi. Drug decrease the urinary
excretion of calcium? -Answer- Hydrochlorothiazide
32-year-old woman has new neuro finding while being tx for acute infection of sphenoid sinus.
Imaging shows cavernous sinus thrombosis on left. Additional finding most likely?
-Answer- Inability to abduct the eye
4-year-old male with recurrent UTIs, left kidney found small and non functional; right is normal.
Nephrectomy is done, and the picture shown (dilated ureter and renal calyx). Microscopic exam
of kidney will show which? -Answer- Tubular atrophy tubulointerstitial fibrosis of the kidney

Girl has chronic cough with thick sputum, abdominal cramps in RLQ, and frequent resp
infections. Clubbing of fingers, hyperresonance on chest percussion. Diffuse crackles
and scattered wheezes. On xray - diffuse hyperinflation of the right upper lobe. She ha
a healthy brother. What's the likelihood he is a carrier for the condition? -Answer- 2 of 3
s

15-year-old girl comes with lack of improvement of facial acne. Tried topical clindamycin and
benzoyl peroxide PE severe acne vulgaris. Says, "Please help me." She has never been sexually
active. Tx? -Answer- Isotretinoin
40 year old brought to ER after wife found him unresponsibve- PCO2 60, PO2 50 HCO3
12 - acid base status? -Answer- Metabolic acidosis and respiratory acidosis
Check pH - NONE

Check Bicarb - low - metabolic acidosis

Check CO2- high - respiratory acidosis (hypoventilating)

35 year old woman - undergoes flexible nasopharyngoscopy - Purulent discharge from the right
sphenoethmodial recess. Location of structure? -Answer- superior to superior
concha

18 year old on tennis team -after 5 hours or practicing - tenderness over lateral elbow
pain -inflammation at the origin of what muscle? -Answer- Extensor Carpi Radialis
Brevis

47 y/o woman patient with psoriasis - comes to physician for follow-up- previously given several
topical creams used in specific sequence - no improvement of symptoms - what
is appropriate way to beginning discussion of compliance? -Answer- Most people find it
difficult to stick to routine. How did you do?
44 year old - severe pain and swelling in knuckles and knees - increased erythrocyte count and
Rheumatoid Factor - Treatment with Ibuprofen for 2 week. Plasma after 2 weeks (Leukotriene,
Arachidonic Acid, PGE2, PGH2)? -Answer- Leukotriene Increased
Arachidonic acid Increased
PGE2 decreased
PGH2 decreased
26 year old man - HIV+ , 3 days of reddening skin involving the palms, arms, legs, and
soles. RPR test is positive. Treatment to eradicate the causal organism? -Answer-
Penicillin

6 year old - firm, smooth, umbilicated papules 2 to 4 mm in diameter- causal


organisms? -Answer- Poxvirus

28-year-old man -struck in right eye by a baseball. He has diplopia, periorbital swelling,
and enophthalmos, Upwards gaze impaired- fracture involving the orbital-floor-
Entrapment of which muscles? -Answer- inferior rectus and inferior oblique
fracture involving the orbital-floor

Acetaminophen and nonsteroidal anti-inflammatory agents - reduce fever by decreasing


the synthesis of which of the following factors in the hypothalamus? -Answer-
Prostaglandin E2
75-year-old - sudden onset of partial loss of vision - Visual fields show R- lower
Quadrantopia - pie on the ground - a lesion in which of the following? -Answer- Parietal Lobe -
left
Pie on the ground - Contralertal lesion in parietal lobe - baums loop -bombs on ground
53-year-old - GERD - dietary changes and stress reduction have no effect. What drug
both relieves symptoms and promotes healing? -Answer- Omeprazole
Irreversibly inhibit H+/K+ ATPASE stomach parietal p. 399-2
22yo woman, g1p1, 2-day hx of fever, severe vaginal bleeding. four days ago delivered healthy
male newborn. Temp 38.1 C (100.6 F). Pelvic exam: open cervix, heavy vaginal bleeding. US
shows uterus with no placental tissue or thrombi. If operation required to control bleeding,
ligation of branch of which artery required? -Answer- internal iliac

42yo F with 3-year hx of an intermittent facial rash, including the forehead, eyelids, nose, and
cheeks. Rash seems to be getting worse since she moved from New York to Florida last year.
Spicy foods precipitate a flushing reaction that seems to exacerbate the rash. PE shows
erythema over the nose and cheeks, with scattered telangiectasias and a few papules. Dx? -
Answer- Rosacea
17 year old - evaluated for growth delay - X-ray shows suprasellar calcifications- most
likely explanation for this condition? -Answer- Craniopharyngioma
Most common childhood suprasellar tumors

Remnants of Rathke Pouch - derived from oral/surface ectoderm of oral cavity


64 y/o man - bronchospastic pulmonary disease - wheezing in spite increasing on b- adrenergic
agonist inhaler- hospitalized and systemic prednisone is added - which explains the action of
prednisone in enhancing response on B-adrenergic agonist?? -
Answer- Enhanced action of the agonist at B-adrenergic receptors
Corticosteroids- upregulates B2 receptors + inhibit b2
Cortisol - permissive effect on catecholamines - 336-1
2-week-old male with Patent Ductus Arteriosus - which finding is likely? -Answer- Higher
than normal Left ventricular cardiac output
33-year-old woman - MVC - splenic laceration - 3 units of RBC administered - 5 hours later -
develops Shortness of Breath - crackles heard through lung fields - Chest X-ray bilateral
infiltrates - what causes patient's condition? -Answer- Transfusion-related acute lung injury

8-year-old girl is brought to the physician for a well-child examination. Her mother says that she
has been well except for an occasional cold. Her immunizations are up to date. She is at the
50th percentile for height and 60th percentile for weight. Physical examination shows breast
bud development and a few pubic hairs. The mother asks whether her daughter's development
is normal. Which of the following is the most appropriate initial response by the physician? -
Answer- Your daughter development is normal
Precocious puberty before age 8 in girlsPrecocious puberty before age 9 in boys

A 63 y/o - bladder cancer - starts treatment - significant decreased in tumor size.


Paclitaxel mechanism of action? -Answer- Tubulin Polymerization
Stabilization M Phase
25 year-old - participating in a study of blood glucose - During study, found to glucose of
100mg/dl after 20 hours of fasting- which would contribute to normal glycemia? -
Answer- Alanine

Muscle breakdown - alanine - enter cahill cycle to form glucose


3 y/o boy - recently immigrated to USA - skeletal deformities- frontal bossing, flattening of back
of the skull, defects in enamel of the teeth, chest wall grooves- legs are bowed Serum calcium
and phosphorus low - if bone were examined, what finding likely? -
Answer- increased portion of osteoid rickets
due to osteoid deposition
Vitamin D Decreased, Calcium Decreased, increased PTH

17-year-old boy - acute lethargy, sore throat, fever - positive Epstein Barr Viral Capsid
antigen - what increased in peripheral blood? -Answer- Lymphocytes a. P 164 - atypical
lymphocytes on peripheral blood smear
50 year old with RA - decreased iron, decreased TIBC, increased Ferritin, MCV 77-
What DX? -Answer- Anemia of Chronic Disease

38 year old - epigastric pain and frequent stools - 2 episodes of renal calculi -endoscopy
shows non bleeding duodenal ulcers - PPI therapy is begun - next step in
management? -Answer- serum calcium concentrations

PPI - reduce Mg+ and Ca-+ need to check - leading to fracture risk - p. 399-2

HIV Aids antiviral agent inhibits entry of the HIV virus into CD4+ T cells- which of the
following antiretroviral agents? -Answer- Enfuvirtide
Binds gp41 - inhibiting viral entry p. 201
MO Entry - maraviroc and enfuviritide

Scientists - identified the flawed gene causes 1-6 colon cancer - when gene is working -
gene repairs discriminates between the template and the newly synthesized DNA
strand, correcting mistakes in the newly synthesized gene? -Answer- Mismatch repair
Colon Cancer - Lynch HNPCC

55 y/o- with renal artery stenosis - 50% reduction in renal blood flow - which part of
kidney has the highest concentration of renin in response to reduction in arcuate
pressure -Answer- Cortex

Renin is produced in JG Cells - located in cortex

P. 589 - located in modified smooth muscle of afferent

14 year old - 2 month history of hair loss - PT sad since grandmother died unexpectedly
3 months ago. Underlining skin is normal. Explanation for hair loss -Answer-
Trichotillmania

Compulsively pulling out one hair - causes significant distress despite attempts to stop -
34 year old - 6 week history pf fatigue and shortness of breath - fixed split S2 -
midsystolic pulmonary ejection murmur - A congenital heart defect - patient undergoes
surgical repair - greatest risk for intraoperative injury? -Answer- Atrioventricular Bundl
p. 563
e
66 year old - pain, fever, chills, - left lower abdominal pain - severe constipation over 5
years -a barium of lower GI - 3 separate area of narrowing of the lumen of the distal
sigmoid - Photograph of lesion - DX? -Answer- Diverticulitis
LLQ pain, fever, leukocytosis, wall thickening p. 383-3
> 60 years old -
64 year old women - T2DM and HTN - soreness and muscle Pain in large muscles - She began
rosubastatin therapy - her medicatioons include gemfibrozil, HCTZ, losrtan, and metformin,
elevated serum creatine kinase - what interaction between rosuvastatin and which of the
following? -Answer- Gemfibrozil
Statins - 320- cause
Rhabdomyolysis Myopathy when
used with fibrates
48-year-old man - ER - 1-hour history of vomiting blood - Physical exam shows ascites.
A surgical shunt between which of the following veins - release esophageal bleeding? -
Answer- Splenic and left renal vein

37-year-old - MVC - severe, sharp, chest pain - hyperresonance- decreased tactile fremitus
- adventitious sounds are absent- multiple rib fractures - pathway of chest
pain? -Answer- Intercostal nerve

Multiple rib fractures and pneumothorax - COPD

costal and cervical pleura innervated by intercostal nerve


78 yeasr old women - paroxysomal embolus in left leg - left lower extremity cool and pale,
absence pulses- ECG shows irregularly irregular rhythm - which treatment is most appropriate?
-Answer- Embolectomy
Originated in heart - sudden acute of pain in the leg
Paroxysmal embolus

75 year old man - fever, back pain, pain on urination - long standing prostatitis- gram statin -
gram positive cocci in chains - cause of illness? -Answer- Enterococcus Faecalis Gram + cocci in
chains + UTI = enterococcus

65-year-old - vomiting blood - Hypovolemic shock- placement of catheter in internal jugular


vein for blood transfusions - improper insertion damage to which of sets of underlying
structures? -Answer- Lung and Common Carotid Artery
b. Internal jugular vein - next to common carotid - and lung apex extend above 1st rib
32-year-old woman at 35 weeks' gestation - during her labor - increased oxytocin
concentrations - most likely cause of finding? -Answer- Birth Canal Reflex
Fergusson Reflex - positive feedback loop

75 year old woman - sudden Horner syndrome, dysphagia, difficulty speaking, which
occluded? -Answer- Posterior Inferior Cerebellar

Lateral medullary syndrome - nucleus ambigious (IX, X, XI)

65-year-old man in MVC - photograph of lung at autopsy (Centrally located lesion) - Dx?
-Answer- Squamous Cell Carcinoma

Centrally located- arising or extending into bronchus


Hilar mass arising from bronchus - p. 684
Hypercalcemia - PTHrp- source from cancer

54yo man for exam. P80, R14, BP140/95. Physical exam normal. ACE inhibitor given.
Where is MOA in kidney? -Answer- F - ACE preferentially vasoldiate the efferent
arteriole

Preserves GFR - by back pressure


vasodilation of efferent

68 year old man - 1 month history of not being able to sustain an erection - wife died 2
years ago - able to masturbate - testosterone within reference range -no symptoms of
depression- which pairs of finding in history? -Answer- Libido Normal
Nocturnal Erection Normal

A 30 year with hypothyroidism - receiving T4 - fetus at greatest risk if TSH


concentrations increase? -Answer- Abnormal Brain Development
55-year-old - 6 month history of bone pain - 20 year history of T2DM - receiving hemodialysis -
can't recall meds - Increased phosphate, increased ALP, increased PTH, decreased calcium
bone pain is caused by which of the following in serum concentrations? -Answer-
Increased parathyroid hormone

4 -month-old - brought in because she has seizures - diarrhea for 3 days - consumed only
water for 24 hours because parents ran out of formula - what is likely electrolyte
abnormalities -Answer- Hyponatremia
17-year-old boy returns to locker room after football practice - soaked in sweat- characteristic
odor associated with seat - result of secretion of which glands? -AnswerApocrine
Apocrine - develop in areas of hair follicles
Your armpit smell like an APE- body odor
34 year old women - postpartum depression - I feel guilty nit enjoying my baby more physician
most appropriate step? -Answer- I'm Concerned about bad you are feeling lately. Have you had
any thoughts about death or wanting to be dead?
4 year old boy with Chronic Granulomatous Disease - TX with interferon gammas.
Which is increased after exposure to the drug? -Answer- macrophages producing
interleukin-1 (Il-1)
4-year-old-girl conscious but unable to breathe under general anesthesia with sevoflurane -
succinylcholine administered for intubation - cause prolonged apnea -
Answer- Pseudocholinesterase Deficiency
3 year old boy brought to ER because of nosebleed - can't stop bleeding - PT and PTT are
prolonged - platelet count and thrombin time are within reference ranges - which of
coagulation process if affected? -Answer- Synthesis of Factor X (stuart Factor)

27 year old man - no history of medical illness - never sexually active - minimal contact
with his parents and siblings - not depressed - no hobbies - flat affect - personality
disorder? -Answer- Schizoid
80yo F in ED for 2-day hx of "feeling funny." "Lost my pep." Hx of poorly controlled
hypertension. Just started medication 2 weeks ago- doesn't know the name. BP 130/85.
Pe normal. Serum potassium is 3. Which drug? -Answer- Hydrochlorothiazide
Diuretic TX- HTN, heart failure pg. 609-1
Potassium is low
Hypokalemia metabolic acidosis, hyponatremia, nephrogenic diabetes insipidus
61 year old woman - 9 month history increasing forgetfulness - Mini Mental Status
22/30- most appropriate pharmacotherapy? -Answer- Donepezil
anticholinesterase

Dona, Riva, at gala - increased ACH


51-year-old woman - burning abdominal pain, sweats profusely, 2 cm- mass in proximal
duodenum. Gastrin releasing tumor stimulates which labeled cells in picture? -Answer- B-
parietal Cells
Stain Eosinophilic - fried egg appearance- PINK EGGS
Parietal cells - secrete gastric acid
25-year old develops shortness of breath - after moving from sea level to a mining town
at 3350. At rest, pulmonary hypertension with normal cardiac output - pulmonary
function tests are normal - Which explains pulmonary hypertension? -Answer-
Decreased Alveolar PO2
30 year -old comes to the office - 20lb weight loss after initiating a low-carbohydrate diet
- low fat diet -it is most important to counsel the patient to include which of the
following to maintain a good nutrition? -Answer- Linoleic Acid
Essential fatty acids - OMEGA 6 and Omega 3
Omega 6= linoleic and Omega 3 = alpha linoleic
Must be obtained through diet - vegetables, oils, meats, nuts, and egg
Deficiency - lead to poor growth, skin lesions, reproductive failure

44-year-old man -altered mental status HTN (210/145)- fundoscopic exam shows papilledema
and retinal hemorrhages - A drug with which of the following effects on vascular smooth
muscle is likely to decrease blood pressure? -Answer- Increasing
cGMP
A test has been developed to screen for HPV infections - If the prevalence increases by 50%-
which of the following features of the test will increase? -Answer- Predictive value
of a positive test
. A newborn female external genitalia - 46 XY karotype - shows male genital ducts - the most
likely cause of this anomaly is a mutations resulting in the absence expression of which of the
following hormones or factors? -Answer- Dihydrotestosterone
36-year-old with AIDS - tx with CCR5 receptor inhibitors - which is natural ligand for HIV
co-receptor? -Answer- Chemokine
Maraviroc binds to CCR5

Cysteine-Cysteine Chemokine receptors - found on surface of CD4+


7 year old - difficulty concentrating in school - "feels tired" - hypochromic anemia fatigue
- increased serum creatinine - additional abnormal concentrations in this patient? -
Answer- Increased Blood Lead

Physician prescribes a recently marked drug for 20 patients - After several months, 5 patients
develop increased serum AST and ALT - Physician discontinues drug and reports adverse effects
to the FDA. Physician participated in what phase of clinical trial? -Answer- Phase 4
SWIM
Post marketing surveillance - detects rare or long term adverse effexts

6-month-old - exaggerated reaction to loud noises- Exam shows muscle weakness, increased
start reflex, cherry red spots on retina- most likely cause of this disorder is accumulation of
which of the following lipids in the patients lysosomes? -Answer- GM2
Tay-Sachs - cherry red spot
Gm2 gangliosides accumulate
Deficiency in Hexoaminidase A

60 year old - ER -thrown from care in high speed automobile accident and struck guardrail-
blood in urethral meatus and fractures of the pelvis and left femur. A Foley catheter is unable
to be inserted- which is most likely explanation? -Answer- Urethral Disruption
Suspect urethral injury if blood is seen at the urethral meatus
30 year old - pruritic rash on his lower back - underwent liver transplantation 1year ago
and maintained on immunosuppressive drugs -PE shows vesicles containing clear fluid
associated with ulcerated and crusted lesions- likely diagnosis? -Answer- Varicella-
zoster virus infection
50-year-old man progressive dyspnea on exertion. Dyspnea at rest. Exam of lung tissue biopsy
shows chronic inflammation and fibrous thickening of alveolar septa. Dx? - Answer- Usual
Interstitial Pneumonitis

52-year-old woman admitted to hospital because of breath die to malignant ascites and pleural
effuision- two gynecologists disagree about treatment -- pt comes to primary care pysician to
help address conflicty - what do internal medicine physicians do to address concern? -Answer-
Speak with two gynecologists to attempt to coordinate

36-year-old woman undergoes a total hysterectomy and bilateral salpingo- oophorectomy for
extensive endometriosis of the pelvis. After operation, she refuses hormone therapy. Which of
the following - most likely occur in gonadotrophs if this patient refuses hormone therapy -
Answer- Hyperplasia
Gonadotrophs are the FSH/LH producing pituitary cells. No ovaries, no hormones, no feedback
inhibition.
An investigator - study efficacy of capsular polysaccharides - Neisseria Meningitidis -
Which of the following compounds is most likely to induce increased titers of these
antibodies when conjugated to polysaccharides? -Answer- Flagellin

35-year-old woman with blistering lesions on sun-exposed area of face, arms, hands -
increased AST, ALT, Total Porphyrin, Urine uroporphyrin III, deficiency in what
enzyme? -Answer- Uroporphyrinogen Decarboxylase

Porphyria cutanea tardia - vampire blistering lesion on sun-exposed


European Vampire - hide in cardboard
. 23-year-old drinks alcohol heavily on weekend - which cellular changes most likely occurs
in his liver? -Answer- Fatty Change Acute alcohol consumption - increases fatty change
Cellular Swelling - indicates alcoholic hepatitis Hepatic steatosis - alcoholics hepatitis -
cirrihosis

30 year old man - urinary incontinence for 2 weeks after treatment for pelvic fracture- pinned
against loading dock- distended bladder- absence of micturition- overflow urine occurs - injury
to which? -Answer- Pelvic Nerves

35-year-old - pain and swelling of his right knee and great toe - needle like crystals Exam of tiw
shows swelling and warmth. Aspirate of effected joint shows numerous neutrophils and needle
like crystals what is causes of patients swelling? -Answer- Increased vascular permeability

25 year - nulligravid and infertile woman - irregular menses -- hirsutism - ultrasound shows
ovaries with numerous cysts beneath the capsule. Photograph shown - patient at greatest risk
for developing which of the following? -Answer- Endometrial Hyperplasia Chronically elevated
levels of estrogen can cause endometrial hyperplasia.

. 10 year old - fever, malaise, loss pf appeptide followed be excess salivation 6 weeks after
exploring a cave with her friends. Over the next few weeks, she develops delirium, seizures,
paralysis, and hyperexcitability- virus reached brain by which of the following?
-Answer- Retrograde transport through the nerves
21-year-old - comes to ED after MVC - laceration of spleen and undergoes splenectomy
-Splenectomy most likely predisposes patient to develop future infections - causal
organism? -Answer- Streptococcus Pneumonia
No Spleen Here

Neisseria meningitidis, Streptococcus Pneumoniae, Haemophilus influenza

62 year old - farmer - 8 yeas history of lesions indicated by arrow (photo) - picks lesions
causes bleeding - numerous similar rough plaques on the scalp, face, ears, and
forearms. Likely Diagnosis? -Answer- Actinic Keratosis
Farmer - Picks at it and bleeds -

Premalignant lesions - caused by sun exposure


Increased risk of skin cancer - keratin pearls

A 40 year old - T2DM - TX with pioglitazone - mechanism of action? -Answer-


Stimulation of peroxisome proliferator-activated receptor

47-year-old - severe chest pain - Oxygen Saturation of 85%, CO low, ST elevation,


which is decreased? -Answer- Mixed Venous Oxygen Tension

Decreased oxygen delivery to tissues - decreased cardiac output


CO=rate of O2 consumption/(arterial O2 content-venous O2 content

53-year-old - breast cancer - taking tamoxifen - what is effect on tamoxifen on breast


and Uterus? -Answer- Breast Antagonist
Bone Agonist

Uterus partial agonist.

26-year-old - counseling prior to conception - pedigree - fatal to males not born = which
is most likely risk to live born? -Answer- 50% in females, but near 0 in males
Fatal to males in utero

X linked dominant - males have 50% of getting from mom

43 year old - follow-up exam -underwent cardiac valve replacement - Chest X-Ray
shown - which of the following valves most likely replaced in this patient? -Answer-
Aortic

30 year old - pregnant + gram positive rods - pregnant women - fever, chills, muscles
aches - which causal organism? -Answer- Listeria Monocytogenes

Normal appearing 17-year-old - normal appearing - vagina ends blinding - no uterus or


ovaries to palpate - chromatin negative nuclei - which is likely diagnosis? -Answer-
Androgen Insensitivity Syndrome
27 year-old - 34 weeks pregnant - nausea, vomiting, abdominal pain for 12 hours -HTN 164/104,
Elevated Liver Enzymes. platelet count low eve what do you see on peripheral blood smear? -
Answer- Schistocytes
52 year - ER - sudden chest pain - IV morphine - generalized pruritus and warmth in 1
minute - which substance most likely cause new symptoms? -Answer- Histamine
Pruritus and warmth in 1 minute- Hypersensitivity
59-year-old man - 3-month history of progressive difficulty swallowing - 15lb weight loss -
smoked 1 pack of cigs daily for 40 years - cachectic - mass ulcerated with elevation surrounding
mucosal rim- 6 cm mass in mid esophagus - centrally located mass - examination of the biopsy
specimen of the mass is likely to show neoplastic cells
with which of the following? -Answer- Foci of Keratinization
Squamous Cell Carcinoma occurs in upper 2/3 -
Adenovrius occurs in distal 1/3
34-year-old - periorbital headaches - last 60- 90 minutes - lacrimation and nasal
stuffiness - whichh of the following is most likely diagnosis? -Answer- Cluster Headache
7-year-old - lack of adequate quality protein identified as critical issue- which of the
following is most critical in analysis of child's diet? -Answer- Methionine

33-year-old women - fever, malaise, burning on urination, and scant vaginal discharge
extremely tender vesicles and ulcers - DX? -Answer- Genital Herpes

28-year-old - sudden onset of 30 of shortness of breath - history of cocaine abuse -


diminished pulses in left upper extremity - widened aortic arch - dx? -Answer-
Dissecting Aneurysm

24 year old - sickle cell disease - generalized fatigue -MCV 105- Hematocrit dropped.
Which diagnostic test most likely explain the patients symptoms and decreased
hematocrit? -Answer- Serum Parvovirus B19 IgM antibody test
66-year-old - 1 day history of fever - underwent transurethral resection for prostate 1 week ago
- BP 70/50 Blood cultures grow, lactose-positive, gram negative rods. Which describes the initial
event of hypotension in the patient? -Answer- Lipopolysaccharide stimulation of toll like
receptors
A photograph shows spinal cord injury after MVC - image shows damage to fasciculus
gracilis? -Answer- Loss of sensation to vibration of both feet

47-year-old - aplastic anemia - TX for disseminated aspergillosis with antifungal drug


binds to specific sterols in plasma membrane - which drug? -Answer- Amphotericin B
34-year-old - ER - unconscious after alcohol intoxication - which is interpretation of arterial
blood gas. Ph 7.25 Pco2 23 Po2 78 - Na+ 140, Cl - 105, HCO3 10? -Answer-
Primary Metabolic Acidosis with increased anion gap
Ph low, Bicard low, CO2 low
Na+ - (HCO3+ + Cl-) 140 - (10 + 105) = 25
18-year-old - 2-week history of fatigue - PE shows scleral icterus - Lab shows - Hemogloblin
Low- high bilirubin - increased 2,3 BPG, increased reticulocytes - mostly likely deficiency in
enzyme in erythrocytes? -Answer- Pyruvate Kinase
22 year old - frequent episodes of cutaneous urticaria and laryngospsm - allergic cause
of his urticaria. Which of the following complement proteins is most likely deficient? -
Answer- C1 esterase inhibitor (binds activated C1r, C1s)

45 year - ER - MVC - patients' ventilator - adjusted to tidal volume- application of PEEP


will most likely prevent which of the following complications? -Answer- Absorption
Atelectasis
35-year-old -DKA - treated with insulin - decreased plasma glucose 900mg/dl to 270. which
additional lab finding will likely decreases with insulin therapy? -Answer- Serum potassium
concentrations

67 year sore - fever, sore throat, difficulty swallowing- Corynebacterium diphtheria - the
pathogen produced diphtheria toxin - blocks which of the following processes? -
AnswerTranslation
40-year-old woman - high triglycerides - 380 mg/dl - taking fish oil and a supplement to
treat dislipidemia - MOA of most appropriate vitamin? -Answer- Antagonize VLDL-
cholesterol Secretion

Niacin (vitamin B3) antagonizes VLDL


55-year-old woman - broad bait gait, nystagmus, confusion - confabulations - BMI 21 what is
most likely to see on MRI of the brain shows? -Answer- Mammillary Bodies

66-year-old - pancreatic cancer - comes to office because of 2 day history of black tarry stools -
Upper endscopy shows gastric varices - A CT shows thrombosis of branch of splenic vein - which
is most likely source of bleeding in the patient? -Answer- Short Gastric

29 year old woman - 10 day history of difficulty walking - broad based gait and spasticity -
lesion in white matter of the cerebellum- TX with which of the following agents most likely
improve spasticity? -Answer- Baclofen Gaba B agonist

30 year old - dimness of vision and loss of color vison - 1 month after treatment for TB with
RIPE - Which drug responsible for adverse effect treatments? -Answer- Ethambutol

75-year-old - stroke - 16 hours after onset of left arm and leg weakness - CT scan shows an
edematous right hemisphere - No evidence of hemorrhage- but sparing of the right occipital
and midline frontal corticies - one hour later dies At autopsy - microscopic examination of right
lateral lobe is most likely to show which? -Answer- Shrunken
Eosinophilic Red Neurons within
12-24hours
43 year old woman - irregular jerking movements - profound dementia - her father had similar
disorder and died at age 65 year - huntingtin disease- autopsy of the patient is most likely to
show severe atrophy of which of the labeled structure? -Answer- C = Caudate Nucleus

30-year-old man - intermittent severe lower abdominal pain, right flank tenderness, hematuria,
brother has kidney stones. Patient urine turns cherry red with nitroprusside. Hexagonal crystals
are present in the urinary sediment- Which of substances is most likely increased in patient
urine? -Answer- Cysteine
65 year - prostate cancer - metastasis to L1 underwent orchiectomy - high PSA - Which
is most likely increased secondary to orchiectomy? -Answer- DNA fragmentation
23-year-old develops persistent sneezing each year - sneezing worsens when in garden
- A drug with which of the following mechanisms is most likely treating patients? Answer-
Stabilization of Mast Cells Membranes
55 year old - with a benign nodule in left lobe of thyroid - undergoing a partial thyroidectomy -
A branch of thyrocervical trunk is ligated to interrupt the blood supply of the resected
specimen. It is most appropriate to ligate a direct branch of which of the following arteries-
which of the following arteries? -Answer- External Carotid
Superior thyroid comes off external carotid
Inferior thyroid comes of thyrocervical trunk
. 75-year-old woman - poor concentration and decreased energy. For 6 months, she is
waking up at 4am and been unable to get to sleep. Physical examinations shows no
abnormalities. What is most likely sleep disturbance? -Answer- Major Depressive
Disorder

Insomnia - unable to get back to sleep = depression

39 year old woman with Rheumatoid Arthritis - develops proteinuria following treatment
with penicillamine. Renal biopsy shows IgG and C3 in the glomerular basement
membrane. Dx? -Answer- Membranous Glomerulonephritis
28 year old man - evaluation of infertility - normal physical exam - normal serum concentration
of testosterone, FSH, LH, and prolactin - what is abnormal? -AnswerFructose

A screening program of Chlamydia - C trachomatis - initial screening is 500/2500 - one year later
- an additional 200 - which is annual incidence of C trachomatis? -Answer-
10%
=200 (new cases)/2000 #at risk
# at risk = 2500-500

. 5 year old homocystunuria - improves with high does oral pyridoxine therapy - cystathionine
synthase is deficient but increases to normal when incubated with higher concentrations of
pyridoxal phosphate. When cystathionine synthase is measured in the presence of pyridoxal
phosphate - which lineweaver -burk plot would be most consitent with the enzyme? -Answer-
Area labeled B Competitive inhibitor
28 year old woman - 18 weeks' gestation has palpations - increased serum thyroxine (T4)
concentration in pregnancy - Which test used to diagnose hyperthyroidism in the patient? -
Answer- Free T4

28 year - acute leukemia - receiving high dose chemotherapy - Pancytopenia- leukoocyte count
low, MCV low, Platelet low - antibiotic therapy is started. Which of the following is most
appropriate? -Answer- Granulocyte Colony-Stimulating Factor

50 year old man is found dead at home - he has chest pain 3 weeks ago prior to death
. At autopsy - severe atherosclerosis of 3 major coronary vessels - myocardial softening
and mottling involving the anterolateral wall - Photomicrograph of a section is shown.
which is most likely mechanism of death -Answer- Arrhythmia
-

Ventricular Arrythmia - most common after 24 hours of MI

68 year old - fever, shortness of breath - Xray shows right lower lobe infiltrates - leukocytes
38,000 - peripheral blood smear is shown- which of the following best describes blood smear? -
Answer- Reactive Granulocytosis

19 year old women -1 month of abdominal pain and severe diarrhea - 5.4 KG weight loss
colonoscopy shows non-bloody diarrhea and cobblestone appearance of the colonic mucosa -
stricture in retroperitoneal portion of the bowel - which is the most likely location of the
stricture -Answer- Descending Colon

Which of the following best explain why use of anti-idiotypic antibody is inappropriate
therapeutic reagent for treatment? -Answer- Myeloma Cells do not have membrane surface
immunoglobulin
a. Plasma cells produce secretory Ab's (aka produce Ab's) -- plasma cells do not have
membrane-bound Ig (aka do not have BCR').
9 year-old - tumor in heart - what is the diagnosis? -Answer-
Rhabdomyoma
Tumor heart in child =
rhabdomyoma
32-year-old woman - sister recently died of melanoma - Other family members
similarly affected. Which lesion is most likely on both sun exposed and nonsun
exposed ares on
patient skin? -Answer- Dysplastic
Nevi
Diastolic blood pressure obtained from two groups of 100 asymptomatic -
picture of
Diastolic average? -Answer- Median Group Y =
Higher
Mode Group Y = Higher

72 year old man - history of nonproductive cough -non hodgkin lymphoma- treated
with antineoplastic drugs - chest xray shows diffuse infiltrates- which of the drugs
most likely
cause of patient symptoms? -Answer-
Bleomycin
47 y/o - jaundice - increased serum bilirubin and positive urine bilirubin test -
Which is most likely associated with the positive urine bilirubin test -Answer-
Obstruction of the
Bile Duct
70-year-old - persistent fever despite intravenous broad-spectrum antibiotic therapy 3 days
after operative incision for cervical carcinoma. PE shows placement of central venous catheter -
Blood cultures from cather grow - Gram stain colonies, 4um, elliptical, purple budding
organisms- which is the causal organisms? -Answer- Candida Albicans

A 74-year-old with mild dementia is admitted to the hospital because of congestive heart
failure. Physician recommends coronary angiopgraphy and angioplasty. The patient states that
she wants to go home. It would be appropriate to question patient capacity if? -Answer-
Inability to understand the severity and prognosis of her medical condition informed consent
process
A 74-year-old with mild dementia is admitted to the hospital because of congestive heart
failure. Physician recommends coronary angiopgraphy and angioplasty. "On questioning, the
patient does not know the date [time], the name of the hospital [place], or the name of her
nurse who had just introduced himself [person]." The patient states that she wants to go home.
It would be appropriate to question patient capacity if? - Answer- inability to understand the
severity and prognosis of her medical condition

The gene that codes for a protein normally found in the ER is mutated. Which of the following
mutations is most likely to impair the transport of the protein in the endoplasmic reticulum? -
Answer- Deletion of the hydrophobic amino acid sequence on N Terminus

25-year-old -increased SOB - exertion 2 weeks after birth of first child - JVD - evidence of edema
at lung base, hepatomegaly, and mild pitting edema in lower extremities - which is the
following cardiac abnormalities is abnormality is most likely present? - Answer- Four Chamber
Dilation
Peripartum
Right heart failure - caused by left heart failure

27-year-old - 39 weeks gestation is admitted to the hospital in labor - 5-year history of T2DM-
currently treated with insulin - Difficulty controlling serum glucose - the cervix is 100% effaced
and 7 cm dilated - babies weight is 9lb - what is the following obstetric complications -
Answer- Shoulder Dystocia
38 year old woman -swollwen painful calf for 2 days- gave birth 5 days ago - one day
later she has a cerebral infarction with hemiplegia. Passage of embolus into systemic
circulation is most likely occur in which of the following locations? -Answer- Patent
Foramen Ovale
35-year-old - 2 day history of fatigue and dizziness - He has profuse watery diarrhea for 8 hours
despite lack of oral intake. He returned from remote village in Honduras. PE shows dry skin,
decreased capillary refill - A gram stain = gram negative, comma shaped bacteria - which
described the mechanism of the toxin? -Answer- Activation of Adenyl Cyclase enterotoxin that
permanently activates Gs, cAMP

40 y/o man with interstitial pulmonary fibrosis - had a greater expiratory flow rate than
predicted- which explains this finding? -Answer- Increased radial traction on airways Fibrosis -
increased traction

Clinical trial of 800 - 400 in surgery and 400 in medical management - 75 drop out of surgery
procedure, 50 undergo operation during the study - Intention to treat analysis which accurately
represents how many patients would be analyzed in surgical procedures and medical
management? -Answer- Surgical Procedure group = 400 Medical Management Group = 400
Intention to treat = Include all patients in the groups they were originally in

41-year-old numbness of his right hand - He works as a construction. Sensation to pinprick is


decreased in the thumb and over a portion of the anterior forearm. The physician suspects
compression of C5 and C6 spinals nerves as they exit the spinal canal - weakness of which of the
following movements on the right is most likely to confirm diagnosis? -Answer- Abduction of
the upper extremity

. Left radial arterial and venous blood samples are drawn from 10 chimps while resting quietly.
Results indicate that the total carbon dioxide concentration average in the arterial samples and
24.3 mM in the venous samples. Which accounts for large carbons dioxide in the venous
sample? -Answer- HCO3- transported into plasma
a. CO2 is primarily transported as HCO3 - in plasma - then converted back to CO2 lung

65 year old with - undergoes surgical repair for right internal carotid artery in the cavernous
sinus. 3 days later right pupil is larger than left pupil. Weakness in movement of the eye. CN III
injury - location in the cavernous sinus (photo)? -Answer- area labeled A - cranial nerve 3

83-year-old man - 3 day history of painful blisters on his torso - PE shows numerous .5 to 1cm -
clear fluid filled tense blisters- subepidermal blisters- Production of autoantibodies directed
against which of the following? -Answer- Bullous Pemphigoid
32 year old woman - ER - fever, intense headaches, excruciating pain in joints and
muscles of the arm, took a trip to Caribbean and Cancun the Yucatan region -
Temperature in 104. physical examination shows a mild macular rash on the trunk-
which of the following is the route of transmission of causal organism? -Answer-
Arthropod Vector

. 1 y/o girl- multiples fractures since birth - gray translucent, pointed teeth- Xray of long
bones shows osteopenia- substitution of alanine for glycine residue in type 1 collagen -
which occurred in the patient as a result of this mutation? -Answer- Disruption of the
secondary structure of collagen molecules
Bones Fractures
I (eyes)- blue scelera
Teeth Imperfections
E (ears) Hearing Loss
55 year old - African American - asked by public relations to be filmed - patient consents
to filming and broadcasting - after filming, patient states he doesn't want it shared? -
Answer- Tell the patient that he can change his mind about participating

55-year-old man with HTN comes in with severe headaches and confusion- 190/110 -
faint bruit is heard over the left abdomen - Smoker for 30 years - captopril renal

radionucleotide scans delayed function in kidney - Which of the following is most likely
on abdominal aortography? -Answer- Left Renal Artery Atherosclerosis

55-year-old man with HTN comes in with severe headaches and confusion- 190/110 -
faint bruit is heard over the left abdomen - Smoker for 30 years - captopril renal
radionucleotide scans delayed function in kidney - Which of the following is most likely
on abdominal aortography -Answer- Left Renal Artery Atherosclerosis
48-year-old man - increasing bronzing of skin, hepatomegaly, fatigue, and small testes. Lab
findinds AST, ALT, IRON, Transferrin Increased - Testosterone, LH, FSH decreased which explains
findings? -Answer- increased intestinal iron absorption
34-year-old man- evaluated for light headedness after 12 miles of marathon on hot day -
blood pressure is 80/60 - what changes in autonomic nervous system? -Answer- #
Impulses Carotid Baroreceptor - decreased
sympathetic efferent - INCREASED
parasympathetic - decreased
Baroreceptor - low - low blood pressure - increased sympathetic response

70 y/o old man - fever, night sweats, productive cough, X-ray xhows cavitary lesion in the right
lung- which of the cytokines play the dominant role in inflammatory response to patients lung?
-Answer- interferon-y Th1 - secrete IFN-y - macrophages

63-year-old woman - 5 day history of SOBs and swollen legs - her respirations are labored, and
blood pressure is 130/50. She had large subclavian AV fistula caused by a stab wound to the left
supraclavicular area 15 years ago. PE shows 2+ edema of the lower extremities - likely finding? -
Answer- increased resting cardiac output
A female newborn - 26 weeks gestation found to have hyaline membrane disease -
inspired oxygen maintain 92% and 95% - primary goal of the treatment to protect
following structures? -Answer- Retina
A study is conducted with asthma to compare a new asthma treatment with placebo;Sample
size is 100 subjects and 80%power detecting a mean difference of 0.4 in at a significance level
of 5%. When planning a study, what is the meaning of 80% power -Answer- If the treatment
really changes the mean asthma score 0.4, there is an 80% chance that a study of this size will
find a p-value <.05

Power = When a difference exists and the null hypothesis is rejected is power.
Rx detects a mean difference of 0.4 in asthma in the patients in the treatment group,
then that data falls in the 80% power range, and its significance is 95% (p<0.05
5 year old boy brought to physician for a well-child exam. He recently emigrated from Nigeria -
Skin red-bronze, red hair, continual horizontal nystagmus - Genetic testing shows compound
heterozygoisty of TYRP1 gene - frame-shift and nonsense mutation are detected. These genetic
changes caused the following? -Answer- Lack of Choroidal pigment in macula

A study evaluated relationship between TV exposure and learning disability - Two groups - one
with learning disability and one with the absence of learning disability
(case control) - investigate using? -Answer- Odds/Ratio used to
evaluate cAse cOntrol studies A/O = odds ratio

A study evaluated relationship between TV exposure and learning disability - Two groups - one
with learning disability and one with the absence of learning disability
(case control) - investigate using? -Answer- Odds/Ratio sed to
evaluate cAse cOntrol studies A/O = odds ratio
8 y/o - with ADHD - a drug with the following action is most likely to be useful for his
patient? -Answer- Stimulation of the release of biogenic amine neurotransmitter

66-year-old right-handed woman develops acute onset of spastic left hemiparesis and
weakness of the lower two thirds of the face on the left- the most likely cause of the
condition is a lesion at the following labeled area? -Answer- Area Labeled B
internal capsule on Right

Stroke = opposite - can wrinkle forehead

Bells Palsy= Same Side - can't wrinkle forehead


76 year old 2 week history of constant, intense pain of his right arm - 7 months ago he had
cerebral infarction that resulted in lack of sensation of right side of body - Muscle strength is
normal - Decreased sensation to light touch, vibration, pain, temperature on right side of the
body. What is cause of the patients pain is damage to -Answer- Left Thalamus
Neuropathic Pain after stroke is central Post stroke pain Syndrome caused by
contralateral thalamic lesions
45 year old comes to physician - progressively worsening, constant pain in left thigh over 3
month - X ray shows thickening of the diaphysis and disruption of the cortex - focal areas of
increased calcification? Photomicrograph - Which is the DX -Answer- Chondrosarcoma
Hyaline Cartilage
alignancy - pleomorphic changes + Neoplastic chondrocytes
(malignant) mainly affects the axial skeleton than the appendicular skeleton
14-year-old - regularly undergoing imaging - family history of early onset of cancers lots of
death in family from cancer - a mutation in which of the following genes is the most likely cause
of this family predisposition? -Answer- TP53

60 year old with psoriasis - comes top physician because of progressive joint swelling and pain
of her hands - joint tenderness and effusion - treatment with methoxsalen and ultravioletA light
(PUVA) - therapy would be contraindicated in the patient if she had the following? -Answer-
Porphyria Cutanea Tarda

14 y/o old boy - brought to physician - daily headaches - bilateral aching of temples - mother
states he hasn't been himself. He is clumsy with frequent falls and school grades have
declined.n. PE, shows Broad based + Ataxic gait. He alert and oriented to person place and
time, but slow to answer questions - Chronic abuse of which substances causes patient
condition? -Answer- Inhaled Glue
A 8 y/o - with ADHD - a drug with the following action is most likely to be useful for his
patient? -Answer- Stimulation of the release of biogenic amine neurotransmitter
5 year old boy brought to physician for a well-child exam. He recently emigrated from Nigeria -
Skin red-bronze, red hair, continual horizontal nystagmus - Genetic testing shows compound
heterozygoisty of TYRP1 gene - frame-shift and nonsense mutation are detected. These genetic
changes caused the following? -Answer- Lack of Choroidal pigment in macula Eumelanin = real
Pheomelonin = Fake
A 47 y/o TX with Colchicine for acute gouty arthritis - drug acts by inhibiting the
following in the patient's leukocytes? -Answer- Tubulin Polymerization
46 y/o old 2 month history of prolonged bleeding from minor cuts - mild anemia,
thrombocytopenia, leukopenia - bone marrow shows lipid laden macrophages - if incidence of
disease among Ashkenazi Jews in 1/900 - approximate carrier frequency of mendelin trait in
this population -Answer- 1/15
q^2 = 1/900 --> q = 1/30 Carrier frequency = 2pq. rare autosomal recessive
diseases the carrier frequency of 2pq ≈ 2q.

2pq = 2q = 2(1/30) = 2/30 = 1/15

48 y/o woman - fatigue, weakness, loss of appetite, weight loss. Parathyroid hormone
concentrations are increased - X-ray shows osteopenia - subperiosteal resorption within the
phalanges - which mechanism is most likely to cause the skeletal changes -AnswerParacrine
Stimulation of osteoclasts by osteoblasts
Hyperparathyroidism - bone lesions
Osteoblasts increasing RANK -L expression to bind to RANK on Osteoclasts and
stimulating them ---> inc Bone Resorption
50-year-old woman - 3 days of nausea, vomiting, upper abdominal pain radiates to back - DX
with HIV infection adherent to her antiretroviral medication regimen - low CD4+. PE shows
epigastric tenderness, abdominal distention, absent bowel sounds - lab studies show serum
amylase activity of 450 u/L. Most likely cause of findings is adverse effect of which drug? -
Answer- Didanosine "DIDanosine causes pancreaDIDis"

23 yo woman w/ vagina sore. Sexually active, inconsistent condom use. Pelvic exam nontender
ulcer on vulva. Spirochetes positive on dark field. What is biopsy histology Answer- Obliterative
Endarteritis with lymphocytes and plasma cells

25-year-old woman 3-week history of bleeding gums while brushing teeth and easy bruising -
Pt. delivered healthy child 9 months ago - nomedications - except for oral contraceptive -
Irregular bruises are noted on legs, thighs, and arms. Numerous petechiae. Lab studies show
low platelet count. Cause of her condition? -Answer-
Autoantibodies against platelet glycoproteins
Immune Thrombocytopenia

56-year-old man - surgical resection of duodenum after injuring his small intestine in
MVC- which is most likely expected to decrease after procedure? -Answer- Release of
Cholecystokinin

28 year old - chronic renal failure (creatine clearance less than 20 ml/min) - requires treatment
for volume overload and pulmonary edema-- Which is the most appropriate
pharmacotherapy? -Answer- Furosemide Loop diuretics - work with GFR < 30
Thiazides do not work with GFR > 30
19 year old college student - "hear voices" - which drug is most appropriate treatment of
patient" -Answer- Risperidone

66-year-old - colon cancer - receiving chemo -severe diarrhea - Physician prescribes an


opioid antidiarrheal agent - which opioid prescribed as antidiarrheal agent? -Answer-
Loperamide
19-year-old woman ED 30 mins after falling on outstretched hand - physical tenderness of
anatomical snuff-box. X-ray of wrist shows fracture of which carpal bone? -Answer- Scaphoid
Fall with outstretched hand - fracture = scaphoid
Dislocation = lunate
56-year-old man renal transplant - five minutes after graft blood vessels are
anastomosed to host- he develops hemorrhage and thrombotic occlusion of the graft
vessels - which is most likely mediator of the process? -Answer- IgG plus Complement
4-year-old boy falls and lacerates lower leg to subdermal connective tissue - first step in
tissue repair? -Answer- Formation of Fibrin Clot

57 y/o with SOBS- Thoracentesis planned with patient seated in -- needle placed in
which of the following intercostal spaces in midaxillary line? -Answer- Ninth
While lifting weights, 24yo M develops painful swelling in right inguinal region that cannot be
reduced. Photograph shown of small intestine resected at exploratory laparotomy. Dx? -
Answer- Strangulation
Indirect inguinal hernia - possible complication is strangulation ischemia and
necrosis

4-week-old - male - newborn - persistent nonbilious projectile vomiting after eating - visible
peristalsis - firm 2 -cm ovoid mass in right upper quadrant - likely DX? -AnswerPyloric Stenosis

During experiment, investigator observes - Glutamate depolarizes cells through both the NMDA
and nonNMDA receptros . She finds the initial depolarization phased of glutamate -induced fast
excitatory post synaptic potential is generally mediated by the activation of non-NMDA type
glutamate receptors - which of the following is best explanation? -Answer- NMDA receptors are
blocked by Mg2+ at the resting membrane potential

45-year-old F- 1 week severe abdominal pain - history of gallstones - increased amylase and
lipase - CT scan of abdomen shows enlargement of pancreas and dilation of pancreatic ducts -
gallstones lodged in which of the following locations? -Answer- hepatopancreatic ampulla
ampulla of vater
Investigator studying - pulmonary lymphatic flow - Which increases pulmonary lymph
flow? -Answer- intravenous infusion of 0.9% saline
48-year-old man with possible hypertension. On basis of ten measurements, the patient's
average diastolic blood pressure is 113, and standard deviation is 8. If four rather than ten
measurements are made, which is the expected impact on 95% confidence interval? -Answer-
increase in width

50-year-old - azotemia - renal ultrasound shows bilateral hydroureters and hydronephrosis -


most likely cause is primary carcinoma of which of the following? Answer- Uterine Cervix
cervical carcinoma - associated with hydronephrosis
11 year old boy - persistent pain in right knee - tender distal fermur- x-ray shows osteolytic
mass that he eroded through the cortex - biopsy shows hyperchromatic pleomorphic
nuclei - the cells are surrounded by an eosinophilic matric - tumor most likely spreads to? -
Answer- Lung osteosarcoma.

Immunosuppressive - agent cyclosporin A - potent T lymphocyte - successfully prevent graft


rejection by action of which steps in T-lymphocyte recognition/activation? -Answerinterleukin 2
gene transcription

FOXO transcription factor responds to insulin signaling by altering the transcription of several
genes- which describes the reversible ways in which insulin signaling regulates FOXO activity -
Answer- Nuclear:Cytoplasmic Shutting YES Serine Phosphorylation: Yes.
Ubiquitin Proteolysis: NO
65 year old - TX for metastic breast cancer unresponsive to chemo - never married and has no
children. Throughout her illness, another woman - introduced as "my close fiend" has been
with her during hospitalization. Pt is moved to inpatient hospice - sign states only family
members - Her fiends states "we can't bear to be apart. It would be cruel to separate us now".
Response from physician? -Answer- The two of you seem to have a very important relationship.
Of course you may stay together

62 year old - early Alzheimer's - retired and lives with husbsnd - what is the mostly likely way to
help patient maintain her present functional level? -Answer- instruct the patient to keep notes
and lists to help her memory

40-year-old women - Streptococcus pneumonia - which of the following is produced by the cell
is responsible for the restitution of normal lung function? -Answer- Surfactant

55 y/o man - 2 month history of fatigue and a 3 day history of nausea and diarrhea - 5.4 kg
weight loss Physical exam shows dry skin with hyperpigmentation and delayed cap refill. Lab
show Na+ low. Which serum hormone decreased in patient? -Answer- Cortisol

. A study to assess normal mean serum urea nitrogen in mean ages of 65 years. Which
measurement will give precise but inaccurate estimate of mean serum urea nitrogen in this
group? -Answer- 500 Men from a list of patients scheduled to be examined by the urologist
4 moth baby -rare autosomal recessive skeletal dysplasia involving endochondral bone.
Genetic analysis shows null mutations in gene for a protein that controls traffic of
vesicles into golgi complex. Which is the most likely show on findings? -Answer- dilated
Rough Endoplasmic Reticulum
64 year - cough, dyspnea, chest pain - x-ray - shows density in hilar region- biopsy shows small
round cells with little cytoplasm arranged in infiltrating sheets that have
neither glandular nor squamous organizations. - Which of the following abnormalities in
serum?a. Hyponatremia -Answer- Hyponatremia
Pt with 2 month history of foul smelling, watery diarrhea, flatulence - 10lb weight loss - after
trip top Indonesia - no access to clean water -Photomicrograph of stool specimen-
what is MOA of drug -Answer- Formation of Destructive Free Radicals Metronidazole -
MOA Formation of Destructive Free Radicals

80-year-old woman cannot concentrate urine above 450 mOsmol/kg - she accumulates 450
mOsmol/day - She loses 900 mL of water and 100 in sweat and feces - minimum volume of
water to prevent increase in plasma osmolality? -Answer- 2.0
1 L losses from kidney + 900 mL insensible + 100 mL in sweat and feces = 2L losses -> need to
ingest 2 L of water to replace.

75-year-old man dementia, Alzheimer type in nursing facility - wife and children visit weekly -
trouble eating - in speaking with patients family- most appropriate for physician to address
issues of feeding tube in which manners? -Answer- encourage the family to come to a
consensus based on their perception of the patient's wishes
20-year-old woman 12 weeks gestation - genetic counseling - she has a reapir for cleft lip,
other members of family have cleft lip- which is mostly mode of inheritance of this disorder? -
Answer- Multifactorial Only abnormality = Cleft lip

24-year-old - history of drug abuse has nausea, rhinorrhea, hypertension, and tachycardia - The
most likely cause is withdrawal from which of the following agents? Answer- Heroin

35 y/o veterinarian - headache, cough, fatigue, and myalgia - after delivery of lamb - Temp 102F
- Leukocyte count is 12K - X ray shows patchy infiltrates - gram stain of blood and sputum are
negative for pathogens. After 2 days started on doxycycline and symptoms alleviate- Which is
causal organism? -Answer- Coxiella Burnetti
3-day old female newborn - 4 hour history of yellowing of eyes. Jaunice and mildly
increased total bilirubin (14.5 n <12). New born placed on phototherapy - Which
describes mechanism? -Answer- Increased conversion of bilirubin to water soluble
isomers

Phototherapy = bilirubin is converted to water soluble isomers that are then able to be
excreted by the kidney.

2 week male - yellow skin for 7 days - mother is obese - Newborn severely lethargic and
hypoglycemic - fed with soybean formula until 4 days ago. He has been breast fed-since
then. PE shows jaundice. No glucosuria - Increased serum direct bilirubin
concentration- -Answer- Galactose - 1-phosphate uridyltransferase
17-year-old T1DM - multiple admissions to hospital during past 5 months for DKA - Appears
well She tells the physician that she does not take her insulin regularly - forgets or too busy.
Which is most appropriate initial step? -Answer- Acknowledge patient reason for missing insulin
1. acknowledge resistance to change
2. address discrepancies between behavior and long-term goals
3. enhance motivation to change
4. nonjudgmental

A 48-year-old - complete loss of hearing in his right ear. Which CN on the photograph of ventral
surface of brain stem is most likely damaged? -Answer- C - right vestibulocochlear nerve -
ipsilateral
Loss of hearing in right ear = right vestibulocochlear nerve - ipsilateral
Feeling and hearing loss
Start at the pontomedullary junction and count from superior to inferiorly (or medially to
laterally): VI, VII, VIII, IX.

In a study of ultrasonography - early detection of prostate cancer - 35/50 men with prostate
cancer have abnormal test results and 20 of 200 men without prostate cancer have abnormal
results. What represents specificity? -Answer- 80% specificity = TN/(TN+FP) = 80/100 = 0.8 (in
% will be 80%)
A 24-year-old - 3/6 systolic murmur - Aorta pressure< Left Ventricle. DX? -Answer-
Aortic Stenosis

Transmission of bacterial agent person-to-person contact - most likely to occur in which


infectious disease? -Answer- Meningococcal Meningitis
sharing respiratory and throat secretions (saliva or spit).
Cholera = Fecal oral
16 year old girl - ER - 30 minutes after MVC - BP 70/40 0. PE shows cold, clammy, skin. Chext
Xray shows fracture of 9th rib - rupture of spleen. Ligation of blood supply to spleen - most
likely to damage which additional structure? -Answer- Tail of Pancreas

. A study - mean age of muscular dystrophy - Pt divide into 2 group- Normal and subnormal
intelligence - Normal Age of onset 36 - Mean 3.8 - STD 1.8. If age of onset for normal (gaussian)
distribution - the % of normal intelligence who have onset at less than 9 years of age is closest
to which of the following? -Answer- 99%
Therefore, since we know +/- 2 SD covers 95% of the bell curve, it must be higher than that. The
only option higher than 95% is 99%

A 2 week old F - Mexican - well child exam - baby is inconsolable after feeding with formula -
Mother states "I am sure she has the evil eye". Mother has been passing an egg over the baby's
body- PE shows no abnormalities - Which is most appropriate physician statement? -Answer-
Why don't you continue with what you are doing, and let's try changing her to a different
formula.
12 y/o F - tonic-clonic seizure. Pt has mild mental retardation, had surgery for lens
dislocation. Cystathione concentration decreased. A CT scan shows a small infarction of right
cerebral hemisphere. Which serum amino acid concentration is increased? - Answer-
Methionine
HOMOCYstinuria
Homocysteine in urine,
Osteoporosis,
Marfanoid habitus,
Ocular changes (lens subluxation),
Cardiovascular effects (thrombosis and atherosclerosis stroke and MI),
kYphosis, intellectualdisability, fair complexion.

43 y/o woman - fainted 3x in past 5 months. Weakness in face and right arm. Vital signs in
normal limits. Cardiac exam - mild - grade 2/6 diastolic murmur over the apex - murmur louder
when patient rises from prone position. Cause of patient syncope? -
Answer- Left Atrial Myxoma
Left atrial myxoma can mimic mitral stenosis diastolic murmur with multiple syncopal episodes

triad of
1) mitral valve obstruction (i.e. malaise, symptoms of cardiac failure, syncope, etc.),
2) symptoms of embolism (i.e. facial and right arm hemiparesis in patient), 3)
constitutional symptoms (fever, weight loss)
36 y/o - genetic counseling with husband. No family history of genetic syndromes - They have
read that risk of being carriers of autosomal recessive neurodegenerative disorder that occurs in
1/10,000. Which represents carrier frequency? -Answer- 1 of 50
Hardy-Weinberg equation = 1= P^2 + 2pq + q^2
Carriers q^2 = 1/10,000
Square root. = 1/10,0000 = 1/100
Then remember P + q = 1------> 1/100 + p = 1 (p = 99/100) Lastly plug
back into Hardy-Weinberg Equation as:
2pq = Heterozygote carrier
(2 x 1/100 x 99/100 = 2/100 = 1/50)

32 y/o man - lives at sea level - travels to mountains -develops SOBS om arrival - then develops
exercise tolerance increases after 2 weeks. If this patient's bone marrow were examined, it
would most likely show increased numbers of which cells? -Answer- Normoblasts

32 year old -Diastolic pressure between 100 and 110 mm Hg. Healthy and is not taking any
meds. Serum concentrations are normal. Urine contains only trace amounts of protein and no
albumin. He is unlikely to develop peripheral tissue edema because of regulatory adjustments
in which of the following? -Answer- Precapillary Resistance
Systolic BP is determined by
Cardiac Output
Diastolic BP is
determined by arterioles
A 28 yesr old man - High triglycerides - responds
to gemfibrozil - Nicotinic acid therapy
started. Which of the drugs administered
along with Nicotinic Acid to prevent most
common adverse effect of
therapy? -Answer- Aspirin
A 45 y/o woman - undergo a lymph biopsy -
Surgery resident asks 3rd to get a signed
consent before the procedure is preformed
- Which action by student is most
appropriate? -Answer- Accompany resident
while the resident obtains signed consent
35 y/0 F - undergoes vaginal delivery - A median episiotomy is done to hasten delivery. During
procedure, an incision is made posteriorly from the posterior vaginal
commissure. Which muscle is greatest risk of damage if the incision is torn? -Answer- External
Anal Sphincter
Perineum - Anal sphincter - Rectum

A 74 y/o with COPD -breath sounds are decreased and distant auscultation of chest. In addition
to albuterol, which is the most appropriate pharmacotherapy? -Answer-
Ipratropium
("I pray I can breathe soon!")
Muscarinic antagonist - 241-1

A 65 y/o with erectile dysfunction. Most appropriate for physician to prescribe a drug that
increases the actions of cGMP in which of the following cells types in the penis? Answer-
Smooth muscle of deep artery
A 40 y/o with pins and needles sensation starts in his left hand and then spreads to his
left arm and face. Episodes last for 1 minute. PE show no neuro abnormalities. The
most likely cause of these symptoms is a lesion in which of the following gyri of the
cerebral cortex? -Answer- Postcentral

primary somatosensory cortex, the main sensory receptive area for the sense of touch
33-year-old woman - she has celiac sprue and fat malabsorption. PE shows asthenia. Lab shows
serum calcium concentration of 7.8 mg/dl. Which additional sets of serum findings is most likely
in this this patient? -Answer- Phosphate Decreased
PTH Increased
Calcitriol decreased
celiac sprue = fat malabsorption = Vit KADE

Vitamin D malabsorption = decreased serum calcium—decreased

Vit D Deficiency = ca decreased, Phosphate decreased, PTH increased


A 40 y/o comes to physician for health maintenance exam. Concerned about male pattern
baldness. Which of the following treatments is most appropriate medication for patient? -
Answer- Finasteride
5α-reductase inhibitor (conversion of testosterone to DHT).

A 6-month-old - failure to thrive and intractable diarrhea that hes worse when he feeds
PE shows eczematous dermatitis and mild lymphadenopathy. Mutation is FoxP3, a
transcriptional protein. Most likely cause of patients condition is a decrease in which T-
lymphocyte? -Answer- Regulatory T Lymphocyte
IPEX
.

35 y/o comes to 4-month history persistent cough and intermittent nasal discharge.. He often
awakened from sleep with shortness of breath, wheezing, dry cough, chest tightness. Vital signs
are within normal limits. PE shows no abnormalities. The most appropriate drug blocks the
receptors for endogenous substances that is a metabolite of which of the following? -Answer-
Arachidonic Acid precursor of leukotrienes = arachidonic acid.
A 44 y/o female - steatorrhea, flatulence, and a 22lb weight loss over the past 6t months.
MCV low. Negative stool occult blood. Stool cultures and exam are negativge. A lateral x-ray
shows mild osteopenia. Which is most likely explanation? -AnswerCeliac Sprue decreased
absorption in distal duodenum and proximal jejunum - 381-1

iron deficiency anemia

44 y/o man - farmer - spraying insecticides - brought to ER - because of vomiting, diarrhea,


wheezing, and sweating for 12 hours. He has excess salivation and muscles weakness. Confused
and pupils are small. A drug acting at which of the following labeled sites is most appropriate? -
Answer- Area labeled D - atropine (antimuscarininic) Organophosphates - insecticides, and
function by binding acetylcholinesterase and "deactivating"

44-year-old man - sustained injury in a bar fight. Multiple facial fractures and contusions,
including several swollen right eye. Lots of edemas. Pt refuses to open eye and has double
vision. CT scan shown. Which is most likely explanation of patients visual complaints? -Answer-
Entrapment of the inferior rectus muscle
65 year old man - ER because of substernal chest discomfort. Which of the following
eicosasnoids is most likely contributing to symptoms? -Answer- Thromboxane A2
A man touch surface of hot stove -20 minutes later. A blister develops at the site. Biochemical
analysis of the blister fluid at this time is most likely to show the presence of which? -Answer-
Histamine
Immediate Type 1 Hypersensitive - 20 minutes
Sattar - increase capillary permeability in the postcapillary venules and vasodilation in
the arterioles as actions of histamine.

A 40 y/o m an with terminal small cell carcinoma - Na+ 120 mEq. Which is the most
appropriate therapy? -Answer- Demeclocycline
61 y/o woman - health maintenance exam. She has split S1 over left sternal border. Which
explains first component of this sound? -Answer- Closure of the mitral valve First heart sound
(S1) = two heart valves: the mitral valve and tricuspid valve.
65 year old - progressive ataxia with gait unsteadiness. There is an intention tremor on right
side. Most likely cause is damage to which of the following areas? -Answer- Right
Cerebral Hemisphere

Patient presents with both ataxia and an intention tremor on the right-hand side

cerebellar lesions like we see here always present IPSILATERALLY as opposed to


many other CNS lesions. - Double crosser
7 year old - 2 day history of itchy red rash on right arm and trunk after a hike in woods. Rash
started 1 day after trip. PE shows 2-cm patches of raised, red, elongated blisters that ooze clear
fluid on the upper extremities and trunk. The blisters are linear with patches of erythema.
Which describes the mechanism of patients reaction? -Answer- Release of interferon gamma
Th1 Cells
60 y/o man - 1-month history of headaches and dizziness. His blood pressure is 160/85. PE
exam shows moderate splenomegaly. Lab shows - MCV low, Lymphocytes 10%. Red Cell
distribution Width increased. Which is most likely decreased in patient? Answer-
Erythropoietin'a.
Low EPO in Polycythemia Vera due to Negative feedback on EPO release by kidney.

A female neonate is delivered 38 weeks gestation. PE shows macrocephaly and shortening of


the proximal segments of all limbs. A DX of achondroplasia. DNA sequence of sows a G-A
mutations in the fibroblast growth factor gene on chromosome 4. Biosynthesis of this protein
requires functional signal recognition particle receptor.
This receptor is most likely found at the cellular location? -Answer- Rough Endoplasmic
Reticulum

60 y/o T2 DM - 6 week history of right shoulder pain. The pain occurs when attempts to raise
her arm when lying on right. No weakness, tingling, or numbness. PE shows tenderness to in
the region of deltoid muscle just lateral to acromioclavicular joint. While sitting upright, she is
able to abduct her arm to nearly 90 degrees, but additional abduction is prevented by pain.
Muscle strength is 5/5 in right upper extremity, sensation intact, there is no impingement sign.
DX? -Answer- Subacromial Bursitis

15 year old boy - well child exam - if patient develops severe acne vulgaris that resolves over
the next 3 years, a rapid increase in the secretion of which of the following substances will have
been the most likely cause of acne? -Answer- Testosterone Androgens cause acne
An 85 y/o man - progressive forgetfulness for 3 years. Mental status exam shows difficulty
learning and recalling new verbal and nonverbal material. Which is appropriate
pharmacotehrapy? -Answer- Donepezil ACh-esterase inhibitor.
Alzheimer disease (DonaRiva dancesat the gala)

Investigators conduct a prospective, community based study to assess the effectiveness of a


intervention. Which sets of characteristics best supports causal relationship between the
intervention and child language scores? -Answer- Strength of association, temporal
relationship. Dose-response gradient
An otherwise healthy 26 y/o scuba diver - difficulty breathing after ascending from a 30 minute
dive at 15m. Which physiological signs is most consistent with the x-ray of chest scan? -
Answer- Hyperresonance over most of the left side of the chest a. Decompression sickness
(DCS, or "the bends") involves gases ... Pulmonary barotrauma may result in a collapsed lung
(pneumothorax).

Investigator studying outbreak of virus in skilled nursing home. Virus infectivity is destroyed
by heating to 60 C for 30 minutes. Based on this find, the virus has which characteristics? -
Answer- Enveloped

A 76 y/o woman - brought to physician by her son who is concerned that she is forgetful. He
thinks she should be placed in a long term care facility. She appears cheerful. Which is most
crucial in determining her decision-making capacity? -AnswerShe expresses clear and consistent
values

An 18 month -old girl - failure to thrive and frequent wet diapers. 3rd% for length and weight.
PE shows dehydration, decreased muscle tone, and bowing of legs. Urine studies down ph of 5,
glucosuria, phosphaturia, and generalized aminoaciduria. Which is cause of metabolic acidosis
in this patient? -Answer- Decreased sodium bicarbonate reabsorption in the proximal tubule
A 42 y/o comes to the office because of a 1-year history of an enlarging, painless mass
on her right wrist. Exam of the right show in photograph. The mass is firm nontender
and transilluminates with the application of a pen light. Which of the following is most
likely to occur? -Answer- Spontaneous Regression

ECG of heart study in anesthetized experimental animal. ECG shows (bradycardia).


Which of the following procedures most likely to produce ECG shown? -Answer-
Ablation of the atrioventricular node

A study is planned to assesses HTN control between patients in the USA and Western
Europe. Study population includes 21,053 patients with HTN receiving 291 cardiologist
and 1284 primary care physicians. The main outcome measure is HTN control defined
as a most recent blood pressure reading less than 140/90. Which describes study
design? -Answer- Cross-sectional study
A 2-year-old boy - ER department - severe abdominal pain and vomiting for 3 hours - PE shows
yellowish white papules on back, abdominal tendernes, and hepatosplenomegaly. Serum
studies show an increased in amylase activity and increased total cholesterol, triglycerides, and
chylomicrons. This patient most likely has a deficiency in which activity -Answer- Lipoprotein
Lipase

A 16-year-old - 4-month history of nonbloody diarrhea. Mother concerned she is abusing


laxatives to maintain a low weight. BME is 19. BP 89/61. Which is serum
electrolyte concentration? -Answer- Potassium decreased
Chloride increased
Bicarbonate decreased
Diarrhea causes non-anion gap (i.e. hyperchloremic) metabolic acidosis

excessive diarrhea = excessive loss of HCO3- and K+.


'Chloride levels in the serum will be increased due to the normal HCO3-/Cl- equilibrium,
so as
42-/o history of recurrent pyelonephritis - 2-day history of left flank pain, fever, and chills.
Temp is 101.8F. He appears anxious and in moderate distress/. PE sows left costovertebral
tenderness. Urine is cloudy with a pH of 7.3. CT shows a large renal calculus (photograph).
What type of renal calculus? -Answer- Magnesium Ammonium Phosphate
Staghorn Calculus in adults
63-year-old woman - 2-month history of progressive shortness of breath with exertion. A
chest x-ray shows a mass pressing against the outside of the trachea. Pulmonary
function is most likely to show which of the following? -Answer- FVC decreased
FEV1/FVC decreased

Peak Expiratory Flow Rate decreased


Peak Inspiratory Flow Rate decreased
A cohort study to examine the association between cigarette smoking an lung cancer in 4000
people. 1000 smoke cigs and 3000 do not smoke. After 10 years, 30 cases of lung cancer in
smokes and 30 cases of lung cancer among nonsmokers. According to results, how many lung
cancers attributable to smoking would be expected in a group of
10,000 smokers over the same period? -Answer- 200
Attributable risk = incidence in exposed - incidence in unexposed
=30/1,000 (smokers) -30/3,000 (nonsmokers)=0.03-0.01= 0.02 (so the attributable risk
is about 2%)

Applying it to a population of 10,000:


=0.02*10,000= 200

An 8 y/o boy brought to physician by his mother -noticed a lump on left side of neck-
Exam shows pectus excavatum. 2 mm subcutaneous nodules on lips, and irregular 2cm
mass on neck, anterior cervical lymphadenopathy. 14 year sisyer - similar findings at
age 10. Which studies support diagnosis? -Answer- Serum Calcitonin Concentration
Marfanoid habitus + Mucosal neuromas + Neck mass =

MEN 2B (PMM) Pheochromocytoma

Medullary thyroid CA (Calcitonin secreting), mucosal neuromas


Which of the following pairs of drugs interferes with DNA synthesis by crosslinking? -
Answer- Lomustine and cyclophosphamide
A 75 year old woman - stress urinary incontinence with sphincteric incompetence. Pelvic floor
muscle exercise and the use of vaginal pessary provide inadequate improvement.
Administration of which of the following mechanism of action is most appropriate? -Answer-
Stimulation of a-adrenoreceptors
α1 stimulation (via α1 agonist) constricts the bladder sphincter thereby, preventing sudden
bouts of micturition during coughing/sneezing

35-year-old woman - 6 month history of fever, fatigue, and unexplained weight loss. She has
had 40lb weight loss. Lab show CD4 T lymphocytes 60. The most appropriate treatment
includes a drug that targets which of the following steps of viral replication? - Answer-
Synthesis of viral DNA

A female newborn - respiratory distress - PE shows intercostal retractions. A chest x-ray shows
multiple rib fractures. Skeletal x-rays show multiple fractures of the long bones.
There is no family of history of recurrent fractures. Most likely diagnosis? -AnswerOsteogenesis
Imperfecta, type I

A 42 y/o man - intermittent high-grade fever, dizziness, diarrhea, and fatigue. 5kg weight loss.
During trip to India, bitten by an insect. He states sore developed at the site of bite. Temperature
is 100.4. PE shows splenomegaly and muscle wasting. Lab studies
show pancytopenia. Which vector of patient's infection? -Answer- Sand Fly Visceral
Leishmaniasis
Which of the following types of vaccine is most appropriate to prevent mumps?
-
Answer- Live attenuated virus

Killed = Rest In Peace Always


RabiesInfluenzaPolio(Salk)Hepatitis A
45-year-old - right sided abdominal pain. She pain after high-fat meals. Which is most
likely on abdominal exam in patient? -Answer- Inspiratory pause on right upper
quadrant palpation

A 21 year old - 1 month history of fever and abdominal pain; he also had a 6.8 kg (15-lb)
weight loss during this period. He returned from Middle East 3 weeks ago. PE shows
generalized lymphadenopathy and hepatosplenomegaly. Lab studies show
pancytopenia. Cause? -Answer- Leishmania

. A 50-year-old from Minnesota - 3 day history of fever and malaise. Test for Anaplasma
phagocytophilum is positive. Causal organism is transmitted via the same vector as
which of the following? -Answer- Babesia microti and Borrelia Burgdorferi
55-year-old - construction worker - bilateral basilar, fine, end-inspiratory crackles. A chest X-ray
shows reticular pattern and bilateral diaphragmatic pleural plaques. No atypical cells.
Pulmonary function test is likely to show? -Answer- FEV1/FVC normal
FVC decreased
DLCO decreased
3-month-old girl - 2 month history of enlarging red spot on her abdomen - She appears
well nourished- A photograph of the lesion is shown. Microscopic exam of the lesion is
most likely to show which of the following? -Answer- Densely packed capillaries
Strawberry hemangioma
A 44 y/o woman - Pap smear - dysplastic columnar ciliated cells. If the patient were healthy.
Columnar cells would most likely originate from which of the following areas? Answer- Cervical
Canal
Transformation Zone Endocervix= simple
columnar epithelium Vagina = Stratified
squamous
65 year old with DKA - fever and proptosis of left eye for 3 days- A black eschar is
present of nasal mucosa. Which is likely DX? -Answer- Mucormycosis
36 y/o woman- BP 180/120 - CT scan shows renal artery aneurysm. PE shows no other
abnormalities. A CT scan of abdomen renal angiogram obtained from femoral approach. (String
of beads) Which is most likely cause of findings? -Answer- Fibromuscular Dysplasia 'string-of-
beads'

A 2-week-old girl - asymmetric left pupil Ophthalmologic exam shows coloboma. Incomplete
closure of the embryonic fissure is limited to which of the following structures of the left eye
most likely to spare this patient vision? -Answer- Iris "keyhole" appearance of the pupil,
generally do not lead to vision loss.
A 44 year old - invasive pancreatic cancer - 2 week history of severe right side back and
abdominal pain. 6.8 weight loss. PE shows no abnormalities. An operation is scheduled
to relive pain. Most likely target of pain relief is which of the following labeled sites in
cross section of spinal cord? -Answer- Area Labeled H

Pain & temperature fibers for the right side come in on the dorsal right side, cross at the
anterior white commissure, and travel up in the Spinothalamic tract.
62-year-old woman - brought to physician because her left foot has become cool and
discolored. She has had difficulty walking - Symptoms began 4 hours ago with several acute
episodes of palpitations followed by tingling and numbness of the left foot several hours later.
Which is the most likely explanation for findings? -Answer- Embolus to the femoral artery with
left leg ischemia
Cool, discolored foot with tingling and numbness = blood clot. Cardiac symptoms may indicate
that she has some sort of thrombotic disease

. An antiserum is elicited by immunization with protein X and anti-X antibodies are absorbed
to microtiter wells. Graph shows the amount of radiolabeled protein X bound by antiserum of
varying concentrations by protein Y. With respect to ant-X serum - which of the following
interpretations of the epitopes expressed by the proteins? - Answer- Proteins x and Y have no
epitopes in common
18 year old woman - progressive muscle weakness, poor exercise tolerance, shortness
of breath. She has bilateral ptosis - Examination shows abnormal accumulations of
mitochondria in subsarcolemmal regions. Which additional set of findings is most likely
-Answer- Max Oxygen Consumption decreased
Venous Blood Lactate increased

Energy Production by Glycolysis increased


?

21 year old man ER - 30 minutes after sustaining knife wound in abdomen - He is in shock. He
undergoes immediate operative repair of small intestine and bleeding wound
to the liver. 6 months later he is admitted to hospital for treatment of intestinal obstruction.
During the corrective abdominal operation, it is observed that his previous injury has
regenerated without any evidence of fibrous scarring. Which of the following sites is most likely
being described? -Answer- Liver
70 y/o man with severe congestive heart failure and orthopnea. During this time, urine
output is decreased to 300 ml/day. Which of the following sets of laboratory finds is
most likely? -Answer- BUN 40
Creatinine 2
Urine Na 5+
Specific Gravity 1.025

Prerenal azotemia, BUN:Cr ratio is >= 20;


Activation of the RAAS system

spec grav is high - holding onto sodium so urinary sodium will be low (<20, FENa <1%).
A 26-year-old woman comes to the physician because of progressive itchy red rash on her arms.
She works at an animal clinic. She was scratched on both arms while caring for litter of kittens.
PE shows annular erythematous plaques with central clearing and advancing scaly borders.
There is no regional lymphadenopathy. Most likely diagnosis?
-Answer- Tinea Corporis ring
worm from cat

56-year-old man - severe pain in great toe - He is allergic to aspirin. Microscopic exam shows
joint fluid shows negatively birefringent crystals. Which is most appropriate acute treatment? -
Answer- Colchicine

A prospective study is done to assess two different antihypertensive meds. In the study,
patients were randomly assigned to two groups. At end of study, some participants reported
inconsistent adherence. In their primary analysis, which of the following methods should the
investigators use to analyze data from patients who were not adherent to regime? -Answer-
Include the outcome for each participant in the group to which he or she was randomized
intention to treat
A 2-year-old girl - cross eyed - PE shows moderate strabismus - if not treated, she will
most have deficits in depth perception due to lack of appropriate competitive
interactions in visual cortex. Calcium entry - mediates outcomes of this competitive
process? -Answer- N-Methyl-D-aspartate (NMDA)

A 61-year-old man - prescribed fluoxetine for depression. This drug has its initial effects
on neurons in which of the following structures? -Answer- Raphe Nuclei

a string of nuclei in the midline of the midbrain and brainstem that contain most of the
serotonergic neurons of the brain

45 year old man - found unconscious in the desert - 24 hours ago - he had been hiking
and lost consciousness. Pulse is 124 min and BP 80/40. PE shows multiple
ecchymoses on the upper and lower extremities. Which explains hypotension?
-Answer-

Hypovolemic Shock
dehydration since - in the desert.
A 22 year old - brought to ED 30 minutes after a friend found him unconscious. Drug
overdose is suspected. He is intubated using PEEP - which of the following sets of findings
best describes effects of PEEP? -Answer- Peep Inspiratory Pa.
Positive End-Tidal Pa Positive
Peak Inspiratory Positive End
tidal Volume Positive
40-year-old - enlarging breast tissue - He is taking a diuretic - Which drug is most likely
causes adverse effect? -Answer- a. Spironolactone
A 68-year-old man - treated with NSAIDs for RA - has severe
gastric burning and discomfort. A second drug decrease these adverse
effects, but the patient develops severe diarrhea. The second drug
most likely acts at which site? -Answer- Area Labeled
B PGE analog -misoprostol - create a barrier

A 65 year old - scheduled for physical therapy 3 days following shoulder operation. As part of
regime - he begins to strengthen the subscapularis muscle. Which of the following movements
of the arm should this patient perform against resistance? Answer- Medial (internal) rotation
subscapularis muscle is very important for the Internal rotation of the humerus. The internal
rotation supports the upper arm during abduction and adduction.

A 65 y/o recovering from cerebral infarction due to atherosclerotic vascular disease.


Clopidogrel is added to medication regime because of intolerance to aspirin. Which is most like
mechanism of decreased platelet aggregation due to clopidogrel? -Answer- Preventing ADP-
stimulated platelet activation

A child with septiciemia has an antibiotic clearance (CL) of .09 L/hr/kg, the steady state
concentration is 12ug/Ml. Which of the following is maintenance dose (in mg/kg/day)? -
Answer- 28.8
Maintenance dose formula is (Css × Cl × tau) ÷ F
=(12 ug/mL × 1 mg/1000 ug) × (0.09 L/hr/kg × 1000 mL/1 L × 24 hr/1 day)= 25.92
mg/kg/day

Which of the following best explains impaired action potentials in the affected axons of
demyelinating diseases? -Answer- Increase in axonal capacitance

"myelin speeds the transmission of electrical impulses called action potentials along
myelinated axons by insulating the axon and reducing axonal membrane capacitance"
47-year-old with a goiter for 10 months. Adequate iodine in diet. Antithyroid peroxidases
antibodies are identified in her serum. Serum concentrations for T3 and T4 are decreased, and
serum TSH concentrations is increased. Which of the following is most likely to be involved in
pathogenesis -Answer- Replacement of thyroid parenchyma by lymphoid cells Hashimoto's
thyroiditis Hürthle cells lymphoid aggregates with germinal centers.

Cyclophosphamide and some other chemo agents kill mitotic cells. Which of the following
cellular compartments is most rapidly depleted during cyclophosphamide therapy? -
Answer- Granulocytes
Granulocytes (Mainly neutrophils) are particularly very sensitive to chemotherapeutic
alkylating agents
4 year old - marked yellowing skin. Boys family is vegan. Examinatio shows yellow- tinged
skin, but his sclerae are nonicteric. Which interventions will most likely improve the child's
appearance? -Answer- Dietary Change excess carotene due to his diet
A newborn has external genital appear to be male. Chromosome analysis shows 46 XX
Karyotype. Which is most likely explanation for physician findings? -Answer-
Hyperplastic fetal adrenal glands
A 48 y/o man = 24 hour history of abdominal pain and blood tinged vomiting. Heavy smoker
and drinker. Abdominal exam shows absent bowel sounds, involuntary guarding, and epigastric
tenderness. An x-ray shows free air in the abdominal cavity. DX? -Answer- Perforated peptic
ulcer

78 year old - found unresponsive in backyard - After patient is admitted to hospital, neighbor
says.
"he always said he wouldn't want to be kept on life support if there was no hope of recovery".
Wife died and no children. Friend has been helping with errands. Physician believes prognosis is
poor. It is most appropriate for the physician to state to the neighbor which of the following? -
Answer- "Thank you for telling me this. Your friend's previous statements to you can be helpful
in making decisions for him"

6-month-old girl - brought to office for well child exam. Patient and moth emigrated from
China- No regular health care. Screening for which is most appropriate next step in
management? -Answer- Hepatitis B

A 46 y/o woman - 1 day history of high grade fever, productive cough, and pleuritic chest pain.
Leukocyte Alkaline Phosphatase elevated. Which findings most reliably distinguishes leukemoid
reaction from CML in this patient? -Answer- Leukocyte alkaline phosphatase activity greater
than 250 U/L

. A 75 y/o with prostate cancer has metastases in spine near L2. Which is most likely route of
metastatic spread? -Answer- Prostatic venous plexus to the vertebral venous plexus

A 19 y/o woman - burned over 35% of her total body surface in fire. Which of the following is
most likely to occur in the patient over the next 7 days? -Answer- Increased metabolic rate

A 48 y/o referred for evaluation of possible HTN. Average BP is 113 mm Hg with a 95%
confidence interval of 110 to 116mm Hg. Which most like represents the 99% confidence
interval for this data? -Answer- 108 to 118
Confidence Interval (CI)- range of values w/in which the true mean of the population is
expected fall.
CI of 95% is 110 to 116 then a CI of 99% has to be a range that is wider... 108 to 118
A 45 y/o - 3 days of left arm pain and weakness. Use of Ibuprofen has provided no relief. Exam
shows weakness with external rotation of the shoulder but full strength with elbow in
extension. The most likely cause of the patient's condition is injury to -Answer- Infraspinatus

teres minor action -Answer- laterally rotates arm


A 27 y/0 - headaches and blurred vision. BP 196/142 mm Hg. PE shows papilledema
and bilateral costovertebral angle bruits. Which serum concentrations is most likely
increased? -Answer- Angiotensin
A 38 y/o woman with hypercalcemia, hypophosphatemia, and phosphaturia is diagnosed with
primary hyperparathyroidism. Which of labeled segments of the nephron most likely involved
in the development of phosphaturia? -Answer- B. PCT PTH acts on the PCT to dec Phosphate
reabsorption--> more Phosphate in the urine. PTH also acts at DCT to inc Ca reabsorption

A 20 y/o woman - 10 day history of vaginal discharge, itching, soreness. Pelvic exam shows
vulvar erythema, edema, and thick white vaginal discharge. Vaginal ph 4.2. Photomicrograph of
vaginal discharge is show. Most appropriate pharmacotherapy is an oral agent with which
mechanism of action? -Answer- Inhibition of the cytochrome
P450-dependent demethylation reaction
-Azoles (fluconazole) (inhibit CYP450 demethylation)

A previously healthy 32 y/p - sudden onset of severe headache, seizures, left side weakness,
and nausea that began while attending a party. Friends say there may have been drugs at the
party. Pulse is 120/min, BP 160/100. Neuro exam shows bilateral dilated pupils, hyperreflexia,
and mild hemiparesis. What is the most appropriate next step in diagnosis? -Answer-
Toxicology Screening picks up any other drugs that the patient might have been using.
So even though the pre-test probability is high for amphetamine use ,the tox screen would pick
that up as well.

A 14 year old boy - tibial fracture - leg cast and progresses satisfactorily for 6 weeks.
One week after cast replaced, he has pain and numbness in the foot of the casted
extremity. Decreased sensation to pin prick on the anterolateral aspect of the leg an
the dorsum of the foot. Cause of nerve compression by a tight cast at which site? -
Answer- Neck of the Fibula
d

A pregnant - 28 y/o - symmetric enlargement of pituitary gland - Which of the cells types
accounts for pituitary enlargement? -Answer- Lactotrope
An investigator studying two drugs, Drugs X and Y to determine which has greater
bioavailability in a healthy human volunteer. The drug has similar mechanism of actions. If Drug
X has a greater bioavailability compared with Drug Y, which parameters is most
likely to be greater for Drug X than Drug Y if both are administered at the same dose? -
Answer- Area under the curve

Represents overall drug exposure


A newborn - 24 in long and 12lb - mother has T1DM. Serum glucose is 20. Which of the following
fetal conditions immediately prior to birth most likely precipitated the newborn's postnatal
hypoglycemia -Answer- Decreased gluconeogenesis baby's mother has Type 1 Diabetes mellitus,
it is plausible that they had elevated blood glucose levels during or shortly before birth.
Insulin does not cross the placenta, but glucose does, so during birth the neonate would
have been hyperglycemic.
This would lead to the neonatal pancreas releasing insulin, driving glucose into cells and turning
down gluconeogenesis; this is why the baby is hypoglycemic right now.

31-year-old - woman - ER because of headache. Exam shows multiple contusions on the face
and torso. Pt states husband assaulted her and she is afraid for her life. She pleads with
physician not to allow her husband to enter the treatment area. What is the physicians most
appropriate next step -Answer- Tell the patient's husband that he is not allowed in the
treatment area, and that he must remain in the waiting area. wife did not grant him the
permission to see him.

A 65 year old with gout - 3 days history of pain and swelling in knee. He is having trouble
walking. Asks physician for disabled parking permit. Most appropriate response by physician? -
Answer- I will authorize the permit, but I recommend that you continue with regular exercise
and only use the permit when you are having severe pain.

DOCTOR decides the eligibility then sends it to the DMV disability


1 y/o - bilateral cleft palate. Both parents have family history of cleft palate. Pattern of
inheritance? -Answer- Multifactorial

Cleft lip and cleft palate have distinct, multifactorial etiologies, but often occur together."
Genetic analyses - obtained on amniocentesis confirm that a fetus has a familial loss of
function mutation that leads to impaired of G-coupled receptors signal transduction. This
defect would most directly impair transcription of genes who expression is regulated by which
of the following hormones? -Answer- Epinephrine epinephrine acts on alpha or beta
adrenergic receptors which are all G protien receptors.
A 32 y/o positive home pregnancy test. A single transaxial ultrasound image of the
uterus shown. The arrow indicates a cystic structure. Which best describes this
structure? -Answer- Yolk Sac

A 35 y/o man - alcoholic - PE shows gynecomastia, hypogonadism, spider angiomata.


Metabolic dysfunction of which organ? -Answer- Liver
A 25 y/o man - brough to ER for 30 minutes after MVC. Abdominal tenderness and swelling.
Treated for spleen rupture. He develops hypotension during operation. Over next 24 hours, he
develops azotemia. Urine volume remains less than 20 mL/h. Urine studies are most likely to
show which of the following? -Answer- Fractional excretion of sodium greater than 2%

A 34 y/o woman - long standing history of alcoholism- frequent foul-smelling stools for 2 weeks.
Admitted for alcohol -induced pancreatitis. Treatment with a pancreatic enzyme replacement.
For dosage adjustments, it most appropriate to monitor which of the following in the patient's
stool? -Answer- Fat
Supplementing with pancreatic enzymes means that you should NOT see for fat in the stool

A 44 y/o woman - 2-week history of episodes of shortness of breath, perioral tingling, vague
chest tightness. Episodes last 20-30 minutes - and relived by alcohol and lorazepam. Pt is
mildly anxious. An ECG shows sinus tachycardia. Lab studies show? Answer- Decreased arterial
PCo2 panic attack.
Hyperventilation drops pCO2 leading to a respiratory alkalosis. po2 is relatively unaffected

A 40 y/o woman - 1 year history of facial puffiness and weight gain. She has URI and HTN. PE
shows increased fat around neck, facial plethora, moon facies, red striae. Serum studies -
increased ACTH - suppressed after an oral administration of dexamethasone. The most likely
cause of findings in this patient is hypersecretion of a hormone from a tumor located in which
organ? -Answer- Pituitary Gland b. Dexamethosone suppresses ACTH = Pituitary Adenoma
A male newborn - delivered 39 weeks gestation - PE shows protrusion of short segment
of small intestine through the abdominal to the right of umbilicus. No layer covering
intestine. DX? -Answer- Gastroschisis
A 68 y/o woman - weakness, fatigue, and weight loss associated with widespread autoimmune
destruction of both adrenal glands. -Answer- Inhibition of H+/K+ ATPase PPIs, which inhibit
gastric H K ATPase

A 68 y/o woman - weakness, fatigue, and weight loss associated with widespread
autoimmune destruction of both adrenal glands. -Answer- Na decreased K+ increased
pH decreased
widespread destruction adrenal glands,

loss of aldosterone - decreased Na+ reabsorption;


increased serum potassium.
not expressing H+ ATPases will lead to H+ not being secreted, creating a more acidic
environment (pH will decrease).

A 31 y/o pregnant woman - 32 weeks gestation- 3 day of fever and chills. Recently immigrated
from Ghana. Photomicrograph of peripheral blood smear shown. Pt is greatest risk of which
complications? -Answer- Hypoglycemia
impair hepatic gluconeogenesis and can also consume glucose for its own metabolic demands.
A 10 year old boy - abrasions on left knee. 1 week later - epidermal cells cover abraded
skin. This stage of wound healing is best explained by the fact that epidermal cells
belong to which of the following classes? -Answer- Labile Cells
regularly regenerate

A 20 y/o woman- 1 year history of low grade fever, fatigue, and joint pain. She develops
a red rash - even on areas not exposed to sun. Malar rash on trunk and upper
extremities. Elevated antinuclear antibody. Studies most likely show a mutation of a
gene encoding which? -Answer- C1q

SLE is associated with deficiency of early complement protein


54 y/o F - with rheumatic heart disease - severe shortness of breath. Crackles are heard over
lung fields. Cardiac exam shows 2/6 systolic ejection murmur. An ECG shows afib and left atrial
enlargement. Which of the following pulmonary mechanisms explains this patient's dyspnea? -
Answer- Intra-alveolar transudates
HF d/t a. fib and left atrial enlargement-> inc hydrostatic pressure->
A 45 y/o - BMI 36 - hypercholesterolemia - Pravastatin is prescribed. Patient is
increased risk of which of the following conditions? -Answer- Myopathy
A 44 year old woman - 3 week history of severe headache - Papilledema - Lumbar puncture
shows an high opening pressure. Treatment with acetazolamide is begun. This drug will most
likely improve patients symptoms by which mechanism? -Answer-
Decreased CSF production by the Choroid Plexus
Carbonic anhydrase inhibitors - treat ICP by reducing cerebrospinal fluid (CSF) production at the
choroid plexus.
A 3 day old male - Down Syndrome - repeated episodes of vomiting bile-stained fluid.
Stomach is distended. Pregnancy complicated by polyhydramnios. DX? -Answer-
Duodenal Atresia
double bubble
43-year-old woman - undergoing chemo for Acute Myelogenous Leukemia - given blood
transfusion for anemia. Shortly after transfusion is started she develops chest pain and
shortness of breath. Results show patient's blood group is O and transfused blood group A.
Which effectors cause of symptoms? -Answer- Preformed IgM antibodies
60 year old woman - lump in breast - fluorescent in situ hybridization shows HER2/neu
receptor is overexpressed with greater than 10 receptors per cell. Which mechanism
cause overexpression of patients tumor? -Answer- Amplification

30 y/o man - increasing fatigue and dyspnea. Fixed wide split S2 and a grade 2/6
murmur heard nest at the left upper sternal border. DX? -Answer- Atrial Septal Defect
A pt with T1DM - prolonged episodes of hypoglycemia following injections of insulin.
Cause of prolonged hypoglycemia? -Answer- Impaired release of glucagon
glucagon secretion is inhibited by hyperglycemia, somatostatin and insulin
A 70-year-old woman - 3-month history of severe pain in joints. PE shows rheumatoid nodules.
The immunologic mechanism of this patient is most similar to which? -AnswerSerum Sickness
A 78 y/o progressive loss of hearing in right ear. Bone conduction is greater than air
conduction. Cause of hearing loss? -Answer- Otosclerosis
BC>AC = conductive hearing loss
AC>BC = sensoneurial hearing loss
82-year-old - fecal incontinence for 2 months. Underlying cause of patient's condition? -
Answer- Decreased function of puborectalis muscle
A 23 year old - woman - family history of breast cancer- concerned about consuming irradiated
food. Initial response by physician? -Answer- Because irradiated food is not radioactive, you will
not be exposed to ionizing radiation by consuming it.
A 24 y/o woman - weeks purple spots on legs. Underwent splenectomy - for immune
thrombocytopenia. Platelet count was low and increased after surgery. 6 weeks ago
peripheral blood smear shows presence of Howell-jolly bodies. Today no howell-jolly
bodies. Which explains current lab findings? -Answer- Accessory spleen
32-year-old - pregnant - in labor - membranes ruptured spontaneously. Epidural analgesia in
initiated. Fetal heart rate shows spontaneous decelerations. Umbilical cord compression is
suspected. Which is best explains the decrease in fetal heart rate because of compression? -
Answer- Increased fetal systemic vascular resistance blocking umbilical veins- reduces the flow
in parallel circulation; thus increases the TPR

70-year-old man - decreased urinary output and fatigue during past 2 days. 6 month history of
joint pain - Treatment with Ibuprofen. Lab show increased BUN/Cr. Cause of increase in
BUN/CR -Answer- Inhibition of prostaglandin synthesis
43. A 50-kg experimental animal receives 1L of 5% saline intravenously. Which of the
following changes in body fluid is most likely in response to 5% Saline -Answer-
Intracellular Volume decreased

Intracellular Osmolality increased

Extracellular Volume increased


Extracellular Osmolality increased
A 50-kg experimental animal receives 1L of 5% saline intravenously. Which of the following
changes in body fluid is most likely in response to 5% Saline? -Answer- Intracellular Volume
decreased
Intracellular Osmolality increased
Extracellular Volume increased
Extracellular Osmolality increased
50 y/o woman - immigrated from the Dominican Republic 20 years ago. Positive
Strongyloidiasis antibody titer. Infection has following modes of transmission? -
AnswerPenetration from the larvae from the soil to the skin

A male newborn with malformed extremity. Woman's husband, mother, and sister also are in
delivery room. Most appropriate initial action? -Answer- Ask the mother and sister to leave the
room and inform the parents of the abnormality. patient privacy
A 25 y/o man - orchiectomy for testicular cancer. A biopsy shown - which is most likely
cells of origin of neoplasms? -Answer- Germ Cells

Seminoma is the most common testicular tumor. It's a germ cell tumor. Commonly see
"fried egg cells".
A 63 y/o - BP 220/160. Required a drug that decreases heart rate and systemic vascular
resistance. Which drug? -Answer- Labetalol
Nonselective alpha & beta antagonists end
- carvedilol -
labetalol

A 65 year old - increasing shortness of breath - low back pain. High output cardiac failure.
Radiologic exam showed intermixed osteosclerotic and osteolytic areas in pelvic bones.
Elevated Alk phos. DX? -Answer- Osteitis Deformans (Paget Disease)

. A 5 y/o boy - URI - develops easy bruising- Platelet count low. Photomicrograph of tissue.
Mechanism of thrombocytopenia -Answer- Immune Destruction of Platelets

39 y/o with systolic ejection murmur heard at the apex and does not radiate. Murmur is
louder when standing. ECG shows left ventricular hypertrophy. Most likely mutation?
- Answer- B-Myosin
A 56 year old F - 3 week history of increasing weakness of her arms and legs. 30 year smoking
history - PE shows pronounced weakness of the hip girdle muscles and lesser weakness of the
shoulder girdle. Chest X-ray shows hilar lung mass. ECG shows malfunction of the NMJ.
Impairment of which of the following is most likely cause of these findings? -Answer- Pre-
synaptic release of acetylcholine
A 26 y/o man - Peace Corps Volunteer - nausea after eating, profuse watery diarrhea -
Returned from 1-year assignment in Haiti. PE shows dry mucous membranes. Stool
culture grows oxidase-positive, curved, gram-negative bacillus. Which is the most
appropriate immediate treatment for this patient? -Answer- 0.9 Saline

A 52 y/o man - inability to achieve an erection 6 weeks after undergoing removal of a


tumor on the distal portion of the rectum - attached to posterior aspect of prostate.
Damage to which nerve? -Answer- Pelvic Splanchnic
Point and Shoot

Parasympathetic - Point
An asymptomatic 30-year-old F - PE shows nontender adnexal mass on left.
Ultrasonography confirms the presence of a single 6-cm fluid filled cyst. Most likely cause
of this ovarian cyst? -Answer- Anovulation when a follicle doesn't rupture (aka
anovulation) then it will form a cyst.

A 30 y/o man with HIV infection comes to office - PE shows no abnormalities - CD4 T
lymphocyte count is 158 and PE shows no abnormalities - Prescribed ritonavir and another
protease inhibitor - Why is the combination effective? -Answer- It increases plasma
concentration of other Pls also metabolized
56 y/o woman - PE shows no abnormalities - Pap smear show high grade squamous
intraepithelial lesions. Cause of the lesions in the patient? -Answer- Microbial
oncoprotein that interferes with tumor suppressor genes

47 y/o - MS patient - bilateral LE numbness - Proprioception and sensation to vibration are


diminished - Symptoms are caused by damage to which of the following? -Answer-
Fasciculus gracilis

A 31 y/o - 28 week gestation - CXR shows evidence of interstitial pneumonia.


Tetracycline is contraindicated because of what toxicity to the fetus? -Answer- Bone
Teeth discoloration and inhibition of bone growth - chelating agent + photosensitivity
A 61 y/o with severe abdominal pain - Polycythemia Vera - PE shows mild scleral icterus.
Abdominal exam shows tender, enlarged liver, enlarged spleen, ascites- DX? Answer- Budd-
Chiari Syndrome
widened hepatic venules - central lobular congestion
5-year-old boy, immunosuppressed because of chemotherapy for ALL, 2 day history of fever,
cough, shortness of breath, febrile (101.8 F), respirations 46/min, with cyanosis. Generalized
vesicular rash - Chest Xray shows nodular infiltration. Most likely cause by
which virus? -Answer- Varicella-zoster virus

43 y/o woman - multiple fractures and lacerations in an automobile collision - develops


tachypnea, tachycardia, and cyanosis - Despite intensive therapy - died in the hospital.
Which is most likely on exam of the lungs at autopsy? -Answer- Hyaline membranes
Trauma - ARDS
An otherwise healthy 82 y/o man - 6-month history of numbness and tingling in his toes.
Sensation to pinprick is decreased in lower extremities below the level of the knee. A
photomicrograph of a peripheral blood smear is shown - Which is most likely diagnosis?
-Answer- Posterior cord syndrome
Hyper segmented neutrophil - Marcocytic Anemia - B12 deficiency - Posterior Column

A research investing prevalence and incidence of ulcers in 100 residents in nursing care - At the
beginning of the study 10 residents had ulcers - During the first year, 5 additional residents
develop ulcers, and another 10 develop in the second year. National standards for nursing care
mandate that the prevelance be less than 20% and the new incidence should be less than 50
per 1000 patient years. Which is the most appropriate conclusion regarding the prevalence and
incidence if decubitus ulcers in this population at the end of the 2 year period? -Answer-
Prevalance Above the standard
Incidence Above the standard
100 residents
prevalence after 2 years is =10 at the beginning, +5 in the first year, +10 second year,
-3 that healed, for a total prevalence of 22 residents or 22/100=22 percent.
Incidence = 15 new cases / 90 residents over the 2 years (100
total residents - 10 that already had ulcers), 15 new ulcers per
180 patient⋅ years.
This would be 83.3 new ulcers per 1000 patient⋅ years if you extrapolated it out -basically
(1000/180) * 15 -- thus, incidence = above the standard.
Electric stimulation of the stellate ganglion - most likely result in an increase in which? -
Answer- Heart Rate

sympathetic stimulation of a GANGLION


A male newborn of has a myelomneninigocele - involves verterbral arches L2-l5 - PE shows
bilateral clubfeet. Which development process is affected? -Answer- Closure of the caudal
neuropore

A 45 y/o man - reclusive and weird ideas - I believe people can read minds. I carry these crystals
to help self-actualize. No delusions or hallucinations. Which personality disorder? -Answer-
Schizotypal
Which integral membrane proteins - results in anchored within the membrane? -Answer-
Extensive hydrophobic interactions between the amino acid chains of the protein and
the membrane phospholipid tails

25 y/o MCV < 80. Hemoglobin electrophoresis showed no abnormalities. Couples


offspring is at greatest risk for which? -Answer- a-thalassemia trait
13 year old - itchy red spots on face and neck - Went on a trip for swim meet. Several teammates
noted small bed bugs (photograph). The patient is at risk which infections? Answer-
Staphylococcus aureus
scratch bites - increase the risk of a staph skin infection (since staph aureus is the most
common skin infection pathogen)

48 year old - farmer - develops muscle spasms, trismus, and seizures - after a puncture wound
to his hand - Mechanism of toxin? -Answer- Blockade of postsynaptic inhibition of spinal motor
reflexes
Tetanospasmin is a neurotoxin that inhibits the release of γ-aminobutyric acid (GABA

Botulism -Answer- inhibits ACh release at the neuromuscular junction

floppy baby

A 62 y/o woman - BP 173/95 - HTN, Hyperlipidemia - Speech is dysarthric. Horizontal eye


movements are impaired bilaterally. Vertical movement intact. Exam shows quadriplegia. MRI
shows a lesion at which location? -Answer- Pons
Locked in syndrome - loss of horizontal eye movement and vertical movement intact
Quadriopelgia basilar artery runs right on top
of the pons

An 11-year-old - found him naked in bed with 4 y/o male cousin. Cousin was crying. 11year-old
school performance has declined. Most appropriate to evaluate for which of the following? -
Answer- Sexual Abuse
A couple - genetic counseling - two siblings with Cystic Fibrosis - 1/25 carrier frequency - Which
approximates recurrence risk that the individual identified by the arrow in the pedigree will
have a child with CF? -Answer- 1/150 carrier status to 2/3
Chance of affected individual = chance of father passing allele * chance of mother passing allele
= 2/3 * 1/25 *1/4 = 2/300 = 1/150
He has a 2/3 chance of being heterozygous (not 2/4 b/c we know he for sure doesn't have CF)
1/25 chance in the population
1/4 chance of a heterozygous couple having a child with CF
An investigator - adverse effects of proteasome inhibitor - bortzeomib. Bortzeomib
administered to group of patients with multiple myeloma - Which process is affected by this
drugs? -Answer- Presentation of antigens to CD8+ T Lymphocytes

An investigator - adverse effects of proteasome inhibitor - bortzeomib. Bortzeomib


administered to group of patients with multiple myeloma - Which process is affected by this
drugs? -Answer- Presentation of antigens to CD8+ T Lymphocytes
proteasome= degradation of malformed proteins
MHC class I is present on all nucleated cells
Failure to display MHC I - triggers a cellular immune response, leading to destruction of the cell.

A 45 y/o man with coronary artery disease and anemia - assess cardiac perfusion- Doppler
echocardiography shows blood flow in LAD is 30 ml/min. Arterial blood oxygen is .15 ml
(O2/mL) Which approximates myocardial oxygen supply in left ventricle in this patient (in mL
O2/min)? -Answer- 4.5
30* 0.15. Think about it, there is x flow with an oxygen concentration of y--so to find out the
delivery you just multiply them together.
64 year - found dead - photograph of the liver obtained on autopsy. Most likely DX? -
Answer- Metastatic Breast Cancer
Multiple lesions = metastasis
A 34 y/o woman - in ED 30 mins- after MVC - Sensation to pain is decreased over left lower
extremity- Which of the following labeled regions of spinal cord is damaged? Answer- Area
Labeled D spinothalamic tract is responsible for contralateral pain and temperature
sensation. Because our patient has lost sensation on the left, the lesion is in the right.

42 y/o - fatigue and abnormally heavy menses - No abdominal or pelvic pain. Exam of
endometrial biopsy shows glandular hyperplasia without atypia - Which describes pathogenesis
of endometrial disorder? -Answer- Binding of permeable ligand to nuclear transcription factor
endometrial hyperplasia - excess estrogen, a steroid hormone that translocate to the
nucleus and binds its transcription factor Sex + Thyroid - nuclear transcription ALL the rest
are peptide.

Which of present in integral membrane-associated proteins in absent from secreted proteins? -


Answer- Transmembrane region
Integral membrane proteins are found within the plasma membrane and span the whole length
across.
65 y/o - DVT - 10 days after hip replacement - receiving subcutaneous anticoagulant
therapy. Platelet count low. Cause? -Answer- Heparin-Induced Thrombocytopenia
IgG antibodies against platelet factor 4
Unlike DNA polymerase - Ecoli, RNA polymerase from bacterium lacks proofreading activity.
Which is most likely reason for lack of proofreading in RNA polymerase does not compromise
the survival of the species? -Answer- Mistake in transcription are not transmitted to progeny
A 52 y/o man - severe recurrent abdominal pain for 6 years - bulky, foul smelling stools
that float. 9 kg weight loss. 20 year history of alcoholism. Which lipids found in
excessive amounts? -Answer- Triglyceride
Alcoholic = pancreatic insufficiency

No Lipases - unable to break down triglycerides

15 y/o boy - nasal polyps and bronchospasms taking a drug. The drug most likely inhibit
which enzyme? -Answer- B Aspirin induced Asthma
5HPETE - peter is a leukotriene

Asprin works at Cyclic Endoperoxides - COX / Prostaglandins


55 y/o T2DM + fecal incontinence - Which additional symptoms in patient? -
AnswerImpotence fecal incontinence - damaged- pudendal n. (S2-S4). pelvic splanchnic
nerves - S2-S4
Point and Shoot
SLUDGE
Ecoli - digested into numerous small fragments with a restriction endonuclease - Wild
type is resistant to digestion with the restriction endonuclease. Mutation occurs in a
gene encoding which of the following? -Answer- Methylase
mutates DNA

65 y/o lesion at sit of peripheral scar on the lung. Site developed at site of pulmonary infarct
- Smoker for 20 years. A lobectomy is done. Most likely type of malignancy? -
Answer- Adenocarcinoma

Most likely cause of lung cancer


Peripheral lung cancer

more common in women.


A 35 y/o man - tremor and difficulty initiating body movements starting since starting
haloperidol for schizophrenia. PE shows masked facies and resting tremor. In addition to
decreasing dose of haloperidol - most appropriate for physician to prescribe which meds? -
Answer- Anticholinergic agent 1st gen antipsychotics extra pyramidal effects
Low dopamine and high acetylcholine - tx with anticholinergic

54 y/o - 1 week shoulder pain - PE shows tenderness of right deltoid - Abduction and passive
shoulder flexion produce pain. Pt has weakness with abduction and internal rotation. Likely
affected? -Answer- supraspinatus
Pain is worst with internal rotation of the shoulder -

consistent with the findings of the empty-can test, which indicates a supraspinatus injury
A med student - caring for 72 year old with lung mass. Biopsy shows carcinoma.
Resident instructs student not to inform patients of results. The morning patient asked "what
did my biopsy show". Most appropriate Response? -Answer- Let me get the resident physician
so we can go over the results

A 34 y/o woman - chronic Hep C - progressive fatigue. - AFAP in reference range - A liver biopsy
will likely show? -Answer- Infiltration of lymphocytes, plasma cells, and macrophages
A 68 y/o woman - 3 month history of nighttime urination and pain in lower back. She has
cervical cancer 15 years ago and underwent hysterectomy - Received radiation to
pelvis. Increased BUN and Creatine. Renal ultrasound shows bilateral hydronephrosis -
with distal narrowing. Explain findings? -Answer- Retroperitoneal fibrosis
Bilateral Hydronephrosis

direct consequence of the external beam radiation


Carcinoma - unlikely to be bilateral
65 y/o man - farmer - skin lesions on his face, arms, and upper trunk for the past few years.
Lesions to be actinic keratoses, and one is squamous cell carcinoma. Which is most likely
responsible for the development of squamous cell carcinoma? -Answer- Disruption of DNA
strands secondary to ionization effect
UVB light from sun
22 y/o - 12 weeks gestation comes to physician because of a 3 hour history of vaginal
bleeding. PE shows uterus consistent in size with a 16 week gestation. Ultrasonography
shows grapelike intrauterine structures and no fetus. Which of the following components
correlates with grapelike structures? -Answer- Hydropic villi with central cisterns

Which of the following sets of serum finding is most likely in a 50-year-old man with
recently diagnosed chronic renal failure? -Answer- Phosphate increased
Ca 2+ decreased
Parathyroid Increased

Not able to trash phosphate


Hypocalcemia - CKD
A study - examine effective of new antihypertensive drug - Subjects to be enrolled most have
moderate systemic hypertension, - approx. 100-200 tested. Drug is most likely in what stage? -
Answer- Phase 2
Working - involves up to several hundred affected patients.
A 39 y/o man - darkening skin and fatigue - T1DM -- Treated with insulin. PE shows
hepatomegaly and testicular atrophy. Which serum concentrations abnormal? -AnswerFerritin
Hemochromatosis, aka "bronze diabetes".
A 40 y/o - low grade B Cell lymphoma - TX with rituximab. Increased risk for which of
the following? -Answer- Bacterial pneumonia

B-cell deficiency can predispose to bacterial infections.


20 year old - ER - acute confusion - all night dance party - PE shows pale, cold extremities - An
ECG shows sinus tachycardia. Which substance ingested? -Answer-
MDMA

A 32-year-old - takes 40 alprazolam. Boyfriend threatened to leave. She drinks excessively and
uses cocaine. She has outbursts whether or not intoxicated. Which personality disorder? -
Answer- Borderline

16 month old boy - recurrent bacterial infections - since 8 months - Serum immunoglobulin
concentrations are uniformly decreased. Which findings is most likely in
this patient? -Answer- Absence of germinal centers in lymph node
X linked agammaglobulinemia
IG decreased in all classes

A 4 year-old - two bacterial UTI - Radiology shows marked dilation of left ureter and renal
pelvis, and minimal left-sided renal function. Left nephrectomy - Microscopic examination of
renal parenchyma - most likely to show? -Answer- Interstitial
Inflammation
Vesicoureteral Reflux from UTI
12 year old girl - Central Africa - 5 week abdominal pain. Fever, increased heart rate
and blood pressure. PE shows hepatosplenomegaly and abdominal ascites. A
photomicrograph of stool sample shows (schistosomiasis) Most appropriate
pharmcotherapy? -Answer- Praziquantel
treat a fluke
35-year-old itchy rash on his hands, waist, and feet 2 days. Wife and kids develop same
symptoms. 2 mm erythematous papules in fingers webs, wrist, umbilicus. Scraping show mites
and eggs. Most appropriate therapy? -Answer- Prescribe permethrin for the patient and his
family

45-year-old woman - ductal carcinoma of the breast - 8 weeks on tamoxifen - homozygous


presence of Cytochrome P450 - likelihood of sister same alleles? -Answer-
25%
homozygous + AR = 25%
Autosomal Recessive disorders - present as enzyme deficiencies.
Autosomal Dominant disorders - present as defects in structural genes
A 50 year old - Chronic gastritis is diagnosed with a marginal zone lymphoma of
Stomach. Which organism? -Answer- Helicobacter Pylori
A 30-year-old man brought to the ER after being stabbed - diagnose of rupture of aortawhat is
expected compensatory mechanism? -Answer- Increased serum angiotensin II concentration
A 56-year-old woman - headache, confusion, difficulty speaking + walking. Mental status shows
confusion and memory loss. Labs show - opening press 260, Protein raised, Glucose Low. EEG
abnormalities in right temporal lobe and periodic lateralized epileptiform discharges? -Answer-
Herpes Encephalitis

An obese 45-year-old woman - T2DM, hypertension, dyslipidemia, osteoarthritis - starts orlistat,


diet, and exercise program. During first month, increased risk of developing? - Answer- Diarrhea

A 38-year-old man - High Altitude Sickness - vacation ion Colorado Ski - Takes Carbonic
Anhydrase Inhibitor to adjust to altitude. Changes to (urinary) pH, bicarbonate, and volume? -
Answer- pH increased
HCO3 mEq/L - increased
Volume - increased

18-year-old - injured hand during football game - tenderness on palmar tip of ring, unable to
flex DIP of right ring finger, able to flex PIP and metacarpophalangeal. Pt injured fibers from
which nerve root? -Answer- C8

A 27 year old- swelling in left arm after spontaneous abortion in second trimester- axillary vein
thrombosis - Bleeding time normal, PT increased, PPT increased, Thrombin Time - Normal. Most
likely cause? -Answer- Antiphospholipid antibodies pregnancy + Sponataneous abortion +
Thrombosis + SLE In pregnancy
A 54 year-old woman - HTN and bilateral renal artery stenosis- taking NSAID- serum Cr
concentrations increase - cause by drug's ability to inhibit which? -Answer- Vasodilating
prostaglandins at the afferent arteriole
35 year old alcoholic man - admitted for vomiting, ascites, periumbilical vein, loss of
consciousness, flapping tremor of the hands. Pathogenesis is most likely directly related to
which? -Answer- Absorption of nitrogenous products from the gastrointestinal tract

Lactulose - MOA -Answer- reduces intestinal ammonia production


The Risk of hemorrhagic stroke from drug X is investigated by a questionnaire about drug use
in 702 patients with stroke and in 1376 control subjects contact by random-
digit. Study Design? -Answer- Case-Control Study
A 28-year-old man (seropositive for HIV) - numerous non-ulcerated purple nodules on the skin-
biopsy of the skin shows which? -Answer- Slit-like vascular spaces with plump spindle-shaped
stromal cells

kaposi sarcoma - spindle cells


A 22 y/o man - - infertility - Photomicrograph of biopsy of scrotal testes, Which genetic
abnormality? -Answer- 47, XXY
26 y/o - fever, chills, malaise, Cd4+ 700, HIV Viral Load 1 million copies. Serum HIV antibody test
are negative. Which explains patient's condition? -Answer- Acute retroviral infection
72-year-old multiple myeloma. Peripheral blood and T lymphocytes are isolated. B lymphocyte
DNA shows a 1.5kb band. T-lymphocytes J-region shows a 6-kb band. 6kb band signifies which -
Answer- unrearranged immunoglobulin gene

A screening Test developed and applied to 500 subjects with cancer and 500 patients without.
Sensitivity and specificity of each are plotted on a graph as a function of 1- specificity. Which
will most likely rule in cancer? -Answer- A - (1- specificity) = specificity highest
(SP - IN) - most specific highest specificity

A 56 y/o - brief ventricular tachycardia after MI. Arrythmia treated with loading dose of
lidocaine - followed by continuous infusion of the same drug. To maintain drug concentration
(Css), the most appropriate rate if infusion is determine by which of the following).
Cl = drug clearance and Vd = volume distribution. -Answer- CL X Css Maintenance Dose =
Css * CL * t) / F
t is elapsed time between doses (not relevant continuous infusion) F is
bioavailability (which is 100% or 1.0 here because it's given IV)
Loading Dose (Css * Vd) / F

Maintenance dose equation -Answer- Css * CL

Loading Dose -Answer- (Css * Vd) / F

A 35-year-old pain in right arm and tingling in fingers, weakness of extension + pronation,
decreased triceps reflex. Which nervous structures most likely involved Answer- C7 nerve root

82 year old - surgical repair for hip fracture - Healing is slow. Physician concerned about a DVT. A
drug that suppresses the action of which factors is most likely to decrease this patient's risk of
thrombosis? -Answer- Carboxylation of precursor proteins warfarin inhibits the synthesis of
factors II, VII, IX, X, C, and S by blocking reduction of oxidized vitamin K.
The enzyme Epoxide Reductase is inhibited by warfarin.

The reduced (active) form of vit. K is a cofactor for gamma-glutamyl carboxylase


Which of the following is required for synthesis of glucosamine from fructose -6phosphate? -
Answer- Glutamine
Hexoamine pathway - Fructose 6-phosphate to glucosamine requires (glutamine)
Physician receives $100 for referral of drug - should he tell the patient? -Answer- Yes;
the patient may wish to consider the money's influence on the physician's
recommendation

informed consent- includes disclosure, which includes the incentive that the company is
offering

49-year-old gradual onset of numbness and pain in left index finger, finger mottled blue
and pale -

Platelet count - 1,530,00. Bone marrow shows megakaryocytic hyperplasia. Which is


most likely diagnosis? -Answer- Bicarbonate

42 year old man on MPTP - frozen as ice, muscle rigidity, cell bodies effected? -
Answer- Dopaminergic neurons on the substantia nigra

b. TRAPS- Tremors, Rigidity, Akinesia, Postural Instability, Shuffling Gait


c. MPTP - toxic to substantia
A 62-year-old - excision for squamous carcinoma The physician explained the reason for the
procedure, the technique to be used, possible complication, name of physician performing
procedure, and which additional information? -Answer- Alternative treatments

A study to assess cardiac valvular abnormalities in ergot dopamine agonists. Initial study shows
a significantly greater score for patient on dopamine agonist. Research double the patient size
(increase sample size) and keep statistical significance P < .05. This will have which of following
effects on Type I and Type II error? -Answer- Type I Error No change
Type II Error decreased
49-year-old gradual onset of numbness and pain in left index finger, finger mottled blue
and pale -

Platelet count - 1,530,00. Bone marrow shows megakaryocytic hyperplasia. Which is


most likely diagnosis? -Answer- Essential Thrombocythemia
High Platelet count - increased megakaryocytes

Normal RBC and WBC

Erythromelalgia = condition characterized by episodes of pain, redness, and swelling in


various parts of the body, particularly the hands and feet
A 53-year-old evidence of adrenal failure - On CT both adrenal glands decreased in size. Most
likely diagnosis? -Answer- Autoimmune adrenalitis
Addisons disease - bilateral adrenal atrophy and destruction
85-year-old - femur fracture - begins TX with morphine controlled analgesic pump - 3
days later - respirations are 6/min. PE showed pinpoint pupils. Serum creatinine
increased. What explains clinical deterioration after morphine therapy? -Answer-
Morphine is metabolized to active metabolites
45 year old end stage renal failure - ER due to depression, bilateral crackles in chest, confusion,
Mental status exam shows confusion. Serum studies show a decrease in which? -Answer-
Bicarbonate

29 year old woman inflammatory disease involving joints, kidneys, serosal surfaces, anterior
chamber of the eye, choroid plexus. Which mechanism most likely causing
disorder? -Answer- immune complex-mediated cytotoxicity SLE -
Type III Hypersensitivity - Organ damage
A 57 y/o woman - lump in her breast. Carcinoma suspected. X-ray show multiple lesions in
vertebral column, and metastatic disease is suspected. Which of the following veins draining
the breast provides the most direct pathway for malignant cells to the vertebral column? -
Answer- Intercostal

38-year-old man - Down Syndrome - brought to physician by his nephew for a follow-up exam.
Pt lives with sister and her children. He is unkempt, malodorous, and appears anxious, 6.4
weight loss. When asked to interview on patient on his own, the nephew refuses to leave the
room. Most appropriate initial action? -Answer- contact adult protective services
2-year-old - fever, cough, 2 strep infections, multiple ear infections, 101.1, crackles in
lower lobe. PE shows no tonsillar tissue. Crackles are heard in right lobe of lung. Which
most likely immunodeficiency syndrome? -Answer- Bruton agammaglobulinemia

43-year-old eversion fracture - contraction of which muscle caused pain from fracture? -
Answer- Fibularis (peroneus) brevis -
50-year-old - severe chest pain - MI - ST elevation in anterior chest leads- troponin I increased-
Which reflects electrolyte content for myocardiocytes that produced the increase enzymes in
this patient? -Answer- Intracellular (Na+) Increased
Intracellular (K+) decreased
Intracellular Ca2+ Increased
A 45 y/o old - T2DM - low grade fever - catheter inserted below clavicle. Two blood
grow Nonhemolytic, catalase negative - gram positive cocci in pairs and chains -
organism? -Answer- Enterococcus Faecalis
42-year-old numbness and tingling vibration decreased in both legs. Sensation to vibration is
decreased. Deep tendon reflexes. Romberg positive, cause of findings.
Which is most likely cause? -Answer- Sensory neuropathy
. A 3 month girl - well child exam - able to life head, does not smile to human faces, has
not started to coo or gurgle with attention. What is the development status of (Motor,
Social and Verbal and Cognitive)? -Answer- Motor Normal
Social delayed

Verbal and Cognitive delayed

73 year old - obstructive sleep apnea -central obesity - CPAP will decrease which? -
Answer- Blood pressure

62-year-old, HTN nephropathy, chronic renal insufficiency, likely serum concentrations?


-Answer- Inorganic Phosphorus - Increased
Parathyroid Hormone - Increased
Calcitriol - Decreased

10-year-old renal transplant - from living donor - 3 months later there is tenderness at the
site of the graft - serum creatinine increased. Which findings confirm diagnosis of
transplant rejection? -Answer- Lymphocytes infiltrating tubular epithelium
A 52-year-old with stable angina - treated with med decrease preload and increase coronary
artery flow - greatest risk of adverse effect of this medication? -Answer-
Headache
25-year-old - 6 month irregular menstrual periods, acne, hair on lip, closed comodones,
serum shows? -Answer- Fasting Insulin increased
Testosterone Increased
LH increased

A study conducted to assess the prevalence of HTN in Caucasian vs African American.


Determining statistical significance between Drug X and Drug Y. Which statistical test i
most appropriate to determine the difference is statistical significant? -Answer- Chi
Square test

An 84-year-old woman- 2-month history of sore left hand - red, scaly plaque on the
dorsum of left middle finger- squamous cell carcinoma - invasion of basement
membrane facilitated by which of the following? -Answer- Downregulation of e-cadherin
52-year-old with lump in inguinal region, (present for 4 months). Which of the following
will be impaired if herniated gut compresses the nerve lies of external surface of

spermatic cord at the superficial inguinal ring? -Answer- Sensation from the anterior
surface of the scrotum
30 y/o M and 24 y/o F - preconceptional counseling - husband's brother is albino - autosomal
recessive that effects (1:40,000). Alleles in equilibrium, woman family is negative. A pedigree is
shown. Which of the following is the best estimate of child will be affected with oculocutaneous
albinism? -Answer- 1/600

The probability of the father being a carrier is 2/3 - known doesn't have the disease.

Then the probability of him passing it on to his kid is: 1/2 * 2/3 = 1/3

With the Hardy-Weinberg Principle, you can figure out the probability of the mother
being a carrier: q = sqrt(1/40,000) = 1/200
So, 2pq = 2 * 1/200 * 199/200, which approx is 1/100, and the probability of the child
getting this allele is 1/100 * 1/2 = 1/200 Thus:
1/200 * 1/3 = 1/600
A 72-year-old on ketorolac after abdominal operation (Pt has 2- year history of HTN and
T2DM), greatest risk drug induced adverse effects -Answer- Acute renal failure
NSAID - causes acute renal failure
A 79 y/o - brought to ED 30 minutes after he lost consciousness for 30 seconds. - ECG
shows third degree AV block - next step? -Answer- Insertion of transvenous pacemake
r

26-year-old develops hypotension and hemoglobinuria after receiving packed red blood cells-
Which of following is most likely to be involved in her condition? -Answer- Antibody,
complement C5-9

40-year-old woman - dry mouth, itching, burning eyes. PE shows inflammation of both
cornea and sclerae, enlarged salivary glands, and paucity if saliva. She is told she ha
risk of developing lymphoma. What is mode of inheritance of disorder -Answer-
Multifactorial

. A randomized control trial to assess the effect of diuretic therapy on systolic hypertension. A
total of 4736 patients - with systolic BP > 140 and diastolic pressure from 50mm HG to 85 mm
Hg. Which represents the absolute risk reduction for cerebral infarction in patients receiving
diruetic therapy:
Diuretic Therapy: 123 with CI /
2365 Total
Placebo: 194 with I / 2371 Total -Answer-
(194/2371-123/2365)
29-year-old MVA - MVC - Abdominal exam shows rebound
tenderness- Ct shows small
amount of intraperitoneal fluid - surgeon slides hand behind liver-
sudden massive
hemorrhage- blood pressure drops and becomes undectable within 1 minute- what lesion
involved with hemorrhage? -Answer- avulsion of hepatic veins from the inferior vena cava
massive hemorrhage - behind liver - IVC - connects to liver via hepatic veins
37-year-old - excision of 1cm, painless bump on back of neck - 7 days later incision is
erythematous and warm, no pain or drainage- what is the histologic appearance of the
incision? -Answer- Angiogenesis

Red in color, warm, no pain, no drainage

12-year-old boy is swimming in mountain cold stream - he is immersed in 60 F for 20


minutes. physiologic changes? -Answer- Central Blood Volume increased
ADH Decreased

Atrial Natural Peptide Increased


physiological response to hypothermia = vasoconstriction of peripheral vessels in an
effort to keep your core body temperature normal, and thus your organs functioning
properly.
Peripheral Vasoconstriction will increase CBV. Increased CBV will cause an increase in preload,
and thus cause an increase in ANP/BNP.
ANP/BNP has inhibitory effects on the Renin-Angiotensin-Aldosterone System, resulting in
decreased ADH

Following a stroke, a patient is hoarse and cannot detect pinprick or cold on left side of face
or right side of body- damage occurred where? -Answer- Lateral Medulla

54-year-old man- severed fingers slicing meat in deli 6 months ago- decreased sensation In
fingers- finger was successfully reattached. Exam shows muscle wasting. Which cells is most
likely blocking reinnervation of the muscles of this patient's finger? Answer- Schwann Cells
56-year-old man - radical prostatectomy - structures greatest risk of injury during
removal of prostate? -Answer- Pelvic Parasympathetic nerves
S2-S4

A previously health 28 year old - severe abdominal pain - nausea, vomiting, blood in
urine, hypoactive bowel, right flank pain - no rebound, no hernia, testes are normal
-
Likely Diagnosis? -Answer- Nephrolithiasis

Kidney Stones - hypoactive bowel sounds, flank pain, unilateral, distress writhing in pain
42-year-old woman heart burn - relief with ranitidine - The therapeutic effect of is most likely
mediated by receptor who activation increases the parietal cells of the stomach? -
Answer- cAMP
b. H2 receptors are Gs Receptors = increase cAMP
c. Gastrin released by G Cells
A 45 year old with Li-Fraumeni syndrome - participates in study of tumor suppressor gene
disorders. P53 mutation. Which is most likely result if mutation on the transcription of genes
that inhibit cell division and contain the consensus sequence (TATA) at the transcription site? -
Answer- Decreased binding of RNA Polymerase
Ten healthy adults given new oral drug - blood sample analyzed to determine
pharmacokinetics for the first time - Stage of trial? -Answer- Phase 1
small number of healthy - safety studies
Previously healthy 60-year-old F- MI of right ventricle - hepatomegaly- ascites - accumulation of
fluid in the peritoneal space. Which mechanism causes hepatomegaly and ascites? -Answer-
Increased central venous pressure
fluid backup - right heart failure - leads to Elevated pressures in the liver - leads to
hepatomegaly and free fluid accumulation

A 65 y/o - starts using topical fluoruracil for treatment of actinic keratosis. Fluorouracil is
effective because it inhibits which enzyme? -Answer- Thymidylate synthase
22 y/o pregnant woman decreased Alpha Fetoprotein and unconjugated steroids and
increased serum concentration of human chorionic gonadotropin. Which disorder? Answer-
Down Syndrome decreased AFP and increased serum concentration of HcG
A newborn has cyanosis, tachypnea, and retractions, arterial blood gas values while she
breathes room air: pH 7.04, PCO2 65, PO2 36. HCO3 15 - Acid Based Status? -
Answer- Respiratory acidosis and metabolic acidosis
Ph low - acidosis
Bicarb low - Acidosis

pCO2 high - Respiratory response

A 60-year-old man brought to ER - sudden onset 0f right chest pain - fractured rib -
multiple osteoblastic lesions in lumbar spine, pelvis, and ribs - what is most likely
diagnosis? -Answer- Prostate adenocarcinoma
A 3 day old - full term newborn - yellow skin - PE shows jaundice. Hemoglobin concentrations of
17 - Total bilirubin 10mg/dl and indirect component of 8 mg/dl. Jaundice resolves in 5 days
later. A deficiency in what causes the jaundice? -AnswerHepatic UDP-glucuronosyltransferase
activity

63-year-old man - progressive stiffness - difficulty initiating movements - He has a resting


tremor- which structure in the photograph undergoes degeneration in this condition? -Answer-
D - Substantia Nigra
Which of the following drug effects is the most common reason for non compliance with
cyclic antidepressant? -Answer- Anticholinergic

5-year-old - on a farm - diarrhea, cramping, abdominal pain, fever, drinks unpasteurized milk -
Am organism is isolated in stool. which is most likely organism? -Answer- Yersinia
enterocolitica
20-year-old - on crutches -tingling and numbness on dorsum of right hand - PE shows weak
extension of the right wrist and difficulty raising the right arm above the shoulder. Most likely
nerve injury? -Answer- C - Radial and Axillary nerve impacted

62 y/o F - discovers painless mass in left breast-menopause occurred at 50 - 2cm firm, non-
tender mass in upper quadrant, irregular microcalcifications, extremely radio dense. No weight
loss, axillary lymphadenopathy or discharge. Mammography shows
a radiodense mass with irregular margins. It contains clustered irregular microcalcifications - DX
-Answer- Carcinoma of the breast
73-y/o - difficulty urinating and frequent urination - drugs block following labeled sites -
Answer- Blocker of alpha 1

Tamsulosin- difficulty urinating


A 50 year old man - 3 day history of tonic-clonic seizures. One-month episodes of pins and
needles around the mouth and in the hands and feet. involuntary contraction of the muscles of
the hands and feet. PE shows diffuse hyperreflexia - An abnormality of which electrolyte
concentrations -Answer- Calcium
32 y/o - receiving zidovudine - mutation in zidovudine causing resistance. Which
mutation? -Answer- Reverse Transcriptase
52-year-old man - brought to ER - confused - he had a faulty space heater - Reddish tinged skin -
Carbon Monoxide poisoning, supplemented oxygen, best estimate of time for removal of CO-
carrying erythrocytes takes how long? -Answer- 4 Months
35-year-old enlarging nose, facial features, muscle weakness, increased hand and foot size. MRI
shows pituitary adenoma. Further studies show Gas subnit of G portiens lack
GTPase activity- tumor cells have increased activity of the following? -Answer- Adenylyl
cyclase
A 62-year-old man - evaluated for rectal bleeding - Xray of GI track shown. Which explains
feathery appearance of X compared with the portion of Y? -Answer- Greater mucosal surface
area
32-year-old 1 day history of chest pain radiates to her left arm, returned from a trip to
the amazon rain forest. Image of blood smear. What is vector? -Answer- Reduviid bug -
Amazon, Chest Pain, Dilated Cardiomegaly, Cardiac Arrythmia, treponema cruzi
22 year old woman 3 day history of nonproductive cough. 1-week history of fatigue,
SOB, and swelling of her legs. She delivered a male newborn uncomplicated vaginal
delivery. Bilateral basilar crackles are heard. There is 1+ edema in lower extremity:
diagnosis -Answer- Cardiomyopathy

Pregnancy can cause dilated cardiomyopathy causing systolic dysfunction.

Clostridium perfrigens A-toxin affects cells and facilitates gas gangrene by which of the
following mechanisms? -Answer- Splitting lecithin to phosphorylcholine and diglyceride
Physician unable to communicate bad news to a patient because the patient reminds him of his
older brother who intimidated him. Which of the following terms describes reaction? -Answer-
Countertransference

12-year-old boy. 6-hour history of severe diarrhea. He appears dehydraterd. Oral hydration
contains which of the following sets of ingredients? -Answer- Glucose and Sodium

86-year-old with three-week history of chronic nonproductive cough. He is tired in the


mornings because the cough interrupts his sleep. In addition to further evaluation to determine
the cough, which of the drugs is most appropriate treatment the patient while avoiding adverse
effects of constipation? -Answer- Dextromethorphan

74-year-old emphysema and lung cancer - wasted appearance -He is impoverished cereal and
toast diet- He had lost 13kg. He has muscle wasting, sunken eyes, loose skin, and edema. He
has a large mass in hilum of lung - edema - What causes the edema? -Answer- Negative
Nitrogen Balance
65-year-old man (T2DM and peripheral neuropathies) wasting of interosseous muscle of left land,
inability to abduct the fingers of the hand; and plantar flexion of the right foot is absent - Which
pairs of nerve are impaired? -Answer- Ulnar and Tibial

7-year-old - excessive urination- weight loss. PE shows poor skin tugor and fruity odor on
breath - HIGH BLOOD GLUCOSE 612 mg/dl - lab studies show increase in: -
Answer- Serum potassium concentration
DKA - decrease insulin - increased serum potassium

29-year-old - 5 week history of fatigue- 4 day history of heart palpitations. She has Primary
Hypothyroid - and treated with Triidothyronine. She says she has a double dose because of
fatigue, fine tremor, deep tendon reflexes are brisk: Serum Concentrations- TSH, Free
Thryoxine, Free Triiodfothryronine? -Answer- TSH Decreased
Free Thyroxine Decreased
Free Triiodothyronine Increased

An unimmunized 1-year old boy is admitted to hospital fever, irritability, stiff neck, cultures of
the nasopharynx and cerebrospinal fluid grow gram negative coccobacilli pili removed from
nasopharynx but absence from CSF - Which process shuts off expression of pili? -Answer- Phase
variation
50-year-old - smoker, increase SOB, chronic cough productive, thick sputum and wheezing.
Anterior and Posterior Chest Wall Increased. Diminished breath sounds and scattered rhonchi.
Which lab abnormality? -Answer- Increased blood HCO3- emphysema leads to CO2 trapping
leading to increase paCO2 in the blood, which gives you a respiratory acidosis

18-month old boy - admitted with tonic clonic seizure, - He has mild development delays -
25th% on growth charts, six hypopigmented macules, flesh colored lesion on back.
An MRI of brain shows multiple lesions in cerebral cortex. Which DX? -Answer- Tuberous
sclerosis complex

A 66-year-old - 2 month erectile dysfunction. He has fatigue and difficulty sleeping. 3 months
ago cerebral infarction 2 month prior - PE shows no abnormalities - Which additional pairs of
findings (Libido and Nocturnal erection?) -Answer- Libido Decreased Nocturnal Erection Normal

A 66-year-old - 2 month erectile dysfunction. He has fatigue and difficulty sleeping. 3 months
ago cerebral infarction 2 month prior - PE shows no abnormalities - Which additional pairs of
findings (Libido and Nocturnal erection?) -Answer- Libido Decreased Nocturnal Erection Normal
31-year-old - concerns about hair loss. Family History of alopecia. If pharma treatment
is elected - use a drug to block synthesis of which of the following: -Answer-
Dihydrotestosterone

56-year-old recovering from pneumonia - No X-ray abnormalities. Which allowed


resolution to occur? -Answer- Maintenance of basement membrane integrity
65 y/o osteoarthritis, pain radiating down distal anterior thigh, knee, medial leg, and foot. Bony
outgrowth of vertebrae compressing one of the spinal nerves. Compression of nerve root in
which of the intervertebral foramina most likely cause her symptoms? - Answer- L-3 to 4
. 17 y/o boy - 1 week of fever, fatigue, sore throat, red oropharynx, bilateral cervical
lymphadenopathy - heterophil antibody positive - 12 month after complete recovery,
which cells still contain virus that caused his sore throat? -Answer- B Lymphocyte
10-year-old girl lacerated her chin. Painful subcutaneous nodule on chin. Sutured Ten weeks
ago later image of biopsy- which describes the pathology features? -Answer- Granuloma
Pts with mucolipidosis II (I Cell Disease) lack of phosphotransferase required to form
mannose 6-phosphate that attach to destined enzymes - mannose-6-phosphate is
normally added to proteins in the cis-Golgi. Which occurs to lysosomal enzymes in
these patients? -Answer- Secreted from the cells
A case control assess exposure to environmental tobacco smoke and sinusitis. Exposed to
Smoke Cases and Not Exposed Cases Shown - What is the prevalence of sinusitis in the general
population? -Answer- Cannot be determined from the data given

A case control assess exposure to environmental tobacco smoke and sinusitis. Exposed to
Smoke Cases and Not Exposed Cases Shown - What is prevalence of sinusitis in the general
population? -Answer- Cannot be determined from the data given
Study on symptomatic proximal DVT, patients randomly assigns (2 groups) to receive
either dalteparin subcutaneous and then oral warfarin, or dalteparin alone for 6 months-
Probability of recurrent thromboembolism 6 months is 10% in daltepain and 19% in
warfarin group. Which is the study design? -Answer- Open-label clinical trial

a. Clinical trial - compares therapeutic benefits of 2+ treatments (warfarin vs. dalteparin)


b. Open-label - both the health providers and the pt are aware of the drug being given

blood pressure, systolic pulsation of the liver. Liver is percussed to 12cm. Which is the
diagnosis? -Answer- tricuspid insufficiency

78 year old severe back pain, worse at night, serum ALK phosphatase, increased lytic
lesions in vertebral columns. Diagnosis? -Answer- Metastatic Carcinoma
31-year-old woman - 2nd trimester of pregnancy- vagina bleeding. BP 140/95 HTN- Ultrasound
shows conceptus with patches of tissue - no apparent embryo. Biopsy shows 69 XXY. The
presence of what would indicate that an embryo had formed within the fetus? -Answer-
Hematopoietic stem cells

24-year-old woman, post pregnancy hyperthyroid, nervousness and tremor - 5lb weight loss -
thyroid 2X as large. Free Thyroxine 2.4, TSH low, thyroid iodine uptake 1%- which likely
explanation for symptoms? -Answer- Release of stored thyroid hormone from a thyroid gland
infiltrated by lymphocytes

A 27-year-old man - smoker- wants to quit, but not sure ready for change.
Contemplation. Which is the next step for the physician? -Answer- Ask the patient to identify
the pros and cons of smoking cessation
A 36 year-old woman- SLE - Treatment with prednisone - drug binds to receptor in
following site? -Answer- Cytosol with translocation into the nucleus
27-year-old man construction worker fell from a ladder - wood splinter grazed inferior pole of
left kidney and splinter now in structure immediately anterior? What structure? Answer-
Splenic Flexure

A 2 day newborn develops bilious emesis, abdominal tenderness, hemodynamic deterioration,


Upper GI- X-ray - shows ligament of Treitz on right of abdomen and duodenal obstruction. Pt
greatest risk for occlusion of following arteries? -AnswerSuperior mesenteric
A 60 year-old- man - SOB+ worked in shipyard. X-ray show reticulonodular pulmonary infiltrates
- Microscopic exam shows elongated structures (Image Ferruginous body) - The fibrosis was
most likely initiated by interaction of these structures with which of the following? -Answer-
Alveolar Macrophage
Previously heathy 45-year-old woman fever, confusion, decreased urine for past 2 days.
Platelet count low. Peripheral blood smear. Which is most like cause? -Answer-
Thrombotic Thrombocytopenia purpura
73-year-old woman dies 7 years after onset of progressive memory loss. Severe intellectual
deterioration. A coronal section of the brain at autopsy (Shown). Numerous neuritic plaque and
neurofibrillary tangles. Cause of ventricular enlargement? -AnswerHydrocephalus ex Vacuo

76-year-old - 1 month history of a pulsatile abdominal mass is diagnosed with an abdominal


aortic aneurysm. CT scan shows horseshoe kidney. Surgical repair is most likely to be further
complicated by presence of? -Answer- Anomalous origins of multiple renal arteries to each
kidney
Healthy 25-year-old - eats a meal. serum insulin increases after a meal - mechanism of
pancreatic circulation of insulin? -Answer- Fusion of an intracellular vesicle with the
plasma membrane

26 year old new mother - constant worry - excessive concern for safety and people
breaking into her house- most appropriate pharmacotherapy -Answer- Sertraline
48-year-old - vomiting blood- severe alcoholic pancreatitis - pseudocyst of pancreatic tail
requires surgical drainage. Exam shows splenomegaly - which is vessels are most likely
thrombosed? -Answer- Splenic Vein

52-year-old -metastatic oat cell carcinoma- receives hospice- severe bone pain—tx narcotic
started- Which treatment is most appropriate? -Answer- Ensure that the patient receives
enough medication to control his pain

38-year-old - chronic headaches- comes to physician for a follow-up exam - She has 3 sons kids
sickle cell disease- She tells physician she used to feel angry and abandoned
- started kickboxing- feels happier and more relaxed? Coping mechanism? -
AnswerDisplacement
6-year-old emigrated from Russia. Unstable gait and incoordination. He has frequent pale, bulky
stool. Neuro exam shows ataxia, absence of DTR, fatty stools, loss of proprioception. Stool
analysis shows an increased fat concentration. Which vitamin deficient? -Answer- Vitamin E
45 y/o p- HIV positive -1-week history of headaches and blurred vision- a lumbar puncture is
done- CSF - positive for fungal yeast - Most appropriate treatment is a drug with which of the
following mechanisms? -Answer- Disruption of the pathogen cell membrane

60-year-old woman - 1 year of involuntary rhythmic jerking, increase jerk reflex, Babinski sign
present. An MRI shows a 2 -cm, round, enhancing lesion with interhemispheric fissure in the
region of the central sulcus. Most likely DX? -AnswerMeningioma

New Virus - causes encephalitis - The viral particle contains and RNA-dependent DNA polymerase
- which of the following types of RNA most likely to be genome of the virus? -Answer- Single-
stranded positive-sense

5-year-old - pain in right eye at night for the past week- Exam shows strabismus and tenderness
in the eye. Examination of the retina shows presence of a mass. Physician explains that boy is
unlikely to develop other neoplasms- mutation started in which of following? -Answer- Retinal
Cells

62-year-old man with dyslipidemia-6 hours of severe muscle aches, generalized weakness, and
brown urine - he recently started on lipid lowering medication, increased serum creatinine,
blood on urinalysis -what medication was added? -Answer- Gemfibrozil

5-year-old boy - red cheeks+ rash over arms and lefs. 1 week history of fever - PE shows red,
lacy rash over the upper extremity and lower extremity. Low hemoglobin.
Cause of anemia? -Answer- Interruption of erythrocyte production
Parvovirus infection, which infects erythroid precursors

57-year-old woman - progressive weakness in arms and legs. She says states that weakness is
most apparent when she tries to lift dishes and when she walks down a flight of stairs. Muscles
strength improves on repetitive manual motor testing.

Sensation intact in all extremities. Which of the following mechanisms explains weakness? -
Answer- Autoimmune downregulation of Ca2+ channels of the presynaptic terminal
10-year-old boy bruises easily since swallowing grandfather pill. Aspirin and Dicumarol
are found. How do we confirm patient took dicumarol rather than aspirin? -Answer-
Prolonged prothrombin time

Dicumarol - Depletes vitamin K stores


factors 2, 7, 9, & 10
39-year-old -woman with Rheumatoid arthritis - TX with multiple medications - including
prednisone and methotrexate - NO progress. The next step pharmacotherapy includes drugs
that block the effects of? -Answer- Tumor necrosis factor-a
17-year-old -taking 2 aspirin tablets for tension headache- 30 minutes after she has difficulty
breathing - what meds in future should she use to treat headaches? -AnswerAcetaminophen
53-year-old woman - fibromuscular dysplasia involving renal artery-develops HTN
evaluated for revascularization procedure. Which of the following histologic findings most
likely? -Answer- Tubular atrophy
81-year-old - ongoing management of HTN for 20 years, high blood pressure (200/110),
To reconfirm the BP, physician puts index finger over the radial artery. At this point the
radial artery becomes non-pulsatile but remains easily palpable even as cuff is further
inflated. Which explains phenomenon? -Answer- Atherosclerosis

An 1814-g (4-lb) male - delivered in hospital at 39 weeks - Develops respiratory distress


and dies 8 hours later. Congenital abnormality- congenital absence of pancreatic islet
cell. Most likely cause of this congenital abnormality is s defect in the initial
differentiation from which of the following precursors? -Answer- Endodermal cells
65-year-old ER 1 day of SOBs - He has a history of hypertension, T2DM, 2/6 systolic murmur.
Pulse Ox is 90% - Cardiac exam shows S3 - The point of maxillary impulse in the axillary line.
Which finding is most likely on pulmonary auscultation? -Answer- Crackles
S3--> dilated cardiomyopathy (eccentric)
8-year-old boy - parents concerned about his weight - BMI 25- 75th height and 95th
percentile weight + BMI - most likely explanation for obesity? -Answer- Calorie
consumption that excess energy expenditure
. 5-year-old with mental retardation- grossly obese and facial features of Prader Willi -
Karotyping and fluorescent in situ hybridization do not show deletion in the usual site of
chromosome 15. Which of the following is most likely to confirms Prader Willi? -
Answermaternal origin of both chromosomes 15

Prader Willi = Paternal deletion (partial or full). Noted for imprinting.

25% of cases are due to maternal uniparental disomy


Nicotinic Acid acts at which of the following labeled sites in the diagram? -Answer- C.
Cholesterol - VLDL
In pt's with Adenosine deaminase deficiency - 50-100 fold increase in dATP
concentrations in T lymphocytes- the increased dATP inhibits which enzyme
compromising DNA synthesis? -Answer- Ribonucleotide reductase
RNR converts ribonucleotides to deoxyribonucleotides,- then - dATP (a
deoxyribonucleotide) would inhibit the enzyme
37-year-old - 1-year HTN -high BP despite TX (190/135) - Fundoscopic exam shows multiple
flame hemorrhages - severe arteriolar narrowing. What disorder of blood vessels is the cause? -
Answer- Hyperplastic arteriolosclerosis

A 4 month old boy small size, difficulty feeding from a bottle, recently adopted from developing
country (unknown medical history), (Photograph of baby cleft lip) Most likely cause of facial
finding (INVOLVING LIP) is a failure of normal fusion of the following pairs of structures? -
Answer- maxillary and medial nasal prominence
38-year-old - admitted after multiple injuries in farm implement accident- The day after
admission. what causes increase in his plasma fibrinogen? -Answer- Acute Phase
Response
Acute phase reactants up-regulated during systemic manifestations: "More FFiSH in the C"
Ferritin, Fibrinogen, Serum amyloid A, Hepcidin, C-reactive protein.

A 13-year-old with teased for "having girl like breasts". Tanner Stage 3 - no health issues-
average height and weight - PE shows soft mound of tissue bilaterally beneath the areola -
Most appropriate next step? -Answer- reassurance

A 48 -year-old - begins Furosemide therapy for pedal edema - associated with biventricular
failure and HTN- 5 days later decreases potassium - what actions should be added to the
regime? -Answer- Decrease the luminal permeability to Na+ in the collecting duct
Amiloride and Triamatrene block EnAC channels on luminal membrane- K Sparing Keep your
SEAT
Pt early stage of hemorrhagic shock - most likely to have what symptom? -Answer- A
weak pulse due to decreased stroke volume
(hypovolemic shock)-

early in shock not enough time for RAAS activation

During first week of life - Newborn has vomiting, severe dehydration, hyponatremia, salt
wasting. Serum 17-hydroxyprogesterone increased. Most likely cause is a defect in
which disorder? -Answer- 21- Hydroxylase
Congenital Adrenal Hyperplasia

28-year old - recurrent pancreatitis with familial hypertriglyceridemia. Administration of a


drug with which of the following effects? -Answer- Decreasing VLDL
. A newborn at 26-weeks gestation- Pt has respiratory Distress and requires intubation and
maximal oxygen support. A chext X-ray shows ground glass appearance in both lungs. Deficient
synthesis of which is most likely cause of respiratory difficulty in the newborn? -Answer-
Dipalmitoyl Lecithin
Dipalmitoyl Lecithin = Lung Surfactant = dipalitoylphosphstidylcholine

Screening tests for fetal lung maturity (Lecthin/Sphingomyelin) > 2 healthy, < 1.5 NRDS

A 6-month-old - development delay- difficulty feeding and recurrent vomiting. He has


hepatosplenomegaly and lymphadenopathy. Examination of a biopsy of bone marrow shows
foam cells. A defect in metabolic pathways is most likely to cause this diorder? -
Answer- Sphingomyelin degradation
Niemann Picks Disease- mental retardation, foam cells, cherry red spots in macula

A 35 year-old-man - (diving accident). Evaluation shows transection of spinal cord superior to


the level of the sympathetic nervous outflow - Which responses will occurs if he gets a systemic
infection? -Answer- Alternation of the thermostatic set point Hypothalamus controls
temperature
A 17-year-old boy (wrestler) - collapsed while competing in wrestling competition. PE shows
pallor and diaphoresis. He is losing weight to compete in his weight class. He is using a long
steam baths and laxatives - Lab show? -Answer- Hypokalemia

A 1-year-old boy impaired respiratory burst of phagocytes - Family history of immunodeficiency


- Patient has greatest risk of infection by which organism? -Answer-
Staphylococcus aureus
Chronic Granulomatous Disease -NADPH deficiency phagocytes need to produce hydrogen
peroxide to undergo oxidative burst-
CGD patients can't kill catalase + organisms (Staph Aureus, Aspergillus) - catalase
enzymes neutralize superoxide

An 11-year-old CC: pain in right thigh for 2 weeks. PE shows warmth tenderness, ill-
defined mass. , solitary mid-diaphyseal ----- osteolytic lesions (small, uniform,

hyperchromatic cells: high nuclear: cytoplasmic ratio). - DX? -Answer- Ewing Sarcoma
A 45-year-old man- alcoholic- chronic pancreatitis + diarrhea - PE shows (decreased
fecal elastase) - what meds for diarrhea? -Answer- Pancrelipase
Pancrelipase = pancreatic enzymes
A 30-year-old man - 2 day history of blood in urine, dull flank pain, voiding large urine- BP is
150/100. Pt father died of unknown kidney disease. CT showed cystic masses in each kidney -
surgical specimen shown (polycystic kidney disease)- Underlying defect involves which
molecule? -Answer- Polycystin
A 42 year-old-woman - tonic clonic seizures- She tells physician that she smelled a foul
odor before seizure- where did seizure arise? -Answer- Temporal Lobe

A 9-year-old boy with signs of Cystic Fibrosis (cough, recurrent upper respiratory tract
and sinus infection)- PE shows clubbing of fingers - increased sweat chloride and

sodium. A defect in patient's bronchial epithelium is most likley causing? -Answer-


Protein Structure

mis-folding --> less CF receptors on cell surface --> phenotypic CF.

During an experiment, 22 y/o receives an infusion of Histamine in Left Brachial Artery -


what changes? -Answer- Arteriolar Resistance decreased
Capillary Hydrostatic Pressure Increased
Capillary Filtration Rate Increased

Histamine causes arteriolar dilation, - increased blood flow into capillaries - increased
capillary permeability in venules -increased capillary filtration

67-year- old woman non-productive cough, crackles on lung bases, NO fever - CT scan
shows bibasilar reticulonodular markings with interlobar septal thickening; Pulmonary
function shows Fev1 (normal), FVC 48% of predicted - DLCO - low- Which is likely DX?
-Answer- Idiopathic pulmonary fibrosis
Two patients, 54 year old man and 76 year old man - receive vancomycin - both are same
weight - GFR > in Patient X than Y. Which parameter is the same in patient X and Y? -Answer-
Loading Dose independent of drug clearance- unchanged in liver or renal dx

A 35-year-old -weakness and fatigue - When 25, she was treated for Hodgkin Disease with
Chemo Drugs- 10 years ago- no evidence of lymphadenopathy- A peripheral blood smear shows
numerous blasts. Which most likely explanation of findings? -Answer- Acute Myelocytic
leukemia
Chemo agents - increase risk for AML
ALL = 0-14
AML= 15-39; 40-59 - Characteristics of Numerous Blasts
CML = 40-59

CLL = 60+

Aurer rods
A 15-year-old - redness from, sunbathing - use sunblock- no blisters - condition? -
Answer- First-degree burn

A 15-year-old - redness from, sunbathing - use sunblock- no blisters - condition? -


Answer- First-degree burn

49-year-old severe hip pain - alcoholic - increasing severe pain in his left hip - He has
pain with weight bearing- Coronal T1 is shown- Most likely cause of patients pain -
Answer- Avascular Necrosis

CASTS Bend LEGS = Corticosteroids + Alchoholism+ Sickle Cell + Trauma + SLE


The Bends + Legg Calves

A 48 year- old renal artery stenosis- undergoes stent placement- femoral access stent -
After entrance into aorta, guidewire should be advanced superiorly just beyond which
structure to approach right renal artery -Answer- Testicular Artery
A 32-year old woman -begins to hyperventilates + blurry vision+ numbness around mouth -
after friend dies - which of the following is most likely decreased in this woman?
-Answer- Cerebral blood flow
vasovagal sympathy - hyperventilation decreased pH, PaCO2,
Bicarb- central chemoreceptors respond to low PaO2 by vasoconstricting cerebral blood flow
A 12-yeasr-old girl - unable to walk for 5 days- her parents are anxious - but she seems
unconcerned about her symptoms. DTR are equally bilaterally. DX? -Answer-
Conversion disorder
60-year-old - SOB and cough - 2 week history of of progressive swelling of her face - Productive
cough with blood tinged sputum -Smoker for 45 years - 2.3 kg weight loss hyponatremia- Which
is cause of facial edema? -Answer- Lung cancer
42-year-old fatigue, excessive thirst, frequent urination with large volume - He is 5'10 -
weighs 61kg - BMI 19 - high ADH- increase serum OsM, decreased Urine OsM. In
addition to low-sodium diet, most appropriate drugs for this patient? -Answer-
Hydrochlorothiazide
Parenteral cholera vaccine (killed) whole bacterial cells have limited efficacy- which is the
primary reason for his failure? -Answer- Inability of the vaccine to elicit secretory antibody at
the epithelial surface
During a clinical study, 15 patients with renal allografts volunteer to undergo treatment with
DrugX - The drug is converted to a purine antagonist that interferes with synthesis
of nucleic acids and toxic to dividing cells. Drug X is most likely? -Answer-
Azathioprine
25-year-old -admitted with septic shock - splenectomy from MVA - Coarse tubular breath
sounds CXR shows bilateral pulmonary opacities, dense consolidation, and right lung effusin -
causal organism -Answer- Streptococcus pneumoniae
Splenectomy - vulnerable against encapsulated organisms
Pt early stage of hemorrhagic shock - most likely to have what symptom? -Answer- A
weak pulse due to decreased stroke volume
A 48 -year-old - begins Furosemide therapy for pedal edema - associated with biventricular
failure and HTN- 5 days later decreases potassium - what actions should be added to the
regime? -Answer- Decrease the luminal permeability to Na+ in the collecting duct
A 13-year-old with teased for "having girl like breasts". Tanner Stage 3 - no health issues-
average height and weight - PE shows soft mound of tissue bilaterally beneath the areola -
Most appropriate next step? -Answer- reassurance

23-year-old - multiple, red, papulovesicular lesions following poison IVY - Which is mechanism
of these lesion?? -Answer- Processing of the antigen by Langerhans cells, leading to activation
of CD4+ T cells

A 48-year-old - fatigue, SOB on exertion, decreased exercise tolerance - X-ray shows decreased
cardiac enlargement and prominent pulmonary vasculature- glucose intolerance -liver
dysfunction- An older brother died at age 52 years with similar symptoms. Which is most
important screening test? -Answer- Transferrin saturation and serum ferritin measurements
triad - cirrhosis, diabetes, skin pigment
glucose intolerance (post pharm) + Familial + Presenting after 40- Total Body Iron over 20g
Bronze Diabetes

66 y/o - SOB+, swelling of the legs. HE MI 6 months ago. Exam shows distended jugular veins,
bilateral crackles, pitting edema in the lower extremities. Serum sodium concentrations 129. -
cause of hyponatremia? -Answer- Dilution of serum sodium due to ADH (vasopressin) secretion
Heart Failure - increased vasopressin secretion

A 2-month old with sore throat because of a 7 day history of hoarseness- mother has AIDS and
used cocaine through pregnancy - PE shows no abnormalities. Laryngoscope shows a nodule on
the left vocal cord. Which viruses is most likely cause of nodule? - Answer- Human
papillomavirus

A 6-year-old- itchy rash in his armpits, both hands, feet, and groin. The itching is most intense at
night. Vitals are within normal limits. Exam of skin shows multiple erythematous papules,
burrows and many excoriated (photograph). Which most helpful in establising diagnosis? -
Answer- Does anyone else in the family have an itchy rash like this

A 13-year-old - routine child exam - Neither mother or child have health concerns - The
physicians asks questions on sexual activity - Which of the following actions by the physician is
most appropriate? -Answer- Ask the mother to leave the room before asking the patient any
questions
A 28-year - old woman - 3 week double vision - blurry vision in left eye - right eye does
not adduct beyond midline - right horizontal gaze - normal - pt has lesion in which
structure labeled structure? -Answer- Area labeled C
Intranuclear ophthalmalgia - lesion of ipsilateral MLF
10-month-old - pale and unresponsive - mother gave her 3 does of loperamide - for diarrhea
- marked distention - If medication is required, most appropriate therapy? -
Answer- Naloxone

A 28-year-old man - 6-hour history of severe left midabdominal pain - abdominal mass
in mid- abdomen - CT scan of abdomen shown - Abnormalities indicted by arrows -
intussusception of which of the following portions of intestinal tract is most likely cause
of patients pain? -Answer- Jejunum

27 year old - ER after injury to ankle while rocking climbing - swelling on medial lateral
angle of left ankle - X-ray of left ankle is shown. The patient injury most likely occurred
when his left foot was forcibly moved into the extreme of which position? -Answer-
Eversion

A 17-year-old - 16 weeks gestations has a mucopurulent vaginal discharge. Culture of


the discharge grows Chlamydia - most appropriate pharmacotherapy? -Answer-
Azithromycin

. 13-year-old girl brought to ER after sudden onset of SOB- soft tissue swelling- Some episodes
accompanied by acute abdominal pain with diarrhea - marked facial edema-
the most likely condition- is a deficiency of which substances? -Answer- C1 inhibitor

A 57-yearold woman - burning shock like pain on left of face - pain last 5-10 seconds-
No neuro abnormalities - most likely diagnosis? -Answer- Trigeminal Neuralgia
facial pain, often triggered by chewing, speaking, or brushing the teeth.
An Rh negative woman, gravida 3, para 2 - increasing bilirubin concentration in amniotic fluid-
umbilical cord hematocrit 6%- transfusion of the following types of packed red cells to the fetus
in utero should correct the anemia? -Answer- O, Rh-negative
16 year old has pain and tingling - while backpacking- symptoms resolve after she
removes the back pack- what are the abnormalities -Answer- Cervical Rib
16-year-old - excessive dieting, constant studying, and social withdrawl. BMI 17. PE shows
skin and fine hair growth - Labs shows metabolic alkalosis - The patient is most likely abusing
which? -Answer- Diuretics Diuretics: [K dec] [Cl dec] [HCO3 inc] [pH inc]
Vomiting: [K dec] [Cl dec] [HCO3 inc] [pH inc]
Laxatives: [K dec] [Cl inc or dec] [HCO3 dec or inc] [pH dec or inc]

Diuretics: -Answer- [K dec] [Cl dec] [HCO3 inc] [pH inc]

Vomiting: -Answer- [K dec] [Cl dec] [HCO3 inc] [pH inc]

Laxatives: -Answer- [K dec] [Cl inc or dec] [HCO3 dec or inc] [pH dec or inc]

54-year-old man has abdominal aneurysm. The estimated cross-sectional areas is 2 cm^2 and
the mean velocity if blood flow is 20cm/sec. Which represents flow rate (L/min
-Answer- 2.4
2cm^2 * 20 cm/sec * 60sec/1 min * L/min
56 y/o - brought to ER - Chest Pain mowing the lawn - Hypotension, PE shows cool,
clammy, diaphoretic skin. 2 hours after admission, pulmonary catherization shows
pulmonary artery pressure 40/25. Left Atrial Pressure 25 g (pulmonary emboli).

Compared with a healthy person- sets of pulmonary pressure changes? -Answer-


Hydrostatic Pressure High,

Capillary hydrostatic pressure high


Interstitial Oncotic pressure Low

cardiogenic shock, the extra blood increases capillary hydrostatic pressure, driving fluid
into the interstitial space.
67 y/o - brother has history of colon cancer. Physician recommends colonoscopy, but the
patient says she would prefer stool culture instead. Physician explains that testing the stool for
occult blood is not appropriate. Physician concerned about? -Answer- Low Sensitivity

A 62 y/o - 1 week of muscle cramps - trouble opening jars and using silverware. PE shows
muscle weakness and exaggerated deep tendon reflexes. What is appropriate systemic
treatment for muscle cramps and spasms? -Answer- agonist at GABAb Receptors

41 y/o schedule for dialysis catheters. Surgeon asks the resident to get the consent. The
resident has never seen this procedure performed and has read little about it. Which of the
following actions by the resident is most appropriate? -Answer- Ask attending to obtain
informed consent
A 2 year old - undergoes resection of right kidney - chronic infection secondary to influx. The
left kidney is grossly normal. Which structural adaptations will eventually occur in left kidney? -
Answer- increase in glomerular size Congenital solitary functioning kidney compensatory
hypertrophy of contralateral kidney

A 54 y/o woman - terminal metastatic pancreatic cancer - She asks physician to prescribe
sufficient meds to commit suicide due to unbearable pain. Physicians refuses
- but will do anything to help pain. Physician decision is most consistent with which? Answer-
Nonmaleficence

22 y/o woman - recently emigrated from rural Mexico - 20 minutes after generalized tonic-
clonic seizures. 6 week history of headaches, nausea, vomiting. Funduscopic exam shows
bilateral papilledema. An MRI of the brain shows enlargement of right ventricle consistent with
Cysticercosis. Which is mostly likely mechanism of
hyderocephalus? -Answer- Obstruction of right interventricular foramen (foramen of
Monroe)
22 y/o woman - recently emigrated from rural Mexico - 20 minutes after generalized tonic-
clonic seizures. 6 week history of headaches, nausea, vomiting. Funduscopic exam shows
bilateral papilledema. An MRI of the brain shows enlargement of right ventricle consistent with
Cysticercosis. Which is mostly likely mechanism of hyderocephalus? -Answer- Obstruction of
right interventricular foramen (foramen of Monro)

59 y/o - develops fatigue and decreased appetite 3 months after receive mechanical aortic
valve replacement. PE shows scleral icterus and elevated JVP. Lab studies show total bilirubin 4
elevated and direct bilirubin 1. Which is the cause of the jaundice in this patient? -Answer-
Hemolysis
Prosthetic heart valves / aortic stenosis- hemolytic anemia secondary to mechanical destruction
of RBCs.

71 y/o man - 6 month history of calf pain and discomfort when he walks. He has HTN, T2DM,
weak dorsalis pedis, and ankle brachial index. The physician knows a drug that would improve
symptoms by altering platelet function and providing direct arterial vasodilation- Which drug?
-Answer- Cilostazol
phosphodiesterase inhibitor; leads to increased cAMP which inhibits platelet
degranulation/activation while also causing vasodilation

A 3 month-old boy - facial abnormalities and weakness since birth. PE shows coarse
facial features and hypotonia. Mucopolysaccharides within reference range.

Concentrations of lysosomal enzymes are increased in serum and decreased in


cultured skin in fibroblasts. Which of the following is decreased in patient's lysosoma
enzymes? -Answer- Decreased Mannose-6-phosphate
l

A Newborn with lethal chromosome defect. He is unable to suck sufficiently. He has episodes of
crying that occur every 2.25 hours and relieved by tube feeding. Which best describes the most
appropriate clinical approach? -Answer- feed by whatever necessary to maintain comfort

An 18 y/o woman - fever, sore throat, fatigue for 1 week. Temp 100.6. PE shows pharyngeal
erythema, generalized tender lymphadenopathy, splenomegaly. Lab show 20% atypical
lymphocytes. Which accounts for atypical lymphocytes in patients peripheral blood smear? -
Answer- Reactive T - Lymphocytes

51-year-old man - acute onset of fever and respiratory failure 6 weeks after cadaveric renal
transplant. He currently takes cyclosporine, bactrim, prednisone, and TMP_SMX. Temp high -
Respirations increased. Diffuse inspiratory and expiratory crackles. Creatinine increased. X-ray
of chest shown. Cause? -Answer- Cytomegalovirus Organ transplant patients are at an
increased risk of CMV pneumonia.
A 2 month old - 6 week history of constipation. She has poor feeding, sleepiness, and yellow
skin. Her parents are migrant workers and have not received routine medical care. PE shows
lethargy, hypotonia, jaundice, large fontanelles, macroglossia, and umbilical hernia. Most
appropriate next step to establish diagnosis? -Answer- Thyroid Function Test CRETINISM

A 39 y/o reports bright red spots on toilet paper after defecating. Rectal examination
shows large swollen rectal veins. Which is predisposing cause of these lesions? - Answer-
constipation
Large swollen rectal veins --> patient has external hemorrhoids.
Swollen and inflamed veins in the rectum and anus that cause discomfort and bleeding. The
most common cause of external hemorrhoids is repeated straining while having a bowel
movement.

A 45 y/o develops proteinuria and hematuria. She has taken 3g of ibuprofen for headaches daily
for the past 2 years. Which of the following is most likely obtained on renal biopsy? -Answer-
Tubulointerstitial nephritis
Acute interstitial renal inflammation. Pyuria (classically eosinophils) and azotemia occurring
after administration of drugs that act as haptens, inducing hypersensitivity (eg, diuretics,
NSAIDs, penicillin derivatives, proton pump inhibitors, rifampin, quinolones, sulfonamides).

A 92 y/o - recently admitted to a nursing home because of progressive dementia has large
purpuric lesions over the dorsa of both forearms and hands. Family states shew had similar
lesions over the years. Lesions occur in absence of trauma. Platelet count is normal. Which is
most likely explanation for purpura? -Answer- atrophy of dermal collagen

10-year-old boy has mass in ventral midline of the neck just inferior to hyoid bone.
This structure is developed from which of the following embryonic tissues? -Answer- Endoderm
of Foramen caecumThe frequency of autosomal recessive disease in a population is 1/1600. A
deletion has been identified that accounts for 80% of mutations at this locus. 20% of mutations
result in point mutations. Frequency of deletion carriers in the population is closest which of
the following?
A 62-year-old woman - progressive bilateral lower abdominal pain and distention. Pelvic
exam shows adnexal mass. Malignant cells. Paclitaxel is initiated. Treatment targets
which in tumor? -Answer- B- Tubulin

hyperstabilizes polymerized microtubules (made up of alpha- and beta- tubulin)


Taxes Stabilize society

The frequency of autosomal recessive disease in a population is 1/1600. A deletion has


been identified that accounts for 80% of mutations at this locus. 20% of mutations resul

t
i
n
p
o
i
n
t
m
u
t
a
t
i
o
n
s
.
F
r
e
q
u
e
n
c
y
o
f
d
e
l
e
t
i
o
n
c
a
r
r
i
e
r
s
i
n
t
h
e
p
o
p
u
l
a
t
i
o
n
i
s
c
l
o
s
e
s
t
w
h
i
c
h
o
f
t
h
e
f
o
l
l
o
w
i
n
g
?
-
A
n
s
w
e
r
-
1
/
2
5
=sqrt(1/1600) = 1/40
(1/40)*80% = .02
2* 1* .02 = .04 = 1/25

A 46 y/o has dyspnea, orthopnea, midsystolic murmur best heard over the cardiac apex. An ECG
shows a left atrial abnormality. Echo shows enlarged left atrium and normal sized ventricles.
Which findings? -Answer- Mitral Regurgitation

A 19-year-old woman - refractory pustular acne - unresponsive to several topical and systemic
therapies. Treatment with isotretinoin. Which of the following pharmacologic effects is
dependent on binding to the RXR receptor? -Answer- Teratogenic effects in the embryo
hormone superfamily of nuclear receptors (NRs) that predominately function as transcription
factors with roles in development, cell differentiation, metabolism, and cell death

A 35 y/o man comes to physician because of a 2 month history of inflamed thickened silvery
scales on scalp, trunk, elbows, and knees. This condition not responsive to coal tar,
calcipotriene, and triamcinolone. Which drug is most appropriate? -Answer- Methotrexate
A 55-year-old with T2DM, HTN, hyperlipidemia develops myalgias with weakness. Serum
CK increased. Urine is positive for myoglobin. Which drugs responsible for
symptoms? -Answer- Pravastatin
statin-induced myopathy
In patients with breast cancer, metabolically stable agonists of gonadotropin-releasing
hormone are effective because they inhibit release of which? -Answer- Gondatropin by the
pituitary Gland
GnRH agonists like Leuprolide - continuous fashion, they downregulate the GnRH receptor in
the pituitary and ultimately decrease FSH and LH.

83 y/o home bed-ridden and confused. Temp 96F. Hypotensive (BP 85/50). Given 1 lite
of saline hypotension. Pulmonary catheter inserted (Cardiac Output high, Pulmonary
Cap Wedge Pressure Low, Systemic Vascular Resistance low) - cause of hypotension?
-Answer- Early Septic Shock

distributive shock which is marked by massive vasodilation causing decreased SVR,


decreased preload / PCWP, and increased CO
r

23-year old - 1-month history of weakness and muscle pain after vigorous exercise. (Family
history similar). A muscle biopsy specimen shows ragged red fibers. Mutation in a gene located
in which structure on photomicrograph? -Answer- Area labeled B Mitochondrial myopathies
show "ragged red fibers" on muscle biopsy due to the accumulation of diseased mitochondria
in the subsarcolemma of the muscle fiber
A 32 y/o woman - receives prescription for tetracycline and is instructed to not to take
tetracycline with milk. If tetracycline is taken with milk, its absorption is decreased due to
which? -Answer- Formation of complexes with mineral ions
A 23 year old F - Sjogren syndrome 6 years - she has burning pain in her toes for past
month. Which neurotransmitters is most likely mediating patients pain? -Answer-
Substance P

undecapeptide present in the CNS and the peripheral nervous system.


involved in the synaptic transmission of pain and other nerve impulses.
22 y/o fever, chills, muscle aches. 6 hours ago - injection with Penicillin G for syphilis.
Temp is 102. PE shows no abnormalities. Which explains patient's symptoms? -
Answer- release of bacterial products

Administration of Penicillin for Syphilis may lead to the Jarisch-Herxheimer reaction

Herxheimer reaction is that treatment results in the sudden death and destruction of
large numbers of treponemes, with the liberation of protein products and toxins
A 48 year old - 2 hours of dizziness and palpitations with sustained, rapid, irregular heartbeat.
ECG shows AFIB. If Amiodarone administered, the patient is at greatest risk of which adverse
effects? -Answer- Hypothyroidism
amiodarone - associated with a number of side effects, including thyroid dysfunction (both
hypo- and hyperthyroidism
Removal of Thymus at birth results in severely impaired immune responses. However, if the
thymus is removed from adults, little if any deficit in immune responsiveness occurs because of
which of the following reasons? -Answer- Thymic lymphocytes produce before thymectomy are
long lived
Thymus produces all of your T cells by the time you reach puberty.
By age 75, the thymus is little more than fatty tissue.
thymus produces all of your T cells by the time you reach puberty.
Thymocytes are long-lived and that's why you can lose your thymus without impairment of your
immune system.

A 4-week infant - vomited after feeding for past 2 days. Recent episode was very forceful
regurgitation through the nostrils. Physical exam shows mild abdominal distention with visible
peristalsis. Explanation of these findings? -Answer- Hypertrophic Pyloric Stenosis olive-shaped
mass in epigastric region,

A 45-year old woman - asymmetric enlargement of the thyroid gland during PE 6 weeks ago.
She underwent adrenalectomy for pheochromocytoma 3 years ago. Thyroid lesions are
composed of spindle cells arranged in small clusters. Deposits of amyloid are present between
neoplastic cells. Foci of C-Cell Hyperplasia are also present. The
lesions are confirmed as malignant. Which of the following marked is most appropriate for the
development of the recurrence of the thyroid neoplasm? -Answer- Calcitonin Patient has
medullary carcinoma. Malignant proliferation of parafollicular "C" cells that produce calcitonin
and have sheets of cells in an amyloid stroma.
Patient likely has MEN 2A or 2B with the presence of medullary thyroid cancer and
pheochromocytoma

62 year old man - pain in left hip for 3 weeks. An x-ray of the hip shows a 4.5cm, destructive,
osteoblastic lesion with associsted fracture in the proximal femur and two separate smaller
lesions in the bony pelvis. A needle core biopsy of the femur shows metastic carcinoma. Which
is the most likley primary site of the cancer? -Answer-
Prostate
Osteoblastic lesions are associated with prostatic adenocarcinoma mets.
Increased serum ALP
OB = osteoblastic
OC = osteoclastic/osteolytic
P=prostate, B=breast, K=kidney, T=thyroid, L=lung
P--B--K--T--L
P = OB

B = OB/OC
K = OC
T = OC
L = OB/OC
During a clinical study, investigator tests a new drug for treatment of breast cancer. A
population of 10,000 patients with breast cancer is recruited and randomized into a treatment
group and control group. Analysis shows a p-value of 0.1 with no significant difference in the
treatment outcomes between the experimental and control. When repeating, which is most
likely to decrease making a beta error? -Answer- Increase sample size
You are aiming to increase power and you can do so by increasing sample size (reduce B error).
"power in numbers

23-year-old man - 3-year history of schizophrenia. He has been taking an antipsychotic


medication for hallucinations, paranoia, and disorganized thoughts. Pt does not make eye
contact. Lab show serum sodium of 120 mEq/l, urine sodium of 8, and urine osm of 80. Which is
most likely cause of lab finding? -Answer- Psychogenic Polydipsia hyponatremia + a low ADH =
psychogenic polydipsia

A 12 year old - girl - 3 months ago, she sustained a complex fracture of the left tibia, which
required cast immobilization. The cast is removed. Examination of the left calf shows that it is
smaller than the right calf. Which of processes in the patient's myocytes is most likely cause of
findings? -Answer- polyubiquitnation
atrophy is decrease in tissue mass due to decrease in size (increased cytoskeleton degradation
via ubiquitin-proteasome pathway and autophagy
A 45-year-old man - tooth abscess - He has not seen a physician or a dentist in over 10 years. He
lives alone, unemployed. He has no friends, maintains minimal contact with relatives. No
thought disorder. Which is most likely DX? -Answer- Schizoid personality disorder

60 y/o prolonged apnea - after general anesthesia. Plasma cholinesterase is abnormal.


Which agent is most likely responsible for prolonged apnea? -Answer- succinylcholine
Strong AcH Receptor Agonist cause respiratory depression duration of action of
Succinylcholine is determined by its metabolism by plasma. cholinesterase. abnormal
plasma cholinesterase (=pseudocholinesterase), it will lead to delayed metabolism
complications = hypercalcemia, hyperkalemia, and malignant hypertension

Two days after admission to hospital for drug induced thrombocytopenia, a 37 year old woman
develops severe headache and becomes comatose. She has few petechia and low platelets. A
CT scan of the head shows intracranial hemorrhage. A platelet transfusion is recommended, but
the patient is a Jehovah's witness and stated she did not want a transfusion of any blood
products. Husband wants her to have a transfusion.
Most appropriate management? -Answer- Do not proceed with transfusion
Autonomy = always, always follow the patient's wishes (in this case you are aware that the
patient did not want a transfusion

13 y/o girl has episode of severe cellular rejection necessitating maximal immunosuppression 6
months after a cadaveric renal transplantation. Two weeks later, she develops
lymphadenopathy and hepatosplenomegaly. Exam of lymph nodes shows monomorphous
population of B lymphocytes. Which finding is most likely on evaluation of these cells? -Answer-
Epstein Barr Virus Genome
A new drug has the following effects on the activity of the primary arachidonic acid
metabolites:
neutrophils, Leukocyte chemotaxis - inhibition
Platetlets - no effect
Mast Cell - no effect
Endothelium - no effect
Which of the following is the most likely site of drug action (image shown)? -AnswerLeukotriene
B4
Leukotriene B4 is a potent chemotactic molecule for neutrophils. Selective loss of this would
have no effect on platelets, mast cells, or endothelium C5a, IL-8, LTB4
4 y/o has a a fever, abdominal cramping, and repeated bloody mucoid stools with tenesmus-
Several other children in the daycare have a similar illness. Direct microscopic exam of
methylene blue-stained fecal smear shows numerous neutrophils. Which is causal organism? -
Answer- Shigella Sonnei

A 3y/o boy is brought to physician because of 1 month history of pale skin. PE shows pallor. Lab
studies show: Hemoglobin and hematocrit low. A photomicrograph of a peripheral blood smear
is shown. Genetic testing is most likely to show which of the following? -Answer-
Heterozygoues mutation of Ankyrin Gene
Hereditary spherocytosis - defect in proteins interacting with RBC membrane skeleton and
plasma membrane (ankyrin, band 3, protein 4.2, spectrin).
Mostly autosomal dominant inheritance (so heterozygous mutation since you only need one
mutant allele to get the disease).

A 50-year-old woman - brought to ED by her husband - found her in semicomatose state with
an empty bottle next to her. The bottle had contained a sodium salt drug with CNS effects.
Alkalization of her urine will cause greatest absolute increase in elimination of the
unmetabolized drug if it has which of the following pharmacokinetic profiles? -Answer-
A 50-year-old woman - brought to ED by her husband - found her in semicomatose state with
an empty bottle next to her. The bottle had contained a sodium salt drug with CNS effects.
Alkalization of her urine will cause greatest absolute increase in elimination of the
unmetabolized drug if it has which of the following pharmacokinetic profiles? -Answer- pKA
6.0 protein binding in plasma = 20% Volume Distribution = 1 L/kg

drug to be cleared by the kidney, it must first be filtered in the glomeruli.


Drugs with a high VD have more of the drug in tissue - not available to filtered by kidney Drugs
with high protein binding won't be filtered either.
We want a drug with low Vd and low binding if you want it cleared via the kidneys/urine pKa is
pH at which any drug is at its 50% ionized stat if pKa of drug is 6---at pH 7 we will start
eliminating

.18 y/o women screening for colon cancer - Her father and grandfather died of colon cancer at
ages of 36 and 45 years, respectively. PE shows no abnormalities. Colonoscopy shows approx.
200 lesions throughout the colon. Colectomy is done
(Photograph) Which is the chance that the offspring will develop adenocarcinoma? -
Answer- 50% inherited
Familial adenomatous polyposis is an autosomal dominant mutation.
Thousands of polyps arise starting after puberty; pancolonic; always involves rectum.
Prophylactic colectomy or else 100% progress to CRC.
Autosomal dominant diseases have, on average, 50% chance of being passed down to offspring.
23y/o excessive bleeding while brushing teeth - PE shows multiple ecchymoses. Intensive
chemotherapy is planned. Before this treatment is initiated to decrease this patients risk for
kidney failure, it is most appropriate to administer a drug that inhibits or binds to which of the
following sites? -Answer- Xanthine Oxidase
Allopurinol inhibits xanthine oxidase. It is used for chronic gout as well as prevention of
tumor-lysis associated urate nephropathy
A 17-year-old comes - swollen tender ankle. X-ray strongly suggestive of osteosarcoma.
She and her parents are told only that "further evaluation is needed." Next Steps? -
Answer- Speak to both girl and patient and family

A 17-year-old comes - swollen tender ankle. X-ray strongly suggestive of osteosarcoma.


She and her parents are told only that "further evaluation is needed." Next Steps? -
Answer- Speak to patient and her parents about findings
Sex Drugs or Psych - speak to patient alone
A 25 year old woman - mass in right axilla. 1 week history of malaise, headaches, and night
sweats. Pt adopted a kitten and has sustained several bite and scratch marks. Tenderness in
right axiallry lymph node. The skin over the node is erythematous, tough, and warm. Bartonella
Henselae antibody test is positive. A biopsy of this lymph node is most likely to show which
histological pattern? -Answer- Granulomas containing stellate micro abscesses + Axillary mass
catch scratch competent immune system - stellate necrotizing granulomas.

A 35-year-old man - comes to physician to discuss donating kidney to his sister who has T1DM.
Which describes likelihood that this donors HLA type will match his sister? -
Answer- 1:4
You have a 25% chance of inheriting the same HLA markers as your siblings. Two siblings
have a 25% chance of being genotypically HLA identical,
a 50% chance of being HLA haploidentical (sharing one haplotype), and a 25% chance that they
share no HLA haplotypes.

A 50-year-old woman - restrictive pulmonary disorder with no obstructive component is most


likely to have which of the following sets of pulmonary volumes? -Answer- FRC - Decreased
Residual Volumes - Decreased
Vital Capacity - Decreased
restrictive lung disease- all lung values are DECREASED ("restricted") except for the
FEV1/FVC ratio.

FEV1/FVC ratio may be normal or increased

29 y/o - 1 week history of fever, muscle aches, sore throat, and non-productive cough.
Rhonchi
Heard on auscultation of chest. CXR shows bronchopneumonia of right lower lobe. Symptoms
persist after 7 days of amoxicillin therapy. Cold Agglutinins positive. Which of the following
features of the causal organism best explains the ineffectiveness of the pharmacotherapy? -
Answer- Absence of peptidoglycan Mycobacterium - does not CELL WALL

A 16 year old - 2 month history of wheezing and shortness of breath while participating in
track meets. Diagnosed with exercised-induced asthma. Exposure to which is most likely to
precipitate a similar episode in this patient in the future? -Answer- Cold, Dry Air exercise
induced asthma. when exercising, breathing through the nose is decreased and replaced by
more vigorous mouth breathing. This means the air is not humidified and warmed up. The air
is less dense and the asthma ensues.

A 43 y/o female -initial prenatal visit. PE shows uterus consistent in size with a 10-week
gestation. This patient is at increased risk for a child with Down Syndrome if she has which of
the following sets of ultrasound and serum findings? -Answer- Fetal Ultrasound
Translucency - Increased
Pregnancy Associated Plasma Protein- Decreased
Human Chorionic Gonadotropin - Increased

A male newborn - decreased blood thyroxine on newborn screening. PE shows no


abnormalities. Serum T3 and T4 decreased - Serum free T4 and TSH within reference ranges.
Which best explains findings? -Answer- Thyroid Hormone Binding Globulin deficiency
Decreased total, normal free (unbound) = Thyroid hormone-binding globulin deficiency
Thyroxine-binding globulin (TBG) deficiency is characterized by low serum total T4 but normal
free T4 and TSH;

the diagnosis is confirmed by measuring TBG concentrations


A study conducted to assess the effect of a new screening test on the mortality of prostate
cancer. Prior to implementation, the overall 5-year survival was 70%. After screening of random
population, 5-year survival rate increases to 85%. No changes in treatment. The decrease in
mortality could be due to which type of bias? -Answer- Lead Time
caused by early detection being confused with increased survival. early detection makes it seem
as though survival has increased, but the natural history of the disease has not been impacted.

50 y/o come to physician because of a 6-week history of depression, anxiety, difficulty


sleeping, decreased appetite, and poor memory and concentration. He has an MI 3 months
ago. He has one glass of wine each evening. Which of the following most likely decrease this
patient's risk of mortality over the next 2 years? -Answer- Antidepressant Therapy
6-week history (e.g. >2 weeks) of depression
(1), difficulty sleeping (2), fatigue (3), decreased appetite (4), and poor
memory/concentration (5)
At a postnatal checkup - 6-week female newborn weighs 20 ounces less than delivery. Vomits
and regurgitates bile-tinged fluid after every feeding. Which of the following is the most likely
cause of abnormalities? -Answer- Annular Pancreas bile in the vomit;
abnormal rotation of ventral pancreatic bud forms a ring of pancreatic tissue encircles 2nd part
of duodenum; may cause duodenal narrowing (arrows in) and vomiting
An investigator compares the DNA sequence of a newborns with achondroplasia and
healthy newborns. DNA sequence shows a G-C mutation in the FGF3 gene on
chromone 4.

Achnodroplasia 5'TACCGGGT-3'
Healthy 5'TACGGGGT-3'
As a result of this mutation an MspL restriction enzyme site is created that allows for a
convenient diagnostic test. Which of the following sequences most likely represents the
substrate specificity for Mspl? -Answer- 5'CCGG restriction enzymes bind palindromes. both
5'CCGG or 3'GGCC would have been acceptable in this scenario.

18 y/o F - never had a menstrual period. PE shows a 1 -cm invagination where vagina would
normally be present. No uterus palpable. Lab show FSH, LH, estradiol, and testosterone in
normal range. Which of the following structures is most likely normal? Answer- Ovaries
mullerian agenesis. Normal ovaries but absent uterus.
Underdevelopment of the Mullerian system leading to congential absence of the vagina.
Usually no cervix or uterus.

25 y/o man - severe Pain in low back radiates down his left leg- he started a weight- lifting -
tried to lift a 200lb bar form ground over his head. Pain began immediately after this attempt.
Cause of this patient's pain? -Answer- Rupture of an intervertebral disc radiculopathy - disc
herniation (radiates to the leg aka lasegue sign), degenerative osteoarthritis (positional
relieved with rest), spinal stenossi(pain with standing),
spondylorpathy(relieved with exercise,dominant at rest),
spinal metastasis (constant pain, worse at night, not relieved by positional changes vertebral
osteomyelitis(focal tenderness, acute back pain, and fever), aortic dissection (severe
restrosternal pain, radiating to the back)

A 28-year-old man - excessive thirst and polyuria. Labs


Baseline Serum Glucose 75, Serum Osm 310, Urine Osm 175

Water deprivation - no change in serum glucose, serum Osm, or urine OsM


With ADH added - Urine OsM icnrease

Which is the primary site of the pathophysiolohic process in this patient? -AnswerHypothalmus
Central DI= responds to vasopressin, high serum Na
Nephrogenic = responds to nothing, normal serum Na

80 y/o - T2DM - 2 month history of severe constipation. Laxatives provide no relief.


Distention of abdomen. Patient most likely has dysfunction of the following nerves? Answer-
pelvic splanchnic
Parasympathetic- no rest and digest

80 y/o - T2DM - 2 month history of severe constipation. Laxatives provide no relief.


Distention of abdomen. Patient most likely has dysfunction of the following nerves? -
Answer- pelvic splanchnic
a. Constipated - Parasympathetic- no rest and digest

25 y/o - F - 2 day history of fever, malaise, and a rash. Single painless lesion on labia which
resolves spontaneously. Sexually active and does not use condoms. PE shows diffuse rash on
palms and soles. A sensitive but non-specific rapid serologic test uses which reagents is most
likely to support diagnosis? -Answer- A phospholipid
(Cardiolipin) antibodies on charcoal particles
Single painless lesion on labia
Visualized by immunofluorescence or dark-field microscopy; serology is important - two types
of antibodies:
Cardiolipin sensitive but not specific for treponoma

A 44-year-old woman - pap smear for atypical squamous cells. Protein is known to promote
cell growth and malignancy by causing p53 protein degradation. This degradation begins
when the p53 protein is targeted by which of the following types of cellular enzymes? -
Answer- Ubiquitin Ligase starts degradation cycle p53 targeted degradation cycle

A 28 year old woman - Eastern European Jewish Descent - lump in her left breast. Maternal
grandmother was diagnosed with ovarian cancer, her maternal aunt was diagnosed with breast
cancer at age 36. Pt concerned about genetic mutation. Genetic testing most likely to show a
mutation in a component of which of the following pathways? -Answer- Recombinational
Double Stranded DNA breaks
Defective homologous recombination is seen in breast/ovarian cancers with the BRCA1 gene
mutation.
Graph (PICTURE) shows radiolabeled protein X that is bound by anti-X antibodies in
the presence of varying concentrations of protein Y. With respect to anti-X. which of the
following interpretations regarding the epitopes expressed by proteins X and Y is

correct? -Answer- Proteins X and Y express the same epitopes If they share the same
epitopes, it will have a downward slope.
If they share none of the same epitopes, the line will be horizontal across the graph

A previously healthy 32 year old - 2 hours of severe epigastric pain, nausea, and vomiting blood.
Pulse is 125/min, respirations are 18/min, and blood pressure is 85/45 mm Hg. PE shows cool
skin and tender abdomen. A decrease in which hemodynamic parameters is most likely to cause
patients hypotension? -Answer- Preload
Vomiting blood and cool skin indicates this is a type of hypovolemic shock
Decreased EDV means that the "filling volume" is decreased, which also means the
preload will be decreased

Which of the following changes in the cardiovascular system occurs with normal Aging?
-Answer- Increased Basal Systolic blood pressure

vascular scleorsis and stiffening- changes with aging- leads to high sys BP
A 55-year-old - constipation since starting an OTC medication 2 weeks ago for chronic,
persistent cough, and sinus congestion. Which ingredients in this medication cause
symptoms? -Answer- Dextromethorphan antitussive - (antagonizes NMDA receptor) mild
opioid effects when used in excess Mild abuse potential

19 y/o college student -sudden onset of right-sided chest pain and difficulty breathing after an
accident in which he was thrown from his bicycle. He has difficulty walking and cannot climb
stairs because of pain and shortness of breath. He is slightly cyanotic, afebrile, and tachypneic.
Which of the following is most suggestive that fractured ribs
caused the respiratory problem? -Answer- subcutaneous crepitus pneumothorax -
inspiratory stridor
When there is a fractured rib it will cause a trauma pneumothorax which can cause air to
escape and become trapped under the skin leading to crepitus.
Subcutaneous crepitus is very specific sound referencing air finding its way into the skin
which you can hear but also feel by rubbing your hand over the affected area
A physician prescribes newly marketed drug to 45 patients. Over the next several weeks, she
notes good efficacy, but elevated enzymes in 5 patients. Physician unable to find any official
data linking the new drug to liver dysfunction. Which is most appropriate action of physician? -
Answer- Discontinue the new drug in people who have effects and report the cases

14 y/o girl - 1 month history of migraine-like headaches, vomiting, and multiple left sides
seizures. She has hearing loss. Her mother and maternal grandmother have high tone deafness.
Pt most likely has a mutation of which of the following? -AnswerMitochondrial tRNALeu
It's a mitochondrial disease. - present in successive generations, always with maternal
transmission - damage to high-energy tissues - CSF lactic acidosis

An 18-y/o boy - diagnosed with Schizophrenia - Identical Twin brother is unaffected. Parents
are concerned that twin will have the disorder. Which represents brother's risk for developing
schizophrenia? -Answer- 50%
psychosis of about 50% in monozygotic twins seems to be a realistic estimate, which is
significantly higher than that in dizygotic twins of about 10-19%
32 y/o fecal incontinence after pregnancy - what was damaged? -Answer- Damage to
anal sphincter

A stretch injury during childbirth will result in damage to the external ureteral and anal
sphincters and damage to the pudendal nerve (S2-S4). This can lead to decreased
sensation in the perineal and genital area and fecal or urinary incontinence
78 y/o - 3 week history of severe weakness and fatigue. 16lb weight Loss - Daughter usually
bring groceries twice per week. Daughter on vacation. He is only eating crackers and black
coffee. PE shows generalized muscle weakness. Serum studies most likely to show an increased
concentration of which hormones? -Answer- Cortisol b. cortisol's functions is to increase
gluconeogenesis, lipolysis, and proteolysis. c. helps maintain blood pressure even in the setting
where he is malnourished.

66 y/o old man - comes to ED after MVC. Pronounced dead on arrival. Photomicrograph of a
section of distal left ureter taken at autopsy are shown. Which of the following was the most
likely predisposing factor in the development of the ureteral lesions in this patient? -Answer-
Cigarette Smoking
transitional cell carcinoma, which smoking is a common risk factor for; it can involve the
renal pelvis/calyces.
histo image shows the papillary nature of the tumor

A 36 y/o woman - 1-month history of joint pain, frequent headaches, and fatigue. PE shows
edema of the upper and lower extremities. An abdominal CT scan is shown. Which findings
most likely seen in this patient? -Answer- Splenomegaly

56 y/o -frequently burned herself while cooking over the past months. Exam shows loss of pain
and temperature sensation in both upper extremities. Touch, vibration, and proprioception are
normal. Some wasting of her hands. Which of the following findings on MRI of the head and
spine? -Answer- Syrinx of the central region of the spine C4 - T5
70 y/o man - 3-month history of weakness and a 22lb weight loss. 2 pack of cigarettes
daily for 50 years. An x-ray shows a 4-cm lesion in right upper lobe. Serum calcium
concentration is 13.5 mg/dl. A malignant lesion arising in a segmental bronchus is seen during
lobectomy. DX? -Answer- squamous cell carcinoma
ARISE FROM BRONCHUS , Cavitation, Cigarattes, HyperCalcemia (produce PTHrp)
23 y/o woman - fever, hypotension, and DIC for 24 hours. She splenectomy following trauma.
Which bacteria is cause of sepsis? -Answer- Streptococcus pneumonia Encapsulated organisms
run rampant in patients who have no spleen, whether physically or functionally. (Recall the
wide-array of sequalae sickle cell patients experience thanks to their functional
autosplenectomy.)

Female Newborn - Respiratory Distress, Cyanosis, Nasogastric tube placed - what embryologic
events cause issue? -Answer- incomplete formation of Pleuroperitoneal membrane
Diaphragmatic hernia

A 70-year-old - woman - transferred to rehab service 2 days after operative repair of a fracture
of femur. She weighs 6Kg (143 lb). PE exam in unremarkable. Most important predictor of
success in rehab -Answer- Activity level before the fracture
Activity level increase in Bone density(Specially Weigh bearing) and OsteoBlastic activity
27 y/o woman - 3-month history of intermittent headaches, palpitations. Blood pressure
125/80 in a chair. When asked to sit on the exam table, she has flushing, and her blood
pressure is 185/115. PE shows no abnormalities. Most likely diagnosis? -Answer-
Pheochromocytoma

When standing up, the body normally activates sympathetic system to avoid orthostatic
hypotension.

But since there is now an additive effect of the pheochromocytoma adrenergics, it will
lead to a hypertension

Pt with 1-week history of diarrhea has reduced tissue turgor. Arterial pH 7.3. Serum
bicard is 15 mEq/L. Pt is most likely to have which of the following? -Answer-
Decreased arterial PC02

chronic diarrhea leading to normal anion gap metabolic acidosis


pH = HCO3/CO2
12 y/o girl - dentist found many unerupted and supernumerary teeth. PE shows frontal bossing,
hytpertelorism, and retained deciduous teeth. Decreased ALK phos. A CXR shows hypoplasia of
clavicles. Skull X-rays show open sutures and Wormian bones. Mutation of CBFA1 gene. Which
cell types is affected by the mutation? -AnswerOsteoblast
ALP is decreased. Osteoblast activity is measured by bone ALP
A 57-year-old man - hemoglobin concentration of 18.5 g/dl. A peripheral blood smear shown.
Findings most consistent with which disorder? -Answer- Chronic Obstructive Pulmonary Disease
Absolute polycythemia vera, which is due to high altitude or lung disease.

38 y/o African American - recently diagnosed with hypertension - patient aggreed to initiate a
regime including physical activity, low sodium diet, and pharmacotherapy. PE shows no
abnormalities. What is the role of ethnicity when selecting appropriate medication? -Answer- It
should play a role because the efficacy of certain classes of medications varies among different
ethnicities
A new antiplatelet agent for prevention in stroke. In a large randomized trial, the rate of stroke
are lower in patients receiving the new agent than in patients receiving standard treatment.
Results Stroke New Platelet Drug
Women =. .12 .04
Based on results which of the following is the absolute risk reduction in women? Answer- 8%
Absolute risk: the difference in risk (not the proportion) attributable to the intervention as
compared to a control.
(.12) - (.04) = .08
ARR = 8%

65 y/o man - substernal chest pain, nausea, sweating for 1 hour. 10-year history of T2DM. Pulse
is 120/min. BP 98/60. PE shows a JVD of 12cm. Crackles are heard halfway up lungs fields. ECG
findings? -Answer- ST elevation
Continued or severe ischemia (>20 minutes) leads to transmural necrosis involving most of the
myocardial wall (transmural infarction); EKG shows ST-segment elevation. ST will first decrease;
however, after 20 min it will increase (elevate)

30 y/o comes to physician - 3-month history of pain in his jaw and left arm. Intense thirst and
urination. Found to have peptic ulcer for 2 month. Serum calcium 13.5. PTH 110 (N= 10-65). X-
ray normal. Which explains impaired calcium homeostasis? -Answer- Increased Osteoclast and
Maturation and activity hypercalcemia (stones, bones, groans, THRONES [increased diuresis},
and psychiatric overtones).

Following a wedding reception - 25 adults develop low grade fever, abdominal cramping,
vomiting, and diarrhea. No blood in stool, No pathogens on stool culture. Several family
members exhibit similar symptoms 3 days after initial outbreak. Causal organisms? -Answer-
Norovirus

Legionella Pneumophilia -Answer- gram negative rods

use silver stain.

Grow on charcoal
yeast extract medium with iron and cysteine.

Aerosol transmission

Noperson-to-person transmission

31 y/o man - large, yellow, soft mass deeply infiltrating the gluteus maximus muscle.
Microscopic exam shows irregular vacuolated cells and clear cells with frequent mitoses.
DX? -Answer- Liposarcoma malignant, increased mitotic activity most common soft-tissue
tumors in adult high mitotic index & infiltrative nature indicate that the mass is malignant
An otherwise healthy 4-month-old girl is brought to physician because of lesion shown
(strawberry hemangioma). The lesion has been present for 2 months and growing rapidly.
Which is expected over the next 5 years? -Answer- spontaneously involution strawberry
hemangiomas tend to grow and then randomly involute. appears early in life, grows rapidly
and regresses spontaneously by 5-8 years old
A 2-year old boy - increased thirst, urinary frequency, and failure to thrive. He is 5th %
for height and 7% for weight. PE shows dehydration and decreased muscle tone. The
diagnosis of Fanconi syndrome is made. Which sets of changes in fractional
reabsorption in the kidneys is most likely in this patient? -Answer- All Decreased
Amino Acid Decreased
Glucose Decreased
Phosphate Decreased
HCO3 Decreased

A 28-year-old - man - fatigue and back pain for 6 weeks. 13-lb weight loss. DX of TB of lumbar
spine. When placed in supine position, patient holds right lower extremity in rigid extension.
Spread of the infection to the lumbar vertebrae is most likely causing the patient to guard
which muscle in supine position? -Answer- Psoas Major flexion of hip
"rigid extension", hence he does not want to flex
A 53-year-old woman - 1 year history of abdominal pain. Travelers diarrhea and UTI.
Avid traveler to Middle East, South America, and Asia. Photomicrograph of stool
sample. Which infectious agent? -Answer- Schistosoma Mansoni
lateral spine
55 y/o women follow-up examination - She has a family history of skin cancer and avoids sun.
Takes Vit D. PE no abnormalities. Serum studies show decreased 25 -
hydroxycholecalciferol concentration. Decreased production of which precursors in skin
is most likely? -Answer- Cholecalciferol

D3 (cholecalciferol) from exposure of skin (stratum basale) to sun, ingestion of fish,


milk, plants.

D2 (ergocalciferol) from ingestion of plants, fungi, yeasts

A 45-year-old man - fever, chills, dysuria, and a tender, enlarged prostate. Which is
most likely causal organism? -Answer- Escherichia Coli

Prostatitis is characterized by dysuria, frequency, urgency, low back pain. Warm,


tender, enlarged prostate.

Acute bacterial prostatitis—in older men most common bacterium is E. coli.

33y/o Falling on outstretched right hand - PE shows swelling on palmar side of wrist. No
pain in anatomical snuff box. Most likely cause of findings dislocation of which of the
following bones? -Answer- Lunate
Dislocation of lunate may cause acute carpal tunnel syndrome
A mother who has toxoplasmosis gives birth to an asymptomatic female newborn. Which of the
following tests on newborn will indicate that the newborn has congenital toxoplasma gondii
infection? -Answer- Anti-toxoplasma IgM antibody concentrations
baby does not get any maternal IgM, IgA or IgE as they do not cross the placenta, so if
IgM is found it may suggest the baby has encountered an infection in utero.
IgG is passed down to the baby as a means of passive immunity until the baby can form their
own antibodies of different types. So, if you see anything other than IgG (e.g. IgM) you know it
must be d/t an infection.

50 y/o man comes to the physician because of persistent cough for past 2 months. He is
farmer and started itraconazole therapy for histoplasmosis acquired through chicken coops.
Current medications include hydrochlorothiazide, enalapril, atenolol, omeprazole, and
metoclopramide for HTN. An interaction between itraconazole and which following drugs
most likely accounts for the lack of effect of itraconazole? - Answer- Omeprazole
CYP 450 inhibitor (SICKFACES.COM)

S odium valproate
I soniazid
C imetidine
K etoconazole
F luconazole
A cute alcohol abuse
C hloramphenicol
E rythromycin/clarithromycin
S ulfonamides
C iprofloxacin
O meprazole

M etronidazole
A miodarone
Grapefruit juice
A 75-year-old woman - burning, itching, and rash. Exam shows clustered skin lesions overlying
two ribs near the right axillary line. Some of the lesions are fluid-filled blisters and have crusty
scabs. Light microscope shows which findings? -Answer- Multinucleated giant cells and
neutrophil infiltrates
Shingles. Herpes simplex and herpes zoster viruses cause abnormal cell division in epidermal
cells, and this creates multinucleated giant cells.
A Tzank smear showing multinucleated giant cells is characteristic of Varicella Zoster
Virus infections

A 5-year old girl with AIDS develops vesicular rash. Antiviral therapy is ineffective and
the lesions become hyperkeratotic. The genetic mutation responsible for the
ineffectiveness involves the activity of which enzymes? -Answer- Thymidine-Kinase
60 y/op man - 2 pillow orthopnea - severe dyspnea, edema of lower extremities. Onset was
preceded by an episode of prolonged substernal chest pain for 5 days. Which of the following is
most consistent with these findings? -Answer- Jugular Venous Pressure
12 mm Hg
patient has heart failure.
Normal JVP is 6-8 mmHg.
A 10-year-old girl - not yet menstruated. PE shows absence of breast bud development and no
pubic or axillary hair. The most asks what is the first objective sign of puberty? -
Answer- Breast Bud Development first
Females: Thelarche -> Pubarche -> growth spurt -> Menarche
Males: Testicular growth -> Penile Growth -> Pubarche -> Growth Spurt ->
Spermatogenesis
Monoclonality of Neoplastic Cells in endometrial carcinoma can be best determined by analysis
of which of the following? -Answer- X chromosome-linked isoenzymes Neoplasia is new tissue
growth that is unregulated, irreversible, and monoclonal. Clonality can be determined by
glucose-6-phosphate dehydrogenase (G6PD) enzyme isoforms. G6PD is X-linked.

56 y/o man - worsening fatigue and lower extremity edema. poorly controlled HTN - BP
195/110. BUN is 70mg/dl, CR 7.0. Which is most likely pathologic finding in kidneys? -
Answer- fibromuscular hyperplasia of arterioles
56 y/o woman - brought to ER - progressive SOBS. 2 weeks ago she underwent coronary
artery bypass. Dullness to percussion. CXR shows fluid in left pleural cavity. Thoracentesis
yield 1200 ml of pale, milky chyle. Which produced iatrogenic chylothorax? -Answer-
Placement of the central line via the left jugular vein nicking the thoracic duct, specifically at
its inlet, the left subclavian.
Placement of the central line via left internal jugular vein can cause damage to thoracic duct
Placement of pulmonary artery can cause damage to right lymphatic duct.

A 30 year old won with multiple sclerosis- 2 week history of shooting pains in cheek- lasts for
less than 1 second. Cause of pain is plaque in which location? -Answer- pons trigeminal
neuralgia
MIDBRAIN - 1,2,3,4
PONS- 5,6,7,8
Medulla 9,10,11,12
Serum cholesterol study for women by age. Assuming gaussian distribution, what is
probability that a woman between 50 and 54 has a serum cholesterol greater than 296?
Age Cholesterol
50-54 246+50 -Answer- 16%
68% of the data will fall within 1SD of the mean.
100%-68% = 34
16% will fall above and 16% will fall below 1
SD. 2STD = top 5%

36 y/o - not urinated for 24 hours. Intermittent renal calculi - Ultrasound shows bilateral
hydronephrosis. Creatinine increased, K+ increased - Bilateral nephrostomy tubes are placed.
Which predicts likely changes in urine and potassium excretion during next 24 hours? -
Answer- Increased Urine Output Increased Urine Potassium
urine output increased: pretty simply they are putting tubes in to increase the urine flow into
the bag.
If you increase urine flow rate you increase K+ secretion. That's why diuretics like thiazides and
loops increase K+ secretion

A 26-year-old man - stab wound in LUQ - laceration of fundus of the stomach. During
mobilization, he develops bleeding due to damage to short gastric arteries. Short gastric
arteries are a branch of which arteries? -Answer- splenic Short gastric a. branch from the
splenic a.
Branches of the celiac trunk that constitute the blood supply to the stomach: common hepatic,
splenic, and left gastric.
17-year-old girl - 4-day history of headache and vomiting. Right sided hemiparesis.
Serum pyruvate and lactate increased. Deficiency in NADH dehydrogenase. Pedigree
shown. Which is mode of inheritance of the disorder? -Answer- Mitochondrial

mitochondrial - passed by the mother

Mothers have diseased children; Fathers don't


A 48 y/o - 6 month history of angina - Cholesterol 450, LDL increased. Treatment with statin is
begun. Statins decrease serum cholesterol by which mechanisms? -Answer- Upregulate LDL
receptors
Statins decrease cholesterol synthesis, which indirectly Statins indirectly cause increased LDL
receptor expression on hepatocytes

A randomized clinical trial - compare difference between 2 surgical procedures. Procedure A


has a lower rate wound infection Procedure B (relative risk of 0.66 with confidence interval of
.30-1.45. Which statement represents the comparison between
Procedure A and B in a clinical setting? -Answer- Neither procedure is superior
The CI value contained 1, which means that its insignificant
CI including 1 fails to reject the null hypothesis*
Odds ratio/*relative risk**, CI including 1 fails to reject the null hypothesis*

During an experiment, an investigator isolates abnormal elastin from connective tissue of mice.
Analysis of elastin shows a decreased number of desmosine cross links. Which amino acids in
the abnormal protein would be most likely different compared with normal elastin? -Answer-
Lysine
Lysine is used in elastin and collagen cross linking; it is cross linked by lysyl oxidase to
make collagen fibers

Desmosine is an amino acid found uniquely in elastin, a protein found in connective


tissue such as skin, lungs, and elastic arteries.

Desmosine is made up of four lysine residues. Therefore abnormal elastin is likely


missing lysine necessary for the formation of these desmosine cross-links.
A 50-year-old man - 3 hour history of confusion and difficulty seeing. He sees white spots that
interfere with vision. 3-year history of alcohol disorder. He has been drinking methanol. He is
confused and incoherent. PE shows dilated pupils and diminished pupillary reflex. Cause of
visual symptoms due to an excess of which of the following metabolites? -Answer- Formic Acid
Methanol is converted to formaldehyde via alcohol dehydrogenase (ADH) and formaldehyde is
converted to formic acid (formate) via aldehyde dehydrogenase (ALDH).

A 60-year-old woman - T2DM - substernal chest pain radiates to her left arm. She is diagnosed
with MI. 2 days later she develops acute respiratory distress. PE shows elevated JVP. Crackles
are heard over the lower half of her lungs. A grade 4/6 apical systolic murmur is heard.
Mechanism of worsening pain? -Answer- Rupture of a papillary muscles
most important MI complications that occur within a 2-5 day span are papillary muscle
rupture and interventricular septum rupture.

Papillary muscle rupture leads to severe mitral regurgitation, heard as a systolic murmur
at the apex.
56 y/o - automobile crush injuries of both legs. 36 hours later he develops oliguria,
hyperkalemia, and increased BUN. Which is most likely cause of findings? -
AnswerRhabdomyolysis
Rhabdomyolysis can present looking like a kidney injury
The electrolyte findings are just like renal failure (Inc. K+, inc. PO4-, dec. Ca)
To differentiate between rhabdomyolysis and kidney injury, you check the urine to see if there
are any RBCs. In rhabdomyolysis there are no free RBCs in the urine

8-year-old boy - evaluated for ventricular systolic overload. ECG shows left axis deviation of
the main QRS vector (upward in lead 1 and downward in lead II). Which is most likely
explanation for patients condition? -Answer- Coarctation of the aorta leads to increased LV
overload causing LV hypertrophy and a L axis deviation.

30-year male - completed Hodgkin Lymphoma 2 months ago-calls physician on an increasingly


frequent basis. He takes temp several times per day. After evaluation he shows no evidence of
recurrence, he is increasingly fearful that the disease many not be detected. DX? -Answer-
Adjustment Disorder with anxiety
Emotional symptoms (eg, anxiety, depression) that occur within 3 months of an identifiable
psychosocial stressor (eg, divorce, illness) lasting < 6 months once the stressor has ended.

A 3-month old boy - 1 month history of lump on right side of his groin. Bulge increases when
patient crises. PE shows soft mass in the right groin that decreases easily. Which best
describes the relationship to the hernia sac? -Answer- Lateral to the inferior epigastric and
superior to inguinal ligament b. Inguinal hernias are usually reducible
c. indirect inguinal hernia. It enters internal inguinal ring lateral to inferior epigastric
vessels and is superior to the inguinal ligament.
d. failure of processus vaginalis to close (can form hydrocele).

A 72 y/o woman - dysphagia- surgical evaluation of a mass in posterior mediastinum. During


procedure, the thoracic duct is inadvertently damaged near the mass. The injury is most likely
to impair normal lymphatic return to which structures? -Answer- Right Kidney
The left upper extremity and breast are drained by the axillary lymph node.
The kidney is drained by the thoracic duct. The heart has its own lymph system going on
surrounding the heart
The sequence surrounding the first 2 exons of the human B globin shown (exons in bold
capital letters). The translation start codon is underlined. A mutation of G-A at position

355 is likely to lead to B-thalassemia by which mechanism? -Answer- Disruption of


normal splicing creation of a new 3' splice site

B-thalassemia is due to point mutations in splice sites and promoter sequences

mutation occurs within an intron (a gene segment which is transcribed [DNA->RNA] but not
translated). RNA splicing enzyme(s) grab RNA and "loop it"; an intron is cut out and
the exons on either side of the intron are adjoined, like this:
A 77-year-old woman - difficulty fastening buttons. Weakness in intrinsic muscles of hands and
loss of sensation in the little fingers. Causes of findings? -Answer- C7-T1
Foraminal Stenosis
Little finger = ulnar nerve
C8-T1 are the roots of the ulnar nerve, which is a branch of the medial cord. The ulnar nerve is
not found in the carpal tunnel

A 9 year old - girl - poor growth during past year - 3rd percentile for height and 10th
percentile for weight. PE shows no abnormalities. Visual field testing shows bitemporal
hemianopia. Lab show a growth hormone deficiency. An MRI of the brain shows a calcified
cystic mass in the suprasellar region. The tumor is most likely derived from? Answer-
Diverticulum of the roof of embryonic oral cavity craniopharyngioma.

Most common childhood supratentorial tumor. Derived from remnants of Rathke pouch (oral
ectoderm). Calcification is common. Cholesterol crystals found in "motor oil"-like fluid within
tumor.
A cystic suprasellar mass with calcifications and enhancement of the wall or solid
portions in a child or adolescent is almost always a craniopharyngioma.
A 50-year-old man - brought in by wife because of personality change and cognitive decline. He
used to be kind - now yelling all the time. He almost invested all family money in a startup /
scam. He has inappropriate cursing. A CT scan shows atrophy of the frontal lobes bilaterally.
DX? -Answer- Pick Disease
Fronto-temporal dementia characterized by personality change is usually Picks
A 1 -month old - 5 days of vomiting after feeding - vomiting is forceful - PE shows decreased
skin turgor. Which set of serum findings - most likely in this newborn? Answer- Na 132
K 3.2
Cl- 90
HCO3 37
hypochloremia, hypokalemia, hyponatremia, and metabolic alkalosis chronic
vomiting, you lose electrolytes and a lot of acid.
It triggers metabolic alkalosis which is why all the serum values are low (or on the lower end of
the normal range) except for bicarbonate.
66-year-old fatigue, SOBs, temp. X-ray of chest shows cavitary fluid level in right lower lobe of
lungs. Culture of sputum gram stain grows Klebsiella pneumonia. Symptoms resolve with
antibiotic treatment. Which is found in right lower lobe 6 months later? - Answer- Focal
pulmonary fibrosis
Visible air-fluid level = large lesion
Six months following resolution of symptoms you can expect healing in the form of a
scar; that is, fibrosis but only in a single spot. '
Fibrosis seen in chronic diseases such as pulmonary fibrosis is often responsible for
organ dysfunction and even organ failure
16 y/o - 4 day history of severe vaginal itching discharge and a yellow discharge. She uses
condoms inconsistently. Saline wet mount shows motile organisms. Causal organisms? -
Answer- Trichomonas vaginalis thin, yellow-green, malodorous, frothy discharge and vaginal
inflammation / itching. Lab findings: pH >4.5 and motile trichomonas

50 y/o discharge from both breast- History of HTN and T2DM - She takes hydrochlorothiazide.
PE shows a thin, milky discharge from both breasts. Which is responsible for this patient's
condition? -Answer- Drug Effect
three major causes of galactorrhea as nipple stimulation, prolactinoma of anterior pituitary,
and drugs (see 16.1 - Breast Pathology). Only drug effect is an answer choice for this question.

A 6-week-old girl - 6-day history of vomiting a small amount of milk 2 to 3 times daily. She is
average weight and height. PE shows no abnormalities. Which is most likely cause of this
patient's findings? -Answer- Immature Lower Esophageal Sphincter vomiting a small amount
No abnormalities, only some vomiting, looks well w/ no failure to thrive. Most likely immature
LES.
esophageal spasm tends to present with pain and dysphagia. common
disease that impacts babies

60 y/o - 6-week history of cough productive blood tinged sputum. He has a 6.6 lb weight gain.
Smoker 30 years. BP 165/101. A CT scan shows pulmonary lesion. Serum Sodium 120. Which
additional lab results most likely in this patient? -Answer- Urine NA:
Increased
Serum Osm: decreased
Urine osmolality > serum osmolality
SIADH
Excessive free water retention

Euvolemic hyponatremia with continued urinary Na+ excretion


A 38-year-old woman - sore throat. During exam, physician say "ah" - causes elevation of area
of tip of the arrow in photograph shown. Which nerve tested? -Answer- Vagus uvular deviation
would be testing for CN X palsy

CN X carries efferent to palate. CN IX carries afferent from palate.

Glossopharyngeal nerve function: - Taste and sensation from posterior 1/3 of tongue -
Swallowing - Salivation (parotid gland) - Monitoring carotid body and sinus chemo- and
baroreceptors - Elevation of pharynx/larynx (stylopharyngeus)
African American - hyperpigmented papules, cheeks, jawline, neck - cause? -Answer-
Pseudofolliculitis Barbae
34 years - HIV positive - severe abdominal pain for 12 hours. He has been receiving
antiretroviral therapy for past 2 weeks. Prior to starting meds, CD4+ 35/min. A CT scan of the
abdomen shows enlargement of lymph nodes. Improved function of which cells is required for
the following? -Answer- CD4+ T Lymphocytes
CD4+ cells activate B-cells which form follicles and cause enlargement of lymph nodes.
Therefore, in an AIDS patient, to enlarge the lymph nodes, the CD4+ dysfunction must be
resolved.

25 y/o - 42 weeks' gestation - admitted for failure to begin spontaneous labor. Infusion of
oxytocin is initiated. Activation of which signaling pathway mediates therapeutic
effect for this patient? -Answer- Phosphoinositide hydrolysis Oxytocin
uses IP3 signaling pathway.
GnRH, Oxytocin, ADH (V1-receptor), TRH, Histamine (H1-receptor), Angiotensin II, Gastrin.
FA mnemonic: "GOAT HAG"

A 35 year old woman - HTN - BP 153/106 mm Hg. A bruit is heard lateral to umbilicus on the
right side. Which diagnostic test is most appropriate to evaluate HTN? -AnswerMR angiography
of the renal arteries

A 37-year-old nurse stuck with a needle used to obtain blood for HIV positive. Receives
antiretroviral therapy. 4 weeks later develop anemia and neutropenia. Which is the MOA of
antiretroviral responsible for anemia and neutropenia? -Answer- reverse transcriptase NRTI's
are associated with possible side effects of anemia, granulocytopenia, and myelosuppression.

A 70 y/o man dies of coronary artery disease - He had cerebral infarction 8 years ago. A photo
of brain stem is shown. Which neuro deficits most likely present after his cerebral infarction? -
Answer- Spastic Hemiparesis on the right
Left crus cerebri was damaged (see what it should normally look like below). This contains the
corticospinal tract. Since the corticospinal tract decusates at the medulla, below the midbrain
section we're looking at, you would see Contralateral (Right) Spastic Hemiparesis
A 16-year-old student - uncontrollable sleepiness, falling asleep several times a day while sitting
in class. The slightest noise or touch is enough to awaken him. Which characterizes the onset of
his sleep at night?term-163 -Answer- Direct transition from wakefulness to REM sleep
"Narcolepsy can be conceptualized as a disorder of sleep-wake control in which elements of
sleep
direct transition from wakefulness to REM sleep. Basically instead of going through the
early stages and gradually falling into a deep sleep, you just suddenly go from being
awake to being in a deep sleep.
A study is conducted to identify healthy women at risk of cardiovascular disease. Because
atherosclerosis is inflammatory, the concentration of c-reactive protein is increased. Which is
likely site of CRP biosynthesis? -Answer- Liver
CRP is an acute phase reactant, these are synthesized primarily by the liver location (Acute
Phase Reactant)

Investigator looking at effectiveness in TX (substance X) for Sjogren Syndrome. Substance X


produces 2X saliva as placebo. Placebo group contains half as much sodium chloride as that
from Substance X. Bicarb is the same in both groups. Which explains these findings? -Answer-
Ductal ion reabsorption more efficient at low flow rates
Ductal cells reabsorption is hindered at high flow rates
At low flow = High concentration of potassium; low concentrations of sodium, bicarb, &
chloride

at high flow = low concentration of potassium; high concentrations of sodium, bicarb, &
chloride
20 y/o brought to office by mother. Pt is hearing voices and bizarre behavior for 6 months. He
used to be a good student, and now he is failing his classes. Pt appears unkept, distracted.
Mother thinks he is scaring the family because he is hearing voices. Which initial response is
most appropriate? -Answer- How frightening- Do you have an idea what might be causing the
problem?
A 53-year-old man - 4-hour history of severer headache, anxiety, sweating, palpitations.
He has a 2 year history of HTN. PE shows diaphoresis. Which is the effects of
propranolol in this patient? -Answer- Increase TPR and Decrease CO
non-selective Beta blocker
CO - HR decrease-

compensatory increase in TPR


Pilot crashed plane - 2 weeks ago in Alaska wilderness. Pilot has minor injuries - lost
considerable weight. Pilot found good water supply but couldn't find food. The pilot most likely
has which condition? -Answer- Negative Nitrogen balance
Negative nitrogen balance is associated with burns, serious tissue injuries, fevers,
hyperthyroidism, wasting diseases, and during periods of fasting
Positive nitrogen balance is associated with periods of growth, hypothyroidism, tissue repair,
and pregnancy. This means that the intake of nitrogen into the body is greater than the loss of
nitrogen from the body, so there is an increase in the total body pool of protein.

A 45-year-old man - intermittent bloody diarrhea and abdominal pain. Rectal biopsy shows
inflammatory bowel disease. A PT with IBS (diarrhea and abdominal pain) Treated with
monoclonal antibody is begun. The antibody is most likely directed against which component? -
Answer- Tumor Necrosis Factor
infliximab and adalimumabStop
inflammatory events

37 y/o - HIV positive- needs Pain relief numbness in legs, pain, tingling in his feet. He is receiving
antiretroviral therapy. On PE soles of the feet are tender to palpation. Sensation to pinprick is
decreased over his calves and feet. Which drug is most appropriate for long term relief of his
pain? -Answer- Nortriptyline
Inhibit 5-HT and NE reuptake
Peripheral neuropathy, chronic neuropathic pain, migraine prophylaxis

A 35-year-old man - 6 month history of burning abdominal pain occurs 1 to 2 hours after he
eats. He has black stools for 2 days. Antacids and H-2 blockers are not effective. He is sweating
profusely and has lightheadedness when he stands. PE shows epigastric tenderness. A CT scan
of the abdomen shows a 1-cm mass in the pancreas.
Immunohistochemical labeling - most likely to involve the use of antibodies directed to which
substance? -Answer- Gastrin
Zollinger Ellison Tumor - GASTRIN SECRETING

47 y/o female- 6-month history of severe cough and shortness of breath. She has no fever,
chills, or muscle aches. Nonsmoker works as a librarian. She lives alone with 4 parakeets. She is
a knitter and can spin wool. A CXR - shows reticulogranular changes.
Examination shows interstitial infiltrates of lymphoocytes and granulomas. Cause? - Answer-
Contact with Parakeets
Chlamydophila psittaci
Hypersensitivity pneumonitis—mixed type III/IV hypersensitivity

56 y/o - long history of chronic back pain undergoes operative placement of electrode in his
midbrain for pain management. When implant is activated, pain decreases.
Administration of which substances would most likely result in the return of the patient's pain?
-Answer- Naloxone
opioid antagonist - block mu receptors would lead to increase in mediators that induce pain.
A 55-year-old man - severe emphysema comes to physician for evaluation of SOBFindings on
lung exam? -Answer- Percussion Hyperressonant
Tactile Fremitus Decreased
Expiratory Phase Prolonged

37 y/o healthy woman - 6 month history of fatigue - She has 18-kg weight gain during this
period. Round face, truncal obesity, purple striae. Plasma ACTH low < 5 (normal < 120). Serum
cortisol does not become suppressed at low or high dexamethasone doses. Which is is most
likely cause of findings? -Answer- Tumor of adrenal Zona
Fasciculate
i. SALT, SUGAR, SEX ii.
SUGAR = Cortisol

A previously healthy 16-year-old - 2 days history of thick, yellow vaginal discharge and burning
with urination. She does not want her parents to know that she is sexually active. Gram stain
shows gram-negative diplococci. Testing for Neisseria and Chlamydia is ordered. Which is the
most appropriate course of action? -Answer- Prescribe antibiotics without notifying parents

A 27 y/o man - Weakness in Left Hand for 3 weeks. Professional bicyclist. He rides bicycle 8-
10 hours per day. PE shows mild weakness with abduction and adduction of fingers. He is
unable to hold a piece of paper between two fingers. Sensation is intact. Cause of findings is
compression of which structures? -Answer- Deep branch of ulnar nerve at the hook of
hamate
Guyon Canal - associated with Ulnar Nerve - intrinsic hand muscles handlebars
Thumb Opposition - MEDIAN NERVE

A 27 y/o man - Weakness in Left Hand for 3 weeks. Professional bicyclist. He rides bicycle 8-
10 hours per day. PE shows mild weakness with abduction and adduction of fingers. He is
unable to hold a piece of paper between two fingers. Sensation is intact. Cause of findings is
compression of which structures? -Answer- Deep branch of ulnar nerve at the hook of
hamate
intrinsic hand muscles
Guyon Canal - associated with Ulnar Nerve - handlebars
Thumb Opposition - MEDIAN NERVE
12. A 35-year-old man - given cyclosporine following a liver transplant. Which describes
the utility of this drug in this situation? -Answer- Decreased synthesis of interleukins
calcineurin inhibitor, which aims to decrease IL-2

23 y/o - ASD repair 3 months ago - PE shows increase in size of scar. An abnormality is
most likely cause of these findings? -Answer- Collagen Synthesis
keloid

type 3 collagen is first synthesized and then degraded by Matrix metalloproteinase

A 60 year old man - semitransparent nodule on his nose. Exam of tissues shows small
basophilic cells forming tumor islands by clear spaces and extending across the
basement membrane. DX? -Answer- Basal Cell Carcinoma
peripheral palisading, pink pearly, waxy, pink
12. 55 y/o previously healthy man diagnosed with HTN. He has not received therapy. Lab
studies show mild hypokalemia, metabolic Alkalosis, increased plasma renin and aldosterone.
Following administration of ACE-inhibitor, plasma renin increases further.
Most likely cause of HTN? -Answer- Renal Artery Stenosis
ACE-I likely Renal Artery Stenosis
Vasoconstriction of efferent
65-year-old dies 6 months after onset of headaches - Brain autopsy shown in the
photograph. Which is likely cell origin of this neoplasm?? -Answer- Meningeal
Meningioma- parasagittal region -
23-year-old with significant blood loss from MVC - She is given 3 units of blood and has an
uneventful recovery. 4 weeks later she becomes jaundiced. Lab show anemia and
reticulocytosis. Which is most useful in establishing the diagnosis? -Answer- Direct
Antiglobulin Test
Hemolysis due to transfused blood
Delayed onset of hemolytic reaction

15 y/o - 1-day history of a rash on her face, arms, and legs. She noticed rash on sunny day at the
beach. She is receiving ABX treatment for ACNE. PE shows diffuse erythema and mild edema
over the face and extremities. Which ABX cause findings? -Answer- Doxycycline
Bind 30s subunit
BUY AT - 30S--- CELLL @ 50S

5-year-old boy - DX with genetic defect - inability to reabsorb in the proximal tubule.
Serum studies are likely to show which of the following findings at this time? -Answer-
Hypophosphatemia
b. 85% of Phosphate absorbed in PCT
c. 25% Na reabsorbed in TAL

78 y/o dies of CHF- Exam shows passive congestion of lungs, hypoxic changes, hydropic changes
im the renal tubular epithelial cells. Which of the following cause
hydropic change? -Answer- Failure of Na+/K+ pump
ATP STOPS WORKING
Hydropic Changes - one of the early signs of cellular degeneration in response to injury that
results in accumulation of water in the cell.
Hypoxia decreased in aerobic respiration
Cellular injury causes swelling
25-year-old - - collapses in ICU where she works as a nurse. Coworkers report she felt fine all
morning. Shortly after lunch she became pale and diaphoretic and then collapsed. She has a
history of episodes of hypoglycemia without apparent precipitants. She is unresponsive and low
blood 2- concentration, given bolus of 50% dextrose. Several episodes of confusion. Glucose is
less than 20 - bolus of 50% dextrose is administered. Lab studies show.
Glucose 32
C-peptide <0.075
Proinsulin < 1 pmol/L

Which of the following is most likely diagnosis? -Answer- Factitious Disorder


EXOGENOUS INSULIN
Low C-peptide, low proinsulin, high plasma free insulin and low glucose, in a nurse Endogenous
Insulin - increased C-Peptide

An Investigator is studying patients with West Nile Virus infection. During a 5-year period, data
are collected on 25 patients. Demographic information on the patients is reported (age, sex,
ethnicity) as well as information on the source of infection. What is the study design? -Answer-
Case Series
group or series of case reports involving patients who were given similar treatment NO case
controls

Double blind study - many adult participants non adherent- In accordance with intention to
treat, how should data pertaining to non-adhering be treated? -Answer- Analyze all
non-adherent participants as part of the group they were randomized too
Intention to treat - none of patients are excluded
Per Protocol - only those who complete the entire protocol are counted

45-year-old woman - intubated and mechanically ventilated develops fungemia with Candida
albicans during a prolonged hospital stay. Treated with Caspsofungin. What features of the
causal organism will most likely be targeted by this drug? -Answer- Bglucan Carbohydrates in
Cell Walls
Echinocandins - cell wall synthesis
24 y/o man - 8-hour history of abdominal pain, nausea, and vomiting. Family history of
pancreatitis and hyperlipidemia. Triglycerides 3500 mg/dl and lipase of 400 U/L
(N<280). Serum appears milky. Most likely cause is a deficiency of which of the
following? -Answer- Lipoprotein lipase deficiency
Pt dyslipidemia + increased chylomicrons
A 58-year-old man - severe Chest Pain Radiating to back for 30 minutes. X-rays shows do not
show a widening Aorta. Aortic angiography shows an extra lumen in the media of the proximal
aorta. What is most likely cause of finding? -Answer- Hypertension cause aortic dissection False
lumen p. 301
62 y/o mild cough, purulent sputum for 2 weeks. She smoked 2 packs for 42 years. She drinks 3
to 6 beers. PE shows, swelling of the in fingers, arthritis of joints, and clubbing of the digits.
What is the most likely underlying disease? -Answer- Bronchogenic carcinoma

Immediately after a cerebral infarction, a 62 y/o right handed man has decreased fluency
but his comprehension is normal. Pt most likely has lesion in which of the following
areas? -Answer- Area Labeled A - BROCAS areas b. fluency decreased, comprehension
intact
c. Wernicke area - poor comprehension, fluent speech

55 y/o - 2 month history of decreased appetite. 20lb weight loss and an intermittent rash. PE
shows necrolytic migratory erythema over the axilla and groin. Glucose 280 and Glucagon 1500
(N- 20-100). A CT scan shows mass at head of pancreas. Which is most likely occurring in liver? -
Answer- Partial Oxidation of Fatty acids
Glucagon upregulates - CAT1 and CAT2 are important enzymes of fatty acid beta oxidation.
Glucagonoma: migratory erythema
Dermatitis (necrolytic migratory erythema), Diabetes (hyperglycemia), DVT, Declining
weight, Depression
28 y/o - 32 weeks gestation, 4 day history of fever and back pain. PE shows costophrenic
tenderness, photomicrograph shown represents her disease. What mechanism is likely cause? -
Answer- Obstructive nephropathy
pregnancy dilation of the upper urinary tract can occur due to compression of the
ureters by the growing fetus/uterus and the linea terminalis postrenal azotemia

65 y/o woman - 25 year history of chronic alcoholism - vomiting blood—She eats very little,
Exam of her legs show -perifollicular hyperkeratosis and hemorrhage. Abnormally decreased
function of which metabolic pathways is most likely in this patient? -Answer-
Hydroxylation of proline
Vit. C deficiency- curvy features swollen gums, easy bruising, petechiae, perifollicular and
subperiosteal hemorrhages.

16 y/o girl with cystic fibrosus - 3-week history of numbness, tingling of her arms and legs, and
difficulty walking. She has not adhered to her medication region for 6 months. PE shows
bilateral weakness and decreased deep tendon reflexes in the upper and lower extremities. She
walks with ataxic gait. Deficiency in which of the following? Answer- Vitamin E
fat soluble (neurologic with ataxia)
A 33-year-old - HIV infection - generalized tonic-clonic seizures. 2 month history of
headaches. CD4 count is 22. CT shows 3-cm lesion in right cerebral cortex. Serologic
studies - positive for IgG antibody to Toxoplasmosis Gondii. Tx with pyrimethamine and
sulfadiazine is initiated. During next 2 weeks, 3 more seizures. CT shows lesion
increased to 3.5cm. Cause of mass? -Answer- Non-Hodgkin Lymphoma
Single enhancing lesion

Toxoplasmosis - multiple lesions - improved by TMP-xMX


A 45-year-old - man - surgical procedure - Which maneuver by the anesthesiologist would result
in increased intracranial pressure? -Answer- Decreased respiratory rate increased CO2 - cause
vasodilation

62 y/o - physician follow-up - 2 weeks ago discharged after acute MI. Smoked 2 packs a day.
Diet mostly cured meats and fast food. He does not exercise. I know I need to make some
changes in how I live. I just don't have willpower to quit smoking. Which best describes patients
stage of behavioral change? -Answer- Contemplation -
patient acknowledges problem but unwilling to make change
Precontemplation - patient denies existence of problem
A 30-year-old woman - develop serum sickness following parenteral administration of
penicillin. Plasma sample is found to contain a mediator that stimulates histamine
release from mast cells. Which mediators most likely found in this patient's plasma? -
Answer- C3a
b. mast cells activated by C3a and C5a p. 400
45 year old man - counseling prior to bariatric Surgery - BMI 40- PE shows no abnormalities, In
addition to recommending that the patient follow a balanced diet, it is most appropriate to
avoid ingesting excessive amounts of which? -Answer- Starchy foods
cause dumping syndrome- avoid simple carbs
Have small portions

A 32-year old man - 3-month history of swelling and tenderness of both breasts. He is receiving
thyroid hormone and corticosteroid therapy since surgical removal of pituitary adenoma 2
years ago. Began HCG injections for months ago. Which is most likely
binding site of hCG causing gynecomastia? -Answer- Tissue: Testicle
Effect: estradiol production
B-HCG and LH,FSH,TSH share same alpha subunit, so HCG can activate those receptors if its in
high enough quantity.
Activating LH receptor will lead to more Testosterone from the Leydig cells. More testosterone
can lead to more estrogen formation via aromatase.
A 48-year-old man - overdose on ferrous sulfate. Na and K in normal range. What
happens to bicarb and cl serum concentrations? -Answer- HCO3 8
Cl 96

IRON OVERDOSE- high metabolic acidosis


High anion gap metabolic acidosis due to Iron excess (the I in MUDPILES). Serum anion gap =
(Na) - [(HCO3 + Cl)]. Chloride levels low;HCO3 low, which results in the high anion gap.

A 61-year-old man - erectile dysfunction due to spinal cord injury at L-2. Sildenafil used to
correct ED by acting at which of the following labeled structures (Image Shown)? - Answer-
corpus cavernosum
Sildenafil increases blood flow to the penis by dilating the corpus cavernosum (increased NO via
inhibition of PDE5 --> cGMP --> smooth muscle relaxation).
A 32-year-old woman -- confused and lost consciousness - fell from ladder an struck the right
side of her head on the edge of her desk. She felt well enough to continue working. PE shows
edema on right anterior temple. Right pupil dilated. Left pupil normal. What is
DX? -Answer- Arterial laceration

epidural hematoma Meningeal Artery

Pts bumps head, gets back up - severe issues/ dies 6 hours later -- = epidural
hematoma

middle meningeal artery laceration => epidural hematoma. Epidural hematomas are
known to cause transtentorial herniations and CN III palsy
Branch of maxillary artery
A 12-year-old boy - 3 episodes of meningococcal bacteremia during past 4 years. He has had
not other significant infections. The most likely cause of the recurrent meningococcal infections
is a deficiency in which of the following? -Answer- Terminal
Component of Complement MAC Complex (C5-C9) Deficiencies increase
susceptibility to recurrent Neisseria bacteremia.
20 y/o man - states that food had lost its flavor ever since he fell down flight of stairs
and hit his head. PE shows no abnormalities. Neuro exam is most likely to show
damage to which cranial nerve? -Answer- Olfactory
A 33-year-old - 3 month history of pain in right thigh. No history of trauma. PE shows mild
swelling but no erythema. MRI shows tumor in anterolateral muscle. Which muscle is infiltrated
with the tumor? -Answer- Rectus Femoris

51-year-old- develops diaphoresis, tachycardia - 24 hours after undergoing abdominal


operation -BP 155/100 - 2 hours late he has generalized tonic-clonic seizures. Which is
responsible for the adverse effect seen in the patient? -Answer- alcohol withdrawal

64-year-old - surgical repair of abdominal aortic aneurysm. During the repair the testicular
artery is ligated. Anastomotic supply from which of the following arteries will maintain
adequate arterial supply to the left testis in this patient? -Answer- Artery of the ductus deferens
A 33 year old woman - 34 weeks gestation - grade 2/6 systolic ejection murmur heard at
the second intercostal space. The remainder of the PE is normal. Best explanation for
this finding is an increase in which? -Answer- Stroke Volume
SV = EDV - ES

EDV increases in pregnancy due to an increase in plasma volume

"Blood flow to various organs increases during pregnancy to meet the increased
metabolic needs of tissues. Thus, venous return and cardiac output increases
dramatically during pregnancy. Cardiac output gradually increases during the first 2
trimesters with the largest increase occurring by 16 weeks of gestation
A clinical study - evaluate the association of caffeine consumption and pancreatic cancer. In the
study alpha is set at 0.05 and beta at .10. Which of the following best describes the likelihood of
missing associations between caffeine consumption and pancreatic cancer in this study? -
Answer- 10%
Type II beta error
"likelihood of missing an association"
represented by beta. = Power =
1 - beta.
Type I = alpha error

A 10-year-old boy - adopted from Democratic Republic of Congo. PE shows 0.5-, firm,
nontender nodules, two over the right iliac crest, one on the left thigh, and one on the left
knee. Exam of the skin from one of the nodules shows microfilariae. Which is the likely vector in
this patient? -Answer- Black Fly
Nodules show microfilariae
Black Flies, Black Skin Nodules, Black Sight
Onchocerca Volvulus via female blackfly. Black flies, black skin nodules, "black sight"
(aka Blindness). Question stem here describes the black nodules seen in Onchocerca.
A new screening test for color cancer is done on 86 patients with colon cancer and 124
who do not:
Results are shown:
Present Absent
Positive: 67 6
Negative: 19 118

Specificity? -Answer- 95%

A 56 year old man - 3 year history of progressive memory loss followed by aphasia, visuospatial
disorientation, and inappropriate behavior. Father died in nursing home at age 65 from
progressive dementia. At autopsy, brain shows cerebral atrophy, neurofibrillary tangles,
neuritic plaques, amd marked neuronal loss. Mutation in gene encoding for which of the
following molecules? -Answer- Presenilin
early inherited cases of Alzheimer's dementia are associated with mutations in
presenilin-1/presenilin-2, as well as Down's syndrome associated with familial
alzheimers disease

At 2:00am - 32 year old man with T2DM has a decrease in serum glucose concentration to 37
mg/dl. In response to the hypoglycemia, cortisol secreted by the adrenal cortex induces the
synthesis of which of the following in the adrenal medulla? -Answer- Phenylethanolamine N-
Methyltransferase 2:am cortisol secreted by adrenal cortex induces synthesis - catecholamine
synthesis pathway from medulla
At 2:00am - 32 year old man with T2DM has a decrease in serum glucose concentration
to 37 mg/dl. In response to the hypoglycemia, cortisol secreted by the adrenal cortex
induces the synthesis of which of the following in the adrenal medulla? -Answer-
Phenylethanolamine N- Methyltransferase

b. 2:am cortisol secreted by adrenal cortex induces synthesis - catecholamine synthesis


pathway

c. NE is converted to EPI via PNMT, which is induced by cortisol


53-year-old woman - fainting precipitated by exercise and intermittent substernal chest pain -
relieved by rest. Her blood pressure is 120/80. Prominent left ventricular impulse. A late
peaking harsh, midsystolic murmur is heard at second right intercostal space with radiation to
the carotid arteries. Which is best explanation of syncope? -Answer- Fixed
Cardiac Output in spite of increased demand
Aortic Stenosis - unable to push through valve - Syncope, Angina, Left Heart Failure
12 year old boy- persistent rash around his nose and mouth for 4 months. Decreased
appetite and unintentional weight loss. Rash began after he started a new school. Grades
are falling. Rash mild redness and flaking. PE shows conjunctival injection, eczematous
eruption on the nasolabial folds, friable with gold stippling. Cause of condition? -Answer-
inhalant abuse Intellectual decline

A 25-year-old woman - overdose of barbiturates and is found unresponsive several hours later
by her husband. PT is comatose and undergoes endotracheal intubation. 3 weeks later she has
no purposeful response to stimuli. The patient signed a living will - consistent with its directives
that mechanical ventilation be discontinued. Appropriate next step? -Answer- extubate the
patient and discontinue mechanical ventilation, make no attempt at resuscitation

A 63-year-old woman - repair of leaking berry aneurysm in the circle of Willis. Two days later, a
CT scan of the head shows widening of the subarachnoid space. What change in CSF causes the
finding? -Answer- Decreased movement through arachnoid villi arachnoid villi drains the CSF
from the subarachnoid space to the venous system; if this part becomes defective then you can
imagine all that CSF now building up in the subarachnoid space.
acute SAH can lead to decreased absorption and movement via arachnoid villi
Choroid plexus - make
Arachnoid - Absorbs
A 68 y/o - 9 month history of abdominal pain after meals. 24lb weight loss. She avoids eating
because of cramping pain that lasts 1 hour after meals. PE shows thin body frame and a soft
nontender abdomen. BP 160/90 A CT scan shows a 3-cm ectatic
(dialated) aorta. Which pairs of arteries is most likely involved in this patient? -Answerinferior
mesenteric and superior mesenteric Mesenteric cause post prandial pain - AAA compression
of duodenum at 3rd
Chronic Mesenteric Ischemia aka intestinal angina. abdominal aortic aneurysm. Generally, they
tend to occur below the renal branches so I went with the two closest branches- SMA

In a study of antibiotic resistance, strain of Escherichia Coli to ampicillin but sensitive to


streptomycin is cultured with a strain of Salmonella Enteritidis sensitive to ampicillin but
resistant to streptomycin. After 4 hours of cocultivation, Salmonella Enteritidis are resistant
to both antibiotics. Mechanism of ampicillin resistance by the Salmonella Enteritidis? -
Answer- only cookie are independently associated with E Coli OR >1 indicates increased
occurrence of event.

38 year old woman gravida 4m, para 4 - 6 month history of urinary incontinence when sneezing
and coughing - Leakage occurs when patient performs valsalva - Instructed to do kegel exercises
- which unaffected by kegel exercise to strengthen muscles of perineum. Which muscle are
unaffected by the exercise? -Answer- Internal Anal Sphincter
Internal anal sphincter = parasympathetic/sympathetic control external anal
sphincter = skeletal muscle and controlled by pudendal nerve
26-year-old man - 40 minutes after MVC - BP is 90/60. Systolic blood pressure 20 mm
Hg with inspiration. PE shows JVD. There is distant heart sounds. DX? -Answer-
Pericardial Tamponade
Hypotension Distant Heart Sounds, Distended Jugular Vein,
Electrical Alternans

pulsus paradoxus, a sign in which blood pressure decreases drastically during


inspiration. Pulsus paradoxus is a classic sign of pericardial tamponade
23-year-old woman - SOBs for 2 weeks, PE Shows no abnormalities - Na+ 135, HCO3
15, Cl 110- Most likely diagnosis? -Answer- Renal Tubular Acidosis
normal anion gap (10) Na- Cl - HCO3= 10
High Anion Gap needs to be greater than
12 HARDASS
A 30-year-old woman - chronic renal failure and normocytic anemia - Following recombinant
EPO, her hemoglobin concentration increases due to increased activity of 5-aminolevulinic
acid synthase in erythrocytes. This enzyme regulates the reaction involving the condensation
of which compounds? -Answer- Glycine and Succinyl CoA substrates used the first step in
heme synthesis

38 year old woman gravida 4m, para 4 - 6 month history of urinary incontinence when sneezing
and coughing - Leakage occurs when patient performs valsalva - Instructed to do kegel exercises
- which unaffected by kegel exercise to strengthen muscles of perineum. Which muscle are
unaffected by the exercise? -Answer- Internal Anal Sphincter
Not under voluntary control
26-year-old man - 40 minutes after MVC - BP is 90/60. Systolic blood pressure 20 mm
Hg with inspiration. PE shows JVD. There is distant heart sounds. DX? -Answer-
Pericardial Tamponade

Hypotension Distant Heart Sounds, Distended Jugular Vein


A 45 year old man - hypercholesterolemia (LDL 260 mg/dl) - HMG CoA reductase inhibitor is
prescribed. Mechanism of action of treatment? -Answer- Increased Synthesis of LDL Receptors
statins -
HMG CoA Reductase inhibitors prevent the liver from synthesizing its own cholesterol. LDL
receptor expression in order to take cholesterol from the blood

22-year-old woman - nausea and vomiting - 6-month history of headaches. Neuro exam shows
marked vertical gaze palsy - impaired accommodation. Horizontal gaze is normal. MRI of the
brain is likely to show a tumor at which location? -Answer- Pineal Gland
VERTICAL GAZE PALSY - PARINAUD Syndrome
- stroke hydrocephalus, pinelomas
Horizontal Gaze intact - Pontine Paramedian Reticular Formation

An investigator is conducting a study of a novel protein in an experimental animal model.


The protein interferes with desmosomes. Which cell junctions of the skin are affected? -
Answer- Basal Keratinocyte - Suprabasal Keratinocyte
Pemphigus Vulgaris. Separation of suprabasilar epidermis, intact basal keratinocytes Bullous
pemphigoid--> includes separation of basal layer
19-year-old - weakness of the muscles of his neck and extremities - Bladder and bowel function
is normal. Position sense is impaired. DX? -Answer- Polyneuropathy A condition involves
damage to multiple peripheral nerve fibers. Patients typically present with symmetric distal
sensory loss or a burning sensation associated with motor weakness
Vitamin B6 deficiency
Charcot-Marie-Tooth
Guillain-Barré

Forty of 100 attendees at a company party develop explosive watery diarrhea followed by
abdominal cramps and vomiting 12 to 48 hours after the party. All affected people consumed
raw oysters - No other associations between the consumption of specific foods and symptoms.
Causal organism? -Answer- Vibrio Parahaemolyticus associated with consumption of
undercooked seafood like shellfish.

30-year-old man - history of fever, chills, severe headache. He immigrated from Honduras. A
photomicrograph of a peripheral blood smear is shown. (Schuffner Stipling) The patients
overall condition is most likely caused by an infectious agent with which features? -Answer-
Formation of Hypnozoites plasmodium vivax- Schuffner Stipling
chloroquine = clear the infection in the bloodstream,
Treatment with primaquin to clear the hypnozoites,
63-year-old flank pain, tachycardia, hypotension while receiving a blood transfusion in
the intensive care unit. Her urine is wine colored. Which is the cause these findings? -
Answer- ABO Incompatibility

TYPE II hypersensitivity - delayed reaction


Rh Incompatibility - Jaundice
Extravascular Hemolysis -
55-year-old woman - change in behavior over last 2 days - Yesterday she didn't recognize a
picture of her own mother. Neuro exam shows inability to recognize objects unless she touches
them or hears the sound that they make. Lesion in which of the following arteries? -Answer-
Posterior Cerebral
PCA stroke can cause "prosopagnosia" which is the inability to recognize familiar faces. Caused
by bilateral lesions of visual association areas, which are situated in the inferior occipitotemporal
cortex (fusiform gyrus).

A 90-year-old man - 1-week history of constipation and abdominal swelling. He is afebrile. 4-cm
mass is palpated in the right groin. The mass is not reducible, but moderately increased
tympanic bowel sounds are audible over it. Diagnosis? -Answerincarcerated inguinal hernia
protrusion of peritoneum through an opening
usually at a site of weakness contents at risk of
incarceration
A 27 year old man - spinal cord transection at C8 in a skiing accident. In which of the
following situations is he most likely to achieve and sustain an erection? -Answer-
Penile Stimulation
S2-S4

13-year-old with Type 1 Diabetes - has poor glucose control for past 2 months.
Previously the T1DM was controlled. Now family is fighting at meals. Parents argue in
physician's office. Which is the most appropriate next step? -Answer- Discuss further the
impact of the patient's illness on the family getting heated and therefore doc should try to
calm things down before proceeding

ARBS -Answer- -sartan


Renin Increased
ANG I Increased
ANG II Increased
Aldosterone Decreased
A 71-year old woman with non-hodgkin lymphoma is diagnosed with lymphomatous meningitis.
Serum concentrations is 2.3. A course of methotrexate is planned. IV administration of which
adjuvant agent is most appropriate? -Answer- Leucovorin common chemoprotectant used to
avoid Methotrexate toxicities.

A 40-year-old man - 6 months of constipation and weakness. Family history of colorectal


carcinoma in multiple members of both sexes. MCV is 73. Photograph of colon. Analysis
shows mutations in the MHS2 gene. Diagnosis? -Answer- HNPCC syndrome MHS2 gene
mutation
Lynch syndrome AKA hereditary non-polyposis colorectal carcinoma (HNPCC),

30-year-old woman - 2 week history of right flank pain - BP is 168/98. PE shows a 7-cm mass in
right side of the abdomen. CT shows mass in the retroperitoneal space compressing the renal
artery. If perfusion pressure is decreased, but her GFR and RBF remain unchanged, which
mechanism mediates the autoregulation of the renal arterioles in this patient? -Answer-
Decreased afferent arteriolar resistance myogenic mechanism
autoregulation- afferent arteriole controls blood flow
45-year-old man -picture with 10 years of painful ulcers that recur every few months. No
other problems. Photograph shown. Which is likely cause of patient condition?
-Answer-

Aphthous Ulcers
caused by stress
canker sores
40-year-old woman - 25-year history of T1DM has serum concentration of 2.5 mg/dl. - A
creatine clearance test is ordered to determine the GFR. This test is limited to Inulin Clearance
Test, because of the following properties of creatinine? -Answer- Secreted by the proximal
Tubule
Creatinine clearance slightly overestimates GFR because creatinine is moderately secreted by
PCT
40 from Kenya - 3 week history of diarrhea. He recalls zero of three objects after 5
minutes. - Photomicrograph of the head shown (dermatitis) - DX of vitamin deficiency is
made. Vit deficiency is made. The deficient cofactor contains which components? Answer-
Adenine
Pellagra Diarrhea, Dementia, Dermatitis
Vitamin B3 deficiency - Co-Factor for NAD
Niacin combines with Adenine to form B3

47-year-old - abdominal enlargement and rectal bleeding. PE shows internal hemorrhoids.


Increased pressure in which of the following is most likely to cause the hemorrhoids? -Answer-
Superior Rectal Vein
Superior rectal offer IMA
External Hemorroid - Inferior Rectal of Internal pudendal

A 24 y/o woman - diffuse toxic goiter (graves disease) - partial thyroidectomy - months later she
develops muscle cramps, tetany, hypocalcemia. Which labs findings? -
Answer- phosphate increased
parathyroid decreased 25-
Hydroxyvitamin - normal
accidental removal of the PTH glands during thyroidectomy ⇒↓ PTH
↑ removal of Ca²⁺ and Phophate from bone --in kidneys: ↑ Ca²⁺ reabsorption and ↓ PO₄³⁻
reabsorption --↑ conversion of 25, Hydroxyvitamin D to 1,25 Hydroxyvitamin D (Calcitriol - active
form) via ↑ activity of 1-a Hydroxylase deficiency

. 72-year-old - coronary artery disease - 2 -month history of angina - 5 months ago underwent -
stent placement for significant stenoses - symptom free for 3 months - ECG at rest shows no
abnormalities - ST segment changes in II, III, AvF (inferior leads). Cause of recurrent angina? -
Answer- Neointima Formation in the right coronary stent
Stents cause neointima growth - restenosis
Stent thrombosis vs re-stenosis. Stent thrombosis is an acute occlusion of a
coronary artery stent, which often results in acute coronary syndrome.

A 45 year old - sudden onset of severe pain in the left flank - awakened from sleep. Reports
that the pain - now principally located in the scrotum. PE shows no palpable masses in
abdomen. No rebound tenderness - Cause of pain? -Answer- Renal Calculi in the Left Ureter
kidney stone referred to the testicles. no mass in the scrotum- testicular torsion will have
that "bag of worms"

60 year old - cisplatin therapy for bladder cancer - develops paresthesia - Medication damaged
region labeled B - Damage to these cells would most likely cause which motor sign? -Answer-
Hyporeflexia
afferent arc of the muscle stretch reflex has to go through the dorsal rami and dorsal root
ganglia.

10-year-old girl - hair growth on face and increased muscle mass. PE shows clitoromegaly.
Pelvic exam shows a normal appearing vagina. Serum Studies:
LH Low
Testosterone
High
An unregulated
increase in
hormone
production is most
likely to cause
patient
hirsutism? -
Answer- Ovarian
Sertoli -Leydig
Cell
increased
testosterone
Ladies dig
testosterone
A 25 year old woman - lost consciousness at home - fever (103) and history of recurrent upper
urinary tract infections. BP is 82/48 - Diffuse crackles are heard over both lung fields. ABG
shows: pH. 7.18 PCo2 32
Po2 64
Blood culture grows gram negative rods - CXR - shows diffuse infiltrates- Most likely
cause of pulmonary symptoms? -Answer- Pulmonary Capillary Leakage
Fluid in lungs

Urosepsis- ARDS - diffuse bilateral infiltrates on chest radiography...


pneumonia/sepsis (ARDS?)
PO2 is low at 64.

pneumonia there is increased capillary leakage leading to pulmonary edema.


45-year-old - 3 day history of fever and muscle pain - He went hunting - ate polar bear meat.
Temp is 103. PE shows periorbital edema after and muscle tenderness. Which
tests most likely to establish DX? -Answer- Muscle Biopsy
Trichinella - larvae deposit in muscle
Muscle pain + periorbital edema is a classic presentation for trichonella spiralis. Best diagnosis
for this is a muscle biopsy, as the wormy likes to hangout within the muscles.

3-year-old - well child exam- flat purple lesions that measures 2.1 cm on her face Lesions
show cavernous channels. Which is most likely outcome of the lesion? - Answer-
Spontaneous Regression
CAVERNOUS VASCULAR CHANNELS
nevus simplex lesions regress within the first 2 years of life
1 week old newborn - poor feeding, vomiting, progressive lethargy - No congenital anomalies.
PE shows decreased muscle tone and poor responsiveness - reflexes are normal. pH 7.15, bicarb
low, high ammonia (10X) - Cause? -Answer- Organic Acid
Metabolism disorder
Organic acidemias most commonly present in infancy with poor feeding, vomiting, hypotonia, a
high anion gap metabolic acidosis, hepatomegaly, and seizures.
Male newborn - Baby is small for weight and height - head circumference is 31cm Eye
exam shows choriorentinitis - CT scan of the head shows Hydrocephalus and

intracranial calcifications. The newborn congenital infection was most likely acquired by
mother vis which mode of transmission? -Answer- Ingestion of Undercooked meat
Congenital Toxoplasmosis:

Triad intracranial calcifications, Chorioretinitis, hydrocephalus


68 year old woman - 2 hours after vomiting blood - 3 weeks ago began meds for vertebral
compression fracture with osteoporosis. PE shows epigastric tenderness Endoscopy shows
esophageal erosions. Which medication cause hematemesis? Answer- Aldendronate
bisphosphonates - causes esophagitis and osteonecrosis

In a study of antibiotic resistance, strain of Escherichia Coli to ampicillin but sensitive to


streptomycin is cultured with a strain of Salmonella Enteritidis sensitive to ampicillin but
resistant to streptomycin. After 4 hours of cocultivation, Salmonella Enteritidis are resistant
to both antibiotics. Mechanism of ampicillin resistance by the Salmonella Enteritidis? -
Answer- Conjugation plasmid gets transferred from conjugal bridge from one bacteria to
another

55-year-old smoke - comes to physician for a routine health maintenance - She never smoked
cigs. CXR shows 2-cm left pulmonary nodule (coin lesion). Microscopic exam shows nests of
cords of regular uniform round nuclei and rare mitoses. Cells are arranged in trabeculae, and
scattered rosettes. Electron shows dense-core, membranebound granules in the cytoplasm.
These cells resemble which type of respiratory tract cells? -Answer- Neuroendocrine Cell
nests of well-differentiated, "regular" cells is a carcinoid tumor. Additionally, rosettes are
histological features of carcinoid tumors

Kulchitsky cells = pulmonary neuroendocrine cells (PNEC).

A 63-year-old woman - repair of leaking berry aneurysm in the circle of Willis. Two days later, a
CT scan of the head shows widening of the subarachnoid space. What change in CSF causes the
finding? -Answer- Decreased movement through arachnoid villi SAH - communicating
hydrocephalus acute SAH can lead to decreased absorption and movement via arachnoid villi

A 68 y/o - 9 month history of abdominal pain after meals. 24lb weight loss. She avoids eating
because of cramping pain that lasts 1 hour after meals. PE shows thin body frame and a soft
nontender abdomen. BP 160/90 A CT scan shows a 3-cm ectatic
(dialated) aorta. Which pairs of arteries is most likely involved in this patient? -Answerinferior
mesenteric and superior mesenteric Mesenteric cause post prandial pain - AAA compression
of duodenum at 3rd

A full-term male newborn has lethargy, poor feeding, and vomiting after delivery. He appears
flaccid. Serum studies show increased concentration of ammonia and orotic acid. Which labeled
steps in the metabolic pathway is the most likely cause of these findings? -Answer- Area labeled
C.
Ornithine to Citrulline
Ornithine transcarbamolase deficiency.
Carbomoyl phosphate (CP) is shunted to pyrimidine synthesis --> increased orotic acid
production (CPS you would not see orotic acidemia).

A 15 month old - 9 month history of recurrent bacterial infections - Multiple areas of honey
crusted lesions over the lower extremities. Gram stain shows gram positive cocci - no
leukocytes - Leukocyte count very high. Pt most likely has a rare autosomal recessive disease
leading to a lack of CD18 expression. Leukocytes would be
deficient in which characteristic? -Answer- Migration
CD double digits B CD single digits T cells
MR ATM Margination, Rolling, Adhesion,
Transmigration

A 24-year-old woman - 28 weeks gestation. She has asthma - Diffuse wheezes - An inhaled B2
adrenergic agonist is administered. Most likely effect in this patient after this therapy? -Answer-
Tremor
Albuterol - relaxes bronchial smooth muscle (short acting β2-agonist). c. For
acute exacerbations. Can cause tremor, arrhythmia.

A 70-year-old man - 1 week SOB with mild exertion, cough, fatigue, and weakness. The
shortness of breath frequently awakens him from sleep and is partly relieved by sitting or
standing. No Medications. 20 year history of hypertension. BP 180/110. Crackles heard on
auscultation. A chest x-ray shows cardiomegaly. Decreased left ventricular systolic function.
What is SV, EDV, CO? -Answer- Stroke volume Decreased
Left EDV Increased
Cardiac Output Decreased
Decompensated left heart failure

A 32-year woman - 7 day of history of sneezing, nasal stuffiness, and water eye. She has a
history of similar symptoms each spring while gardening. Which cells will be increased in her
nasal secretions as a result of this reaction? -Answer- Eosinophils Mast cells degranulate,
producing histamine which attracts eosinophils. The early stage of an allergic reaction is
mast cell mediated, but the late stage (including mucus production) is mediated by
eosinophils. Charcot Leyden crystals: are made up of Eosinophils

Newborn with ARDS - intubation and 100% inspired oxygen with continuous PEEP - One month
later - a CXR shows increased densities in all lung fields. He develops pneumonia and dies.
Microscopic exam shows prominent interstitial fibrosis surrounding dilated airspaces with
bronchiolar metaplasia. Most likely diagnosis? -Answer- Bronchopulmonary Dysplasia
Tissue damage to lunch with supplemental oxygen oxygen
toxicity due to free radical generation.
A 32-year-old man - deep laceration that injured nerve supply to flexor Digitorum muscles index
fingers. Which of the following movements can be used to specifically test for the function of
the muscle? -Answer- Flexion in distal interphalangeal joint responsible for flexion of DIP.
Medial aspect of the muscle (which flexes the 4th and 5th digit)

An experimental study to examine the function of the pro-opiomelanocortin gene. The gene is
found to encode for mRNA. This protein is a precursor for ACTH and B- lipotropin. Which
processes generate these hormones? -Answer- Post-Translational Modification
PROTEIN serves as a precursor for making the hormones thus it has already undergone
translation

POMC is a prohormone peptide chain.

It is translated and later enzymes cut this peptide in the subpeptides.

A 6 year old - 2 day history of itchy scalp. 10 children have same symptoms. Vitals normal. Exam
shows multiple hair shafts 1-2mm white, globular protuberances. Which cause of patients
condition? -Answer- Pediculus Humanus Capitis
Blood-sucking lice that cause intense pruritus with associated excoriations, "white, globular
protuberances".

A thing 66-year old - confusion. Show severe hyponatremia. SIADH (vasopressin) secretion is
suspected. Which supports diagnosis? -Answer- Urine Osomolality > Plasma Osmolality

Excessive ADH causes the collecting duct of the kidney to reabsorb huge amounts of water
that it should normally excrete. That means that the plasma will now have much more water
relative to solute

A 40-year-old man with AIDS develops severe diarrhea that is refractory to all standard oral
therapies. Diarrhea resolves after administration of a stable analogue of naturally occurring
hormone. Which hormone? -Answer- Somatostatin
Octreotide TX = refractory diarrhea
D cells in the pancreatic islets and GI mucosablocks everything GI-related

("encourages somato-stasis"): decreased gastric acid & pepsinogen secretion, decreased


pancreatic and small intestine fluid secretion, decreased gallbladder contraction, decreased
insulin & glucagon release.'
A 24 year old man - receives injection in the brachial artery. Which changes occur as a
result in response to histamine? -Answer- Cigarette Smoking

transitional cell carcinoma = painless hematuria + papillary growth

A 24-year-old man - receives injection in the brachial artery. Which changes occur as a
result in response to histamine? -Answer- Increased lymph Flow

Histamine increases microvascular permeability, i.e. Fluid + Proteins, this increases


pressure in the interstitium > lymph flow increases

direct action of histamine on the lymphatic smooth muscle via stimulation of H1 (and in
some vessels H2) receptors.
A 37-year-old - drank bottle of whiskey- great despair - broke up with boyfriend. She
describes mood as depressed. She says there is no one I can trust but you. Everyone I work
with is a jerk. The patient has which of the following personality disorders? - Answer-
Borderline

unstable mood and a crazy relationship. She's also splitting


35 year old man - HTN - BP 180/100. TX started with losartan. Which is likely to occur?
-Answer- Increased Serum Angiotensin II Concentration

increased renin, increased Ang I, increased Ang II, decreased aldosterone and
unchanged bradykinin
man and wife - 25-year-old- come for genetic counseling. Both had microcytic anemia and
increased fraction of hemoglobin. Heterozygous null mutation of the B-globin gene, and the
heterozygous mutation known to cause a 50% decrease in B-globin gene function in one allele.
Which is the B-globin function possibilities for this couple's offspring? -Answer- One in four will
have 25% B Globin Function may require occasional Transfusion Thalassemia - recessive
Make a punnett square with a cross
B B+ and B B0;
B+ represents 50% function B0
represents 0% (null) function. the
husband = B B0 genotype wife has
a B B+ genotype.
Cross of these two will result in the following genotypes; BB, BB0, BB+, B+B0 BB =
100% function, BB+ = 75% function, BB0 = 50% function, B+B0 = 25% function

A 62-year-old man - DX with atrial and Ventricular Arrythmia - appropriate pharmacotherapy


was initiated at the time. BP 136/88. PE shows no abnormalities. An ECG shows normal sinus
rhythm with prolonged QT interval. Which drug is most likely cause of these findings -Answer-
Solatol type III + (Type II antiarrhythmic).
(K+ channel-blocking) beta-blocker
activity
decreased heartrate and blood pressure (beta-1 blocking activity), with the QT prolongation -
all type III AADs cause QT prolongation.

55 year old with T!DM - intermittent burning pain in his feet during the past 4 months. Exam
shows allodynia bilaterally. Decreased sensation to pinprick. Motor, DTR, and vibration
sense normal. Which is most likely cause of pain in this patient? -AnswerPersistent
activation of voltage-gated Na+ Channels in Nociceptors allodynia. = intact sensation, it's
just "turned up" way too much.
Diabetes causes non-enzymatic glycosylation - damage neuron

After completing 6 courses of chemotherapy for cancer of the breast, a 45 year old woman -
SOBs, dyspnea, orthopnea. Basilar crackles heard bilaterally. S3 gallop. X rays show an enlarged
heart, bilateral interstitial and alveolar edema, and bilateral pleural effusions. Which
chemotherapy drug causes these findings? -Answer- Doxorubicin
CHEMOMAN
anthracyclines is dilated cardiomyopathy.
Doxorubicin = Cardiac problems
Bleomycin = Pulmonary problems
Cardiac = BILATERAL crackles/effusions since blood is backing up it would affect both lungs
while Bleomycin could present as unilateral. Main though thing is the S3 which makes it
automatically cardiac-related

A 50 year-old man - diarrhea and 13.6 kg (30lb ) weight loss over the past 2 months. He has
pale, foul smelling stools with oil droplets. History of alcohol dependence and abdominal pain.
X-ray shows calcifications in the mid-upper abdomen. Which pathophysiologic mechanisms
most likely cause this condition? -Answer- Generalized Malabsorption
"calcifications in the mid-upper abdomen" is most likely referring to a chronic
pancreatitis. lack of lipase= pale, foul-smelling stools with oil droplets per pt Hx no
amylases, proteases, nor trypsinogen (to activate other enzymes

A 32 year old woman - 20 weeks pregnant - severe heartburn for 3 weeks. Epigastric tenderness
of palpation f the abdomen. Which drug is contraindicated because of risk of spontaneous
abortion? -Answer- Misoprostol
Prostaglandin analog (PGE2) that acts on the stomach to promote mucus protection of the
stomach lining, but also acts in the uterus to encourage contraction, which makes it useful for
abortion.

A 75 year old man - unconscious - collapsed 30 minutes ago. He has abdominal pain and
history of abdominal mass. Smoker for 60 years. PE shows pulsatile abdominal mass and
minimal bowel sounds. Which is cause? -Answer- Atherosclerotic AAA - Atherosclerotic
changes leads to aneurysmal dilatation of abdominal aorta
35-year old woman - fatigue and lethargy- 20lb weight gain - wears a coat because she
is cold. PE shows coarse skin. Thyroid scan shows localization of radioiodine in the
midline of the inferior aspect of the oral cavity. Radioiodine is most likely localized
inferior to which structure? -Answer- Foramen Cecum
lingual thyroid, a persistence of thyroid tissue at the base of the tongue due to failure of
migration

A 58-year-old man - ER - 1 hour after being kicked in the side by a horse. 2 years ago DX with
chronic idiopathic myelofibrosis. BP 90/50 He drinks 12-ounce beers daily.
Abdominal exam shows guarding and tenderness over the left upper quadrant. Hematocrit
increased; Leukocyte count increased. Ultrasound shows intraperitoneal fluid. 4 hours later - a
laparotomy is most likely to show which of the following? -AnswerRuptured Spleen
myelofibrosis can cause an enlarged spleen.

A 10-year-old - intermittent dyspnea - Pt turns blue. Episodes resolves when she squats. A
systolic murmur is heard at birth. Today, 4/6 systolic murmur is heard at the pulmonic area.
Which is most likely in this patient? -Answer- Palpable Right Ventricular
Lift PROVE - Pulmonic Stenosis, Right Ventricular Hypertrophy, Overriding Aorta, VSD

The graph shows expiratory pressure-volume curves that compare a healthy adult with a
patient who has a 9-month history of progressive shortness of breath. Graph TLC decreased.
These findings are most consistent with which of the following? -Answer-
Diffuse Pulmonary Fibrosis TLC
Decreased
restrictive lung disease

A 26-year-old man - comes to physician for a follow-up exam - Physician prescribes a


HMG CoA reductase inhibitor. total serum cholesterol is decreased from 325mg/dl to 235
mg/dl. Cause of the decrease? -Answer- Increased number of LDL receptors on hepatocytes
HMG CoA reductase inhibitors block the ability of the body to produce its own cholesterol

A 24 year old man - fever and chills for 1 day -history of UTI - Ciprofloxacin is begun. Two days
later urine culture grows E Coli resistant to ciprofloxacin. Cause of organisms resistance? -
Answer- Alteration in DNA Gyrase
Cipro = MOA of fluoroquinolones is to inhibit prokaryotic topoisomers II (DNA Gyrase)
occasionally Gram (-) organisms like e-coli develop resistance by mutating their DNA gyrase so
drugs like cipro cannot inhibit
Results of a 5-year screen program for HIV in a population of 10,000 sex workers.
Which is closet to average incidence:

Time 5 years Pop at Risk - 4800 New Patients 250

What is approximation of average annual incidence of infection in this population? -


Answer- 5%

(incidence = new cases / total)

The average of these %s for all the years = 5.58%. So that's close enough to 5%.
A 43 year old woman with a 2 year history of labile HTN - very drastic changes in BP One day
after propranolol is started bp is 214/132. Serum cateccholamine concentrations are 4X upper
limit. A Ct scan shows 2-cm nodule in the right adrenal gland. Which is most likely responsible
for increased blood pressure after the propranolol therapy? -Answer- Unopposed a-adrenergic
tone with beta blockers making excess catecholamines - pheochromocytoma
Starting beta blockers before alpha blockade in pheochromocytoma is contraindicated.
An 80 y/o old -intermittent headache and aching pain in her jaw while chewing. Which is
next step in establishing diagnosis? -Answer- Determination of erythrocyte
sedimentation rate

Giant Cell Arteritis - jaw pain with eating and unilateral headache

An 80 y/o old -intermittent headache and aching pain in her jaw while chewing. Which is
next step in establishing diagnosis? -Answer- Determination of erythrocyte
sedimentation rate

Giant Cell Arteritis - jaw pain with eating and unilateral headache
Identified by determining the erythrocyte sedimentation rate.
A 28 year-old-man - with AIDs - has a high fever, headache, lethargy. Exam shows papilledema
and nuchal rigidity. A mucicarmine-stained smear of CSF is shown. Which
is the primary site of infection with this organism? -Answer- Lungs Cryptococcal
meningitis that is transferred through respiratory droplets
Mucicarmine stain
A 7-year-old boy - metabolic acidosis and persistent phosphaturia. Patient most likely to
have a primary defect of which labeled site in the drawing of the nephron? -Answer-
Proximal tubal
A 55 year old man - 1- hour history of severe chest pain, nausea, and vomiting. He is agitated,
clammy, and sweating profusely. JVP is 12cm and crackles heard in lung bases. Which
hemodynamic changes (Pulmonary Capillary Wedge Pressure, Cardiac
Output, SVR)? -Answer- Pulmonary Capillary Wedge Pressure - Increased
Cardiac Output - decreased
Systemic Vascular Resistance - increased
cardiogenic shock. Characteristic features of cardiogenic shock: increased cardiac pressures and
decreased cardiac output due to impeded forward flow, as well as increased systemic vascular
resistance

35 year old male - 1 week of history of severe daily headaches. Pain is located around the left
eye and associated with excessive tearing and redness, Headaches last for 30 minutes to 2
hours. - No focal finding. DX? -Answer- Cluster Headache. excessive tearing
A 52 year old - severe pain in midback for 2 weeks. History of left breast cancer treated with
mastectomy and chemotherapy. Tenderness in palpation over the thoracic spine. MRI shows
metastases to thoracic vertebral bodies. Which veins is the most likely path for tumor cells in
the breast to metastasize to the vertebral bodies? -Answer- Intercostal Veins
Most direct pathfor breast cancer to metastasize to the vertebra are the intercostal veins.
Breast cancer will cause mixed, lytic and blastic lesions once in bone.

A 57-year-old woman - intubation in ICU after developing progressive dyspnea and hypoxemia.
Pt was admitted for cirrhosis and was admitted to the hospital because of melena. Transfusion
was initiated. Bilateral crackles are heard on inspiration. There are no wheezes. PE shows spider
angiomata on chest and abdomen. Bilateral basilar crackles . CXR shows bilateral diffuse
airspaces without pleural effusions or cardiomegaly. The patint develope which of the following
transfusions reactions? - Answer- Acute Lung Injury
Transfusion-related acute lung injury (TRALI) within six
hours after blood product administration
A 3-year-old boy - epigastric pain and bile stained vomitus. Child had been well prior to
episode. Cause of problem? -Answer- Annular Pancreas
The 35-year-old woman - indicated by arrow - family history of retinitis pigmentosa (Male-
to-Male transmission). Which mode of inheritance can be eliminated based on pedigree? -
Answer- X-Linked Recessive

A 50-year-old man - surgical transection of the obturator nerve for unbalance muscle spasm
affecting the hip. Which muscles is most likely paralyzed by this procedure? -
Answer- Adductor Brevis obturator
nerve
"Innervates the muscles of the medial compartment of the thigh (obturator externus, adductor
longus, adductor brevis, adductor magnus and gracilis)."

34 year old - 2 weeks history of severe back pain, decreased appetite, and malaise. Six weeks
ago began infliximab. An MRI of spine shows osteomyelitis of L1-l2 with destruction of the
interverterbral discs and 1.2 cm abscess. Gram stain is negative. Bone specimen shows a
inflammation with granuloma formation. Cause of findings? -
Answer- Tuberculous Osteomyelitis
TNF-a inhibitor always check for TB or autoimmune

An 80-year-old woman - T2DM, HTN, CKGD. BP 165/95. PE shows edema. Compared with a
healthy individual - which lab finding (hematocrit, PTH, Calictrol) is most likely in this patient? -
Answer- Hematocrit Decreased
Serum Parathyroid Increased
Serum Calcitriol Decreased
chronic kidney disease --> decreased EPO --> decreased hematocrit chronic kidney disease -
-> decreased PO4- excretion --> increased PTH chronic kidney disease -->
decreased 1,25 dihydrovitD (calcitriol) --> increased PTH
A researcher wants to design an experiment to study whether there is an increase in cancer in
patients with exposure to 50 ug/ml in drinking water compared with a persons who has less
than 5ug/l. Which is the most efficient experimental design? -Answer- Case Control Study

A 71-year-old man - sudden onset of dysarthria and dysphagia, dizziness. Exam shows markedly
decreased pain sensation on the right side of the face and complete loss of pain sensation of
the left side of the body. Occlusion in which labeled vessels? -Answer-
PICA
A 4-year-old boy - urine is red. 15 days ago - he had a sore throat, fever, and cough.
Lab show: URINE color brown, RBC Casts, RBCs 30-50, Protein 2. Most likely
diagnosis? -Answer- Proliferative Glomerulonephritis
Group A Strep - URI
RBC casts===> glomerulonephritis

PSGN can progressive to a proliferative glomerulonephritis mechanism

A 3-year-old boy - short stature and normal size trunk, large head with prominent
forehead and nasal bridge and exaggerated lordosis. Cause of the findings is a genetic
abnormality in which? -Answer- Endochondral Ossification

Achondroplasia- autosomal dominant mutation in Fibroblast Growth Factor Receptor 3.


A 4-year-old boy - 1-cm - round midline neck mass just inferior to hyoid bone. Mass is most
likely derived from which structures? -Answer- Tongue
The thyroglossal tract arises from the foramen cecum at the junction of the anterior twothirds
and posterior one-third of the tongue
A public health consultant - contacted for recommendations about primary prevention
techniques for population of women 30 to 40 years old. Which is the most appropriate
primary preventive recommendation for this group? -Answer- Regular Exercise
Primary prevention is an active step taken to head off or mitigate a disease (i.e
vaccination and lifestyle modification

Seconday prevention is early detection of a disease, promoting early intervention. HIV


testing every 6 months, or or annual mammography)

35-year-old African American Man - read pamphlet about importance of screening for skin
cancer - likes to sail and does not burn - PE shows no abnormalities - increased
risk for melanoma at which of the following locations? -Answer- Palms
68-year-old man concerned about change in sexual performance. He started dating s
40-year-old woman - My orgasms are shorter and less intense. Symmetric enlargement of
prostate. Which is likely cause of patient dysfunction? -Answer- Normal Aging Shorter, less
intense orgasms, as well as increased time needed between sex could be related to a slight
drop in testosterone with age

The breakdown of dipeptides and tripeptides to free amino acids takes place in which area of
the gastrointestinal tract? -Answer- Intestinal Mucosa
absorption of nutrients - dipeptides and tripeptides are cotransported with hydrogen into the
Intestinal mucosal cells while single AAs are cotransported with Na+.

A 68 has a loss of pain and temperature sensations on right side of the face and left side of
the body, paralysis of the vocal cord on the right, and absence of the gag reflex on the right.
Which region of brainstem involved? -Answer- Right Dorsolateral Medulla Pain and
temperature sensation of the body is part of the spinothalamic tract, The loss of gag reflex
and paralysis of the vocal cords imply impairment of cranial nerves IX and X, both of which
localize to the medulla Midbrain 1-4, Pons 5-8, Medulla 9-12

A 57 year old man - hoarsness and difficulty swallowing for 3 days - He is unable to elevate the
right side of the palate. Which cranial nerve is most likely damaged?
(IMAGE SHOWN -Answer- Area Labeled F - Vagus
CN 1,2,3,4 - Midbrain CN 5,6,7,8 - Pons CN 9,10,11,12 - Medulla
Inability to elevate the palate suggests damage of the vagus nerve

52-year-old man massive MI. TX with oxygen, beta blockers, aspirin, and reperfusion therapy.
18 days later he develops ventricular fibrillation and dies. A photomicrograph of cardiac tissue
from the site in site is shown. Which describes the appearance of the heart? -Answer-
Granulation Tissue
highly vascular - characteristic of granulation tissue. Scar tissue formation will be closer to 1
month, plus you will see lots of fibrosis on histology.

A 35-year-old woman - difficulty urinating 2 days post-partum. She has a long history of asthma.
Which of the following is both and indication and a contraindication for bethanechol in this
patient? -Answer- Parasympathomimetic Stimulation
Bethanechol agonist on M3 receptors which can treat her urinary problems but it might
exacerbate her asthma symptoms since there are M3 receptors on the lungs.

17 year old brought in by ambulance after head injury - in coma - PE show multiple bleeding
lacerations over the head. Emergent craniotomy is indicated, but physician unable to contact
parents. A decision is made to proceed with operation without permission - the decision is most
consistent with which ethical principle? -AnswerBeneficence
health care providers have a duty to be of a benefit to the patient and should take positive
steps to prevent and to remove harm from the patient.
Consent for minors (FA2020 pg 265): Consent should be obtained from parents, except
for Emergency Medicine.
A 35-year-old man - small cell carcinoma has systemic hypertension and hypokalemia.
The most likely cause of these findings is ectopic secretion of the following hormones? Answer-
ACTH
ACTH increases cortisol → hypertension (alpha-1 upregulation & cortisol can bind to
aldosterone receptors at high concentrations)
ACTH increases aldosterone → hypertension + hypokalemia (K+ dumped in collecting duct)

A 35-year-old woman - undergoes left oophorectomy because of a 5 x5 cm ovarian mass. Which


structure at greatest risk when dividing the suspensory ligament? Answer- Ureter
suspensory ligament has ovarian vessels --> ovarian artery + ureter can be damaged in
oophorectomy
cardinal ligament has uterine vessels --> uterine artery + ureter can be damaged in
hysterectomy
. 40-year-old man - horseback riding - develops swollen mass in left thigh. Mass
becomes circumscribed and firm. DX? -Answer- Myositis Ossificans b. bone formation
in muscle
c. Condition where bone tissue forms inside muscle or other soft tissue after an injury. It tends
to develop in young adults and athletes who are more likely to experience traumatic injuries

=In which of the following stages of the cell cycle are mitotic cyclins synthesized? -
Answer- G2
b. G1 checkpoint: Cyclin D/CDK4 complex with p53 and rb G2 checkpoint: Cyclin B and CDK 1
G1 is associated with higher
numbers D and 4
G2 is associated with
B and 1
59-year-old with gout - repeated episodes of gouty
arthritis unresponsive to preventative
therapy. Which increase uric acid excretion in
this man? -Answer- Probenecid
Probenecid makes
you Pee
Colchicine clenches your
microtubules
Probenecid and high-dose salicylates inhibit
reabsorption of uric acid in proximal
convoluted
tubule
40-year-old man - horseback riding - develops
swollen mass in left thigh. Mass
becomes circumscribed and firm. DX? -
Answer- Myositis Ossificans
bone formation in
muscle
A cancer patient- TX with severe high dose chemotherapy - has severe bone marrow
suppression. Which cytokine most beneficial to stimulating bone marrow? -Answer-
Granulocyte Colony Stimulating factor
Chemo. patient with bone marrow suppression > give G-CSF.

An 80-year-old man - 2-week history of severe, persistent urinary tract infection. An 80 mg dose
of Drug X is administered intravenously after the infusion is complete. 30 minutes after the
infusion his serum concentration is 4ug/ml. Assuming half-life of 3- minutes and elimination
half-life of 24, the volume of distribution of this drug in L the peripheral compartment is closest
to which of the following? -Answer- 20
Vd = [drug administered] ÷ [plasma drug concentration] First convert it all to
g/L because this is how the answer will be: administered: 80 mg = 0.08 g
plasma concentration: 4 ug/ml = 0.004 g/L, Vd = 0.08 grams ÷ 0.004 g/L = 20 L

A 55-year-old man - diagnosed with coronary artery disease. Begins treatment with aspirin to
prevent thrombus. Which effects of aspirin on platelet function is most likely to decrease this
patient risk for thrombus? -Answer- Decreased Adherence
Aspirin inhibits platelet aggregation and produces a mild bleeding defect by inhibiting
cyclooxygenase, a platelet enzyme that is required for TXA2 synthesis."
closest thing to decreased aggregation that still made sense with aspirin's mechanism of action
. A 3-month old boy - yellow eyes and skin and weakness since birth. PE shows
jaundice, large fontanels, a flat midfacial area, hypotonia, and hepatomegaly.
Serum Studies:
Total Bilirubin Increased
AST Increased
ALT Increased
Very-long Chain Fatty Acids Increased
A liver biopsy shows foamy, lipid-filled hepatocytes, necrosis, and absence of specific organelle.
The absent organelle is most likely? -Answer- Peroxisomes
Peroxisome = Membrane-enclosed organelle involved in β-oxidation of very-long-chain fatty
acids (VLCFA) (strictly peroxisomal process). Zellweger syndrome

In a culture of motile spore-forming bacteria, which inhibits bacterial growth by causing


double stranded breaks in DNA? -Answer- Irradiation with X-Rays more energy in a
light wave the more likely it is to cause cellular (DNA) damage.
Ultraviolet has less energy than X-rays (goes through your flesh)
X-Ray > UVA (also able to cause double strand break) > UVB (most likely single strand break -
repaired)
80-year-old - decreased frequency of bowl movements. (1 or 2 bowel movements
weekly). No benefit from fiber intake. She manually supports the posterior vaginal wall
in order to defecate. Which is cause of patient's symptoms? -Answer- Damage to rectovaginal
Septum
A rectocele is a herniation (bulge) of the front wall of the rectum into the back wall of the
vagina. The tissue between the rectum and the vagina is known as the rectovaginal septum

A 65-year-old woman - T2DM - follow-up exam - PE shows no abnormalities. BP 135/80.


Abdominal ultrasound shows decreased size of both kidneys. MR aortography shows bilateral
proximal renal artery stenosis. DX? -Answer- Atherosclerosis MOST common cause of renal
artery stenosis...with fibromuscular dysplasia being the SECOND most common cause
Atherosclerotic plaques—proximal 1/3 of renal artery, usually in older males, smokers.
Fibromuscular dysplasia—distal 2/3 of renal artery or segmental branches, usually young or
middle-aged females

A 65-year-old man - comes to physician because of difficulty sleeping for the past month. He
has impaired concentration and decreased energy. He has recurrent thoughts of death since
having MI 5 months ago. Which drug to add to regime? Answer- Paroxetine
major depressive disorder: loss of interest/anhedonia (need to have this or depressed
mood),sleep problems, weight changes, decreased energy, thoughts of death. Meets criteria
because > 2 weeks timeframe. SSRIs are first-line; paroxetine i
75-year-old man with a 10-year history of progressive renal failure - Lab studies show a serum
urea nitrogen concentration of 40 mg/dl and serum concentration of 3mg/dl. Ultrasound shows
Hydronephrotic and a Dilated Ureter. Most likely cause of patient's renal failure is an increase in
which? -Answer- Hydrostatic pressure in Bowman Space the Bowman's capsule space exerts
hydrostatic pressure of its own that pushes against the glomerulus. Increased Bowman's
capsule hydrostatic pressure will decrease GFR, while decreased Bowman's capsule hydrostatic
pressure will increase GFR.

3-year boy with AIDS develops giant cell pneumonia 3 months after exposure to an
unimmunized cousin who had rash, conjunctivitis, rhinitis, and kolpik spots. The virus
responsible for the pneumonia has which of the following types of genomes? -AnswerNegative
Stranded RNA
Measles = Negative ssRNA virus

. In a study of drug action on neoplastic cells in culture, drug X markedly inhibits cell replication.
A microscopic view of a typical cell incubated with drug X is shown. Drug X is most likely to be
which -Answer- Vincristine
chemotherapeutic drug that inhibit B-tubulin and polymerization into microtubules. The cell in
the picture is stuck in anaphase, with microtubules attached to its chromosomes,
29-year-old jaw pain- 1 hour history of bilateral jaw pain that began after he tried to bite
into a cheeseburger. PE shows extensive drooling and an inability to elevate the
mandible. Xray shows bilateral anterior dislocation of the temporomandibular joint.

Reduction of the TMJ is recommended. Relaxation of which is most likely to facilitate


this procedure? -Answer- Lateral Pterygoid

3 muscles close the jaw: Masseter, teMporalis, Medial pterygoid 1 muscle opens the
jaw:

Lateral pterygoid loosen jaw

ALL are innervated by trigeminal nerve, V3


Mnemonic: M's munch (close the jaw), L's lower (loosen/relax the jaw)

17-year girl comes to the physician because she has never had a menstrual period. She is not
sexually active. She is 5'11 and 110 lb. BMI is 15. Breasts development is tanner stage 5 and
pubic and axillary development is Tanner Stage 1. Pelvic exam shows blind vaginal pouch -
Tanner Stage 1. Which is found on further testing?
-Answer- Increased Serum Testosterone Concentration
Androgen insensitivity syndrome
Defect in androgen receptor resulting in normal-appearing female (46,XY DSD); female external
genitalia with scant axillary and pubic hair, rudimentary vagina; uterus and fallopian tubes
absent due to persistence of anti-Müllerian hormone from testes

A 70-year-old man - 1 year history of shortness of breath and nonproductive cough - unable
to tolerate mild activity. PE shows clubbing of the fingers. Inspiratory crackles are heard at
both lungs. A CT scan shows patchy subpleural reticular opacities. Which pulmonary functions
will show a result greater than predicted? -Answer- AlveolarArterial PO2 difference
pulmonary fibrosis = restrictive (not obstructive)-type disease. no occupational exposure =
idiopathic pulmonary fibrosis

A 50-year-old woman with HIV - follow-up exam. Receiving antiretroviral therapy including
zidovudine and nelfinavir. She is also receiving pentamidine for pneumocystis and azithromycin
for Myocbacterium Avium Complex. Drug induced bone marrow suppression is suspected.
Cause? -Answer- Zidovudine
NRTIs are the main HIV therapy drug that can cause bone marrow suppression (not as common
with NNRTIs). This class includes zidovudine, didanosine, emtricitabine, lamivudine, stavudine,
abacavir. Zidovudine is most known for this side effect.
47-year-old - recurrent joint pain - the onset is abrupt involving the ankles and knee. PE
shows yellow white nodules at the tips of several fingers. A photomicrograph of the joint
fluid is shown. The crystals (YIPA) are most like composed of which? -Answer-
Monosodium Urate

Gout [Monosodium urate] Crystals are needle shaped and ⊝birefringent under
polarized light (yellow under parallel light, blue under perpendicular light
50-year-old - admitted to hospital for management of an acute exacerbation of heart failure.
Multiple treatment for similar episodes (8x times). Bilateral crackles are heard on pulmonary
exam. There is pitting edema of the lower extremities. She is placed on restricted 2L of liters.
Her fluid balance is closely monitored. Nurse find pt drinking in bathroom drinking directly from
faucet and holding a pitcher of water. Explanation for patient's lack of clinical improvement? -
Answer- Factitious Disorder
trying to fake a disease or symptoms -> they're thirsty and selfish with no ability to
regulate their own behavior

58-year-old - 3 day history of malaise, increased urinary frequency, dribbling of urine, and
feeling of incomplete bladder emptying, deep, and dull pelvic pain. Digital rectal exam shows an
enlarged, tender prostate. Which infectious agents cause these findings? -Answer- Escherichia
Coli
Most common cause of acute prostatitis in older men is E. Coli

A 35 year old man- disoriented and hallucinating. He has a 20 year history of alcoholism. On
admission, he has seizures. His blood pressure is 180/100. Serum potassium is 2.5 mEq/L,
and urine potassium is 40 mEq/L. Alcohol withdrawal is suspected. Which of the following
cause the hypokalemia? -Answer- Catecholamine mediated intracellular shifts of K+
mediated intracellular shifts of K+
Catecholamine : 1. increases insulin release --> glu and K will enter cells. Causing hypokalemia
in the plasma 2. Can trigger the beta 1 receptor causing the release of Renin---> Angiotensin 2
(can be contributing / causing an increase in BP)--> aldosterone --> you waste K in the urine.

A 73-year-old woman - has easy fatigability for 2 years. She had an ileal resection of Crohn
disease. She has pale oral mucosa. A peripheral blood smear is shown. Most likely mechanism
of this disorder? -Answer- Failure of conversion of N5- methyl tetrahydrofolate to
tetrahydrofolate
"Structural abnormalities of the terminal ileum, such as Crohn disease and surgical resection,
can cause decreased absorption of vitamin B12." - First Aid General Principle

A 27-year-old woman -underwent a cesarean delivery. PE shows a firm subcutaneous nodule


adjacent to surgical incision. Microscopic exam shows fibrous connective tissue, macrophages,
multinucleated giant cells, fibroblasts, a few lymphocytes. Which substances promotes
fibroblast migration and proliferation most likely led the development of this lesion? -Answer-
Transforming Growth Factor -B TGF-Beta is an important fibroblast growth factor. Pathoma
Ch2, page 20. hypertropic scar or keloid. Both arise due to over-expression of TGF-beta

A 73-year-old woman - has easy fatigability for 2 years. She had an ileal resection of Crohn
disease. She has pale oral mucosa. A peripheral blood smear is shown. Most likely mechanism
of this disorder -Answer- Failure of conversion of N5- methyl tetrahydrofolate to
tetrahydrofolate
b. "Structural abnormalities of the terminal ileum, such as Crohn disease and surgical resection,
can cause decreased absorption of vitamin B12." - First Aid General Principle A 25 year old man
- 3 day history of abdominal cramps and diarrhea. He appears anxious. PE shows piloerection
and abdominal tenderness. Neuro exam normal. Most likely cause of patient's condition is
withdrawal from which substances? -AnswerHeroin
opioid withdrawal sxs (= sweating, dilated pupils, piloerection ("cold turkey"), rhinorrhea,
lacrimation, yawning, nausea, stomach cramps, diarrhea ("flu-like" symptoms)"An obese 57-
year-old man - smoked 2.5 packs of cigarettes daily for 40 years. PE shows mild dyspnea. A CXR
shows a mass in one lobe of right lung. Exam shows squamous metaplasia of the bronchial
mucosa. Which best describes changes in bronchial mucosa?

55-year-old woman - 3 month history of difficulty using her hands and a 3 week history of
muscle cramps. She has 20 lb weight loss Neuro exam shows tongue fasciculations and lower
extremity weakness and atrophy. Sensory exam is normal. A lesion at which of the site most
likely cause these findings? -Answer- Lower Motoneurons
"LMN deficits... dysarthria, dysphagia, asymmetric limb weakness, fasciculations, atrophy
=UMN deficits... pseudobulbar palsy (i.e. dysarthria, dysphagia, emotional lability, spastic gait,
clonus])"

An obese 57-year-old man - smoked 2.5 packs of cigarettes daily for 40 years. PE shows mild
dyspnea. A CXR shows a mass in one lobe of right lung. Exam shows squamous metaplasia of
the bronchial mucosa. Which best describes changes in bronchial mucosa? -Answer- Normal
Ciliated columnar replace by normal Squamous epithelium
Metaplasia - A reversible adaptive response in which there is "Reprogramming of stem
cellsreplacement of one cell type by another that can adapt to a new stress." Squamous
metaplasia (SQM) is a pre-neoplastic change of the bronchial epithelium observed in the lungs
in response to toxic injury induced by cigarette smoke
A 56-year-old - pneumonia found to have advanced lung cancer. Pt refuses palliative
radiation therapy and wants to be discharged. He is alone. He says he will return in
months after he makes money from selling his invention to cure arthritis. Most
appropriate next step? -Answer- Determine whether the patient has decision making
capacity

Does he understand risks and benefits


Autonomy is the most important ethics principle

In this situation, a patient with advanced disease unlikely to be cured is refusing treatment,
which is his right under the principle of autonomy. However, his comments about "returning in
6 months after curing arthritis" are questionable, and warrant determining if he has decision
making capacity.
A 73-year-old man incurable malignant neoplasm of the lung, and his condition is slowly
deteriorating. He request the respirator to be removed. After discussing with his
children, physician decides to remove the respirator. Which describes his action? Answer- Both
legal and ethical
most important ethical principle that supersedes all others is autonomy. From an ethical
standpoint, this patient has the right to refuse further treatment as he is mentally competent,
in this case in the form of having the respiratory removed. From a legal standpoint, the
physician is allowed to discontinue treatment for a patient if that is what the patient wants.

28-year-old with Chronic Hep. C - fever and progressive shortness of breath during the past 2
days. His temperature is 102F. PE shows wasting and intercostal retractions. Crackles heard on
both lung fields. CXR shows diffuse hazy infiltrates. Leukocyte count is 2000. Lymphocyes are
5% To explain the cause of the illness, the patient should be tested for which chronic viral
infections? -Answer- HIV
AIDS diagnosis: ≤ 200 CD4+ cells/mm3 (normal: 500-1500 cells/mm3). This patient has 5%
lymphocytes out of 2000 overall = 100 cells = AIDS.

An investigator- studying new drug for patients undergoing adjuvant radiation therapy. The
drug is administered prior to irradiation to minimize localized tissue damage at the irradiated
site. The drug most likely inhibits which effects of external beam radiation? - Answer- Free
Radical Formation Radiation- damage by generating hydroxyl free radicals two main of
radiation = ionizing and nonionizing. ionizing = radiotherapy - free radical formation non-
iodizing = is from the sun- DNA damage by formation of pyrimidine dimers
3-week-old female - increased concentration pf immunoreactive trypsin. Cystic fibrosis is
suspected. At 4 months, her sweat chloride is greater than 60 mmol/L. Analysis of the 70 most
common CF transmembrane gene mutations are done. Results show a mutation in one allele.
Which explains the findings? -Answer- The patient has another mutation not included in the
previous analysis
you need a mutation in 2 alleles to get CF [since it is autosomal recessive] If you still
have 1 functional copy of the CFTR gene, you can still make the CFTR protein (the
chloride channel/transporter), hence your body won't have any issu

A 13-year-old boy- chest swelling the began 6 months ago. Pt reluctant to take off his shirt and
embarrassed. PE shows bilateral 1-cm, mildly tender, rubbery nodules under the areolar region.
Sexual development is Tanner stage 3. Most appropriate for physician to state which of the
following? -Answer- This will typically resolve within the next 12 to 18 months
Gynecomastia due to puberty (excess testosterone → estrogen) This goesaway naturally
(apparently in 12 to 18 months)
A 35-year-old man - quadriplegia - develops UTI. A culture of urine grows an organism
that is susceptible to aminoglycosides. Gentamicin is administered. If nephrotoxicity
occurs in the patient, which of the labeled sites in the drawing of the nephron shown? -
Answer- PCT

Aminoglycosides are nephrotoxic; nephrotoxic chemicals/drugs cause acute tubular


necrosis (ATN), characterized by damage to the PCT

Aminoglycosides are nephrotoxic; nephrotoxic chemicals/drugs cause acute tubular


necrosis (ATN), characterized by damage to the PCT
A 20-year-old woman (college student) with asthma - exacerbations of symptoms over last 3
months. Classes are stressful. I have a poodle, My roommate smokes. Advise the patient to do
which of the following? -Answer- Ask the roommate not to smoke in the apartment
Symptoms started 3 months ago and she moved in with a roommate who smokes indoors 3
months ago.

A Cohort study is done to evaluate the association between use of video display terminals
(VDTS) by women and the risk of congenital heart disease in their offspring. The relative risk
ratio for congenital heart disease in newborns born to women who work 6 or more hours. VDT
1.1 (95% confidence interval: 0.8 -14). - which is of the following is the p vale calculated for a
chi square test? -Answer- 05 < p < 1.9
P value refers to probability of type 1 error. When the 95% confidence interval includes the null
value
Null (H 0 ) Hypothesis = Hypothesis of no difference or relationship (eg, there is no
association between the disease and the risk factor in the population). If the 95% CI for
odds ratio or relative risk includes 1, H 0 is not rejected.
Since the CI here includes 1, it means that its not significant and therefore p value should not be
< 0.5. Therefore p = 0.5-1.

Drug X is given to a 25-year-old normal patient. Drug cause increase in resting heart from 62 to
74 min. Prior to administration of the drug heart rate increased to 150/min with exercise. With
the drug, heart rate increased - 98 min (less with drug). Mechanism of action of the drug X most
likely involves? -Answer- Partial Agonist at B-adrenergic Receptors
In the presence of agonist partial
agonist act like an antagonist

A 34-year-old brought to ER - fall with outstretched hand- Examination of UE shows hard mass
proximal to wrist joint anteriorly. X-ray shown of the following bones is most likely dislocated in
this patient? -Answer- Lunate
FOOSH - Scaphoid most common fracture, Lunate most common dislocation scaphoid is most
common one to be fractured, lunate is most common to be dislocated.
Straight line to Pinky, Here Comes The Thumb
(Scaphoid, Lunate, Triquetrum, Pisiform)
(Hamate, Capitate, Trapezoid, Trapezium)

A 22-year-old man - involved in a motorcycle collision. PE Shows dysmetria on the right. Muscle
strength is normal. Which labeled structures in the photograph - most likely site of injury? -
Answer- (Area E) cerebellum
Dysmetria ( also, dysdiadokinesia and intention tremor) is lateral cerebellum. (E).
D - Truncal Ataxia is a problem with the central cerebellum
37-year-old - farmworker - 12 hour severe pain in abdomen and legs and spasms in jaw. PE
shows spasms of the masseter and abdominal musculature. Loud noise triggers spasms
causing pt to be intubated. Administration of antitoxin prevents further symptoms that
involve bidning of a toxin to which? -Answer- Synaptobrevin tetatnus -
blocks release of GABA and Glycine Renshaw cells
SNARE proteins

A 55-year-old man with alcoholism is brought to emergency department 30 minutes after


consuming a bottle of methanol. Physician recommends patient be treated with ethanol. To
increase Km of hepatic alcohol dehydrogenase for methanol in this patient the ethanol must act
as which? -Answer- Competitive Inhibitor (Increase KM for substrate).
b. Ethanol or Fomepizole prevents binding of methanol to active site instead of
using fomepizole they just gonna get him real drunk

A 7-year-old girl - brought to the emergency department because of a 1-hour history of


excessive sleepiness and slurring of speech. Mother states girl was well and had eaten 2 hours
ago. Blood glucose is 42 mg/dl. IV dextrose is administered - 5 minutes later her symptoms
resolve. PE shows no abnormalities. C peptide 0.5ng/ml (N=0.8-3.1),
Insulin 32 - N= 5-20. DX? -Answer- Factitious Disorder Imposed on another
(No history of T1DM),
Exogenous Insulin C Peptide is low and Insulin High

A 54-year-old, MI - progressive hypotensive. Which physiologic events is most likely to cause


swelling of parenchymal cells in his vital organs within minutes?? -Answer-
Decrease in Intracellular ATP Concentrations
Hypoxia impairs oxidative phosphorylation—less ATP—Less Na / K + (Na builds up causing
swelling)
Theme of pathology: hypoxia impairs oxidative phosphorylation --> less ATP --> less
Na-K pump activity so sodium builds up in the cell, causing swelling.
Calcium Builds up causing Lactic Acidosis, anaerobic glycolysis, lactic acid production
A 35 year left pelvic fracture in MVA accident. A 3-week period of bed rest for this
patient- most likely to cause which physiologic change? -Answer- Decrease Blood
Volume

Pt is supine, increase preload back to heart

ANP secreted due to RA Stretch, increase diuresis, reduction blood volume


33-year-old women comes to the physician because of a 2-month history of easy fatigability
and dark urine in the mornings. No fever or weight loss. No meds. Vitals normal. Urine blood
and protein.

Direct Antiglobulin (Coombs) test is negative. The result of the acidified serum test is positive.
Which describes the underlying cellular cause of this patients condition? -
Answer- Defect cell membrane anchor protein
Paroxysmal Nocturnal Hemaglobinuria - complement activity against RBC - impaired
GPI anchor - associated with aplastic anemia
Paroxysmal Nocturnal Hemaglobinuria

38-year-old - blood tinged discharge from right breast for 3 months. Menses have occurred at
regular 28-day intervals. She takes no medications. PE shows no breast masses and no palpable
axillary adenopathy. Bloody discharge from the upper outer corner of the right nipple.
Mammography shows no abnormalities. Cause of discharge?
-Answer- Intraductal Papilloma
most common cause of blood tinged discharge, no changes on mammography or radiology
(benign tumor)
A 32-year-old woman - diagnosed with AIDS is at greatest risk for developing which
neoplasms? -Answer- Epstein-Barr Virus Induced Brain Lymphoma
CNS lymphoma AIDS defining illness
A 62-year-old man - 2-month history severe pain of his left leg began shortly after the leg was
amputated below the knee. Throbbing, aching, and shooting pain Located in distal portion of
absent extremity. Cause of patient's symptoms -Answer- Phantom Limb Pain
(distal portion of absent extremity) chronic pre-amputation pain, post-operative surgical
pain and psychological distress. d. Phantom pains - crushing, toes twisting, hot iron,
burning, tingling, cramping, shocking, shooting, "pins & needles" Tends to localize to more
distal phantom structures (e.g. fingers and toes)

A 59-year-old woman - 10-year history of progressive right sided hearing loss. An MRI shows a
large Cerebellopontine Angle mass that has compressed Vestibulocochlear nerve. The mass
arose from which of the following cell types? -Answer- Neural Crest Cells
Cerebellopontine angle mass = Vestibular schwannoma (AKA acoustic neuroma).
Derived from Schwann cells, which are of neural crest origin. c.
Schwannoma - derived from neural crest
COPS- CNS Oligodendroctes, PNS Schwan Cells
A 27-year-old man - he and wife not able to conceive a child. Poor libido and unable to
maintain an erection. Receiving thyroid replacement hormone and corticosteroid
therapy since removal of pituitary adenoma. Testosterone is 0.05 nmol/L (N=10-35).
Azoospermia. What treatment to restore patients fertility -Answer- Injection of gonadotropins.
Removal pituitary removed- loss of FSH and LH need FSH
and LH to produce sperm
FSH stimulates the sertoli cells (line seminiferous tubules) spermatogonia produce
spermatocytes.
Testosterone - Leydig cells stimulated by LH

Serum cholesterol concentrations are measure as part of a community study. The means and
standard deviations are given. Which is the probability that a woman between the age of 50
and 54 has a serum cholesterol concentration greater than 296 mg/Dl?

Age Years 50-54 246 + or - 50 -Answer- 16%


Concentration greater than 296
Age Years 50-54 246 + or - 50 =
296 = 1 STD above the mean
1 SD = 68% of the population
2 SD= 95% of the population (95% -68%) = 27/2 = 13.5% )
3 SD= 99.7% of the population 5%/2 = 2.5

A 43-year-old woman - 1 week history of abdominal pain, nausea, vomiting, itching, fatigue. PE
shows scleral icterus and RUQ tenderness. No rash. Ultrasound shows a large stone in common
bile duct. Serum total bilirubin increased. Serum concentration of which of the following is most
likely increased? -Answer- Alkaline Phosphatase
(concentrated in liver, bile duct, kidney, intestinal mucosa) gall
stone in common bile duct- Cholestasis

A group of physician submits a report to a medical journal - 3 patients with idiopathic


pulmonary fibrosis who developed hepatoxicity following treatment with new drug. In report,
Physicians state they are unaware of any previous adverse effects. What is the study design? -
Answer- Case Series
history and treatment of small group without randomization or controls
in this study - researchers present the history and treatment of a small group of similar
patients, without describing any sorting into groups or randomization.
A 3 year old girl - history of recurrent infections. In vitro, neutrophils form patient can't
kill Staphylococcus Aureus. Defect involving which enzymes? -Answer- NADPH
Oxidase
Chronic Granulomatous Disease -

bad when dealing with catalase positive organisms (like Staph


STAPH is catalase + breaks down ROS

A 37-year-old woman with HIV infection - 6 hours of chest pain and abdominal pain and
shortness of breath with exertion. Receiving antiretroviral therapy. Lab show: Lactate
90, AST and ALT elevated. A drug from which class caused findings? -Answer- Nucleoside
Reverse Transcriptase Inhibitor'
NRTIs (specifically the nucleosides) cause lactic acidosis causes
lactic acidosis, bone marrow suppression NRTI - Bone marrow
supression, Lactic Acidosis, Anaemia
NNRTI - Hepatotoxicity, rash
Integrase inhibitor (tegras) - Myopathy (causing Increased Creatinine Kinase) Protease
Inhibitors (navir) - Lipodystrophy, Hyperlycemia, GI intolerance (think of hormonal effecs)

A man touches hot stove - 20 minutes later a blister develops at the site. Light and electron
microscopy of the inflamed tissue is most likely to show? -Answer-
Interendothelial Gaps in Venules
b. pathoma "vascular permeability occurs at post capillary venules." Burns disintegrate
post capillary venules
c. inflammatory response such as in burns, anaphylaxis or sepsis, intercellular contacts
disintegrate in post-capillary venules leading to intercellular gap formation.

15-year-old cystic fibrosis - mutation in CFTR does not prevent synthesis of CFTR protein but
does prevent CFTR from folding properly. The improperly folded CFTR protein will accumulate
in which of the following cellular compartments? -Answer-
Endoplasmic Reticulum site of protein folding protein folding happens at the RER and the stem
says the protein doesn't fold properly. CF mutation is a misfolded protein and the protein is
retained in the RER and not transported to the cell membrane

3-month-old brought to ED because of shortness of breath for 3 hours. He has upper


respiratory infection for the past 3 days. Pe shows lethargy and hepatomegaly. Labs shows
hypoglycemia, lactic acidemia, ketonemia, and metabolic acidosis. Admin of glycerol or
fructose do not increase glucose concentrations. Glucose goes up after IV admin of galactose.
A defect in which of the following liver metabolic pathwsys is most likely cause of findings? -
Answer- Gluconeogenesis
b. fasting hypoglycemia corrected with sugar, enters gluconeogenesis pathway fructose
bisphosphate deficiency
c. fructose bisphosphatase deficiency =gluconeogenesis pathway below fructose
bisphophatase.
d. Galactose on the other hand enters above it.

4-year-old history of multiple bone fractures and poor wound healing- Photograph (Blue sclera).
Which of the components of wound healing is most likely to be affected as a
direct result of her underlying diseases? -Answer- Scar Formation
Osteogenesis Imperfecta -
Brittle Type 1 collagen - blue sclera
Need type 1 collagen to get scars
18-year-old - sharp chest pain and shortness of breath 1 hour after receiving a non-
penetrating injury in rugby game. PE shows decreased breath sounds and increased tympany
to percussion. CXR show. Pt at greatest risk of which complication? -AnswerRespiratory
Acidosis increased CO2, Low Po2 compensation with resp

Which of the following best describes fbrous proteins that form two-dimensional network for
inner surface of nuclear membrane? -Answer- Lamins
b. lamin is an intermediate filament that specifically provides support to the cell nucleus.
c. Fibronectin is an extracellular matrix glycoprotein, while
d. Don't confuse lamin with laminin (science hates us clearly);
e. laminin is like fibronectin, an ECM glycoprotein and a major component of the basal
lamina of basement membranes.

17-year-old girl - significant blood loss after being injured in an automobile collision.
Blood loss stimulates bone marrow to synthesize which of the following? -
AnswerAminolevulinate
blood loss leads to hemapoiesis. 1st step in heme synthesis FA p 417 ALA) condensation of
glycine and succinyl CoA into delta-aminolevulinate. This is rate-limiting step of heme synthesis.

2-year-old- 12-hour history of headache, loss of appetite, and vomiting. Temp 103.8 BP is 90/50
mm Hg, PE shows nuchal rigidity. Lumbar puncture is done. CSF shows increased protein,
decreased glucose, abundant neutrophils, and gram positive diplococci. Which vaccine would
have prevented the patients infection? -Answer-
Polysaccharide protein conjugate vaccine
b. (Conjugate vaccine given to children under 5)
c. Meningitis caused by Strep pneumoniae, the most common cause of infectious
meningitis in general. The vaccine for Strep pneumo is a polysaccharide protein conjugate
vaccine.

30 year old man - develops tingling around lips and mouth after consuming - puffer fish at a
Japanese restaurant. Fish contains a tetrodotoxin that causes cardiac arrest in high doses. This
patient's symptoms are most likely due to the blocking action of tetrodotoxin on which ion
channels? -Answer- Sodium
tetrodotoxin inhibits sodium channels, prevents depolarization of cardiac muscles Tetrodotoxin,
found in puffer fish, inhibits sodium channels. This prevents depolarization of cardiac muscle
and neurons, which leads to death if consumed in high enough quantity. The symptoms are
vague (nausea, diarrhea, paresthesia)

35 year old with Asthma - 1 week history of abdominal pain, diarrhea, progressive cough, and
wheezing. She recently returned from a trip in papua new guinea. PE shows cutaneous larvae
over the abdomen - CXR shows bilateral central alveolar infiltrates. Stool sample shows (image
of parasite). 23% eosinophils- Which is most appropriate pharmacotherapy? -Answer-
Thiabendazole intestinal parasites get treated with bendazole
Strongyloides infection, as this is the intestinal parasite that shows larva on stool
sample
35-year-old with history of asthma - dies from Histoplasmosis - exam shows lungs 1.5 times the
normal weight. Extensive focal areas of fibrosis and 2-5mm nodules throughout lungs. Biopsy of
nodules show which findings? -Answer- Infiltration of
Lymphocytes and Monocytes
Histoplasma - act like TB and cause cavitary lesions and calcified nodules with fibrotic scarring.
=fungi are combatted by lymphocytes and macrophages, not eosinophils or neutrophils.
32-year-old - painful swelling of right cheek for 24 hours. PE shows swollen and
tender area overlying the parotid gland, Sialolithiasis is suspected. The calculus is
most likely present in a duct that passes through which muscle? -Answer- Buccinator
stensens duct - pierces parotid gland

duct pierces through the bucc. (you can feel it with your tongue)

secretions of the parotid gland are transported to the oral cavity by the Stensen duct. It
83-year-old brought to physician with daughter to discuss complete work-up for dementia. The
patient has mild memory problems. She takes no medications. Mini mental status is 23/30.
Treponema pallidum is positive. Best next step? -Answer-
Discussion of the Diagnosis with the Patient
Privately (talk to her first alone to determine
capacity) -
cognitive impairment could be the manifestation of neurosyphilis. In addition, the doctor
should talk directly to the patient to check for sexual abuse.
d. main goal for this question is avoid discussing issues with relatives without the
patient's permission.

. 15 year old girl - 3 day history of fever, sore throat, and malaise. PE shows pharyngeal
erythema, enlarged tonsils, and tender cervical lymphadenopathy. Incubation with sheep
erythrocytes results in agglutination. The atypical lymphocytes are most likely to be what type?
-Answer- CD8+ lymphocytes
EBV infects B Cells -causes CD8 cells become abnormal' with Mono - monospot test sheep
agglutinin - which are atypical lymphocytes
Lymphocytosis associated with infectious mononucleosis is caused by an increase in the
number of circulating activated T and B lymphocytes.
The atypical lymphocytes are CD8+ T- cells that are activated to eliminate EBV infected B-
lymphocytes.

37 y/o - admitted to hospital because of gradually progressive weakness, anorexia, weight


loss over 6 months. BP 74/40 and skin is Hyperpigmented. Morning serum cortisol
concentration is 2 ug/dl (Low Cholesterol) Which is most likely diagnosis? Answer-
Autoimmune Adrenalitis low cortisol, which causes body wasting.
hyperpigmented since low cortisol increases ACTH secretion from the pituitary, but this
shares a common precursor protein with melanocyte-stimulating hormone (MSH)
often a sign of either adrenal failure or an ACTH-secreting tumor
65-year-old - sudden onset of shortness of breath and discomfort in his chest. PE shows
diaphoresis. An ECG show ST elevation in the anterior leads. Compared with healthy - which
cardiopulmonary change? -Answer- Increased Systemic Vascular Resistance
Decreased Pulmonary Vascular Resistance
Increased Pulmonary Capillary Wedge Pressure Left Atrial
Pressure cardiogenic shock - an anterior wall MI isn't
pumping.

.27-year-old works from home as software engineer- Avoids meetings, "I wish I could have
relationships, but I can't I know they hate me once they get to know me". Which personality
disorder? -Answer- Avoidant
Avoidant personality disorder is characterized by a desire for social relationships, but a fear of
being rejected or feeling inadequate for others.

diaphoresis. An ECG show ST elevation in the anterior leads. Compared with healthy which
cardiopulmonary change? -Answer- Increased Systemic Vascular Resistance
Pulmonary Vascular Resistance- Decreased
Pulmonary Capillary Wedge Pressure- Increased Left Atrial Pressure cardiogenic shock - an
anterior wall MI isn't pumping- SVR increased. CO decreased.

28-year-old man in blood pressure cuff placed around his left arm, the cuff is inflated to occlude
arterial blood flow for 2 minutes and is then removed. Blood flow in the left arm increases by
50% during the next 3 minutes and then decreases to control values -
which humoral substance is most likely involved? -Answer- Adenosine CHALK causes
vasodilation = (CO2, H+, Adenosine, Lactate, K+, K-)

A 22 year old woman - follow-up exam. 5 year history of migraines. She began taking oral
contraceptive 6 months ago, but stopped the contraceptive 1 month ago. The headaches
have since improved. She does not what hormmonal birth control, and she asks for the most
effectibve alternative. It is most appropriate for the physician to recommend which of the
following contraceptive methods for this patient? -AnswerIntrauterine Device non hormonal
most effective alternative COPPER IUD
most effective treatment. Copper IUD = long-acting reversible contraception, most
effective emergency contraception.
21-year-old with asthma - comes to the physician because her current medication regime is not
relieving her symptoms. The physician tells the patient about a clinical trial of a new drug for
asthma. The trial is a large, randomized, prospective, double blinded
study of volunteers with asthma. The trial is most likely occurring at which of the
following phases of drug development? -Answer- Phase 3

large number random assignment, with placebo, any Improvement


The pedigrees of patients with schizophrenia most closely resemble those of patients with
which of the following? -Answer- Diabetes Mellitus, Type 1 Schizophrenia doesn't have a
clear pattern of inheritance. only Type 1 DM doesn't have a clear inheritance pattern
Hox genes play a role in embryogenesis through which of the following processes? -
Answer- Regulation of Transcription

Homebox genes are transcription factors


A 1 month old - male newborn is brought to the physician for a routine examination. Parents
have dark skin and eyes. PE shows hypopigmentation of the skin, light blonde hair, and
translucent irises. The inherited disorder causes this phenotypic expression is most likely due
to a defect in the metabolism of which? -Answer- Tyrosine albinism, which is due to decreased
tyrosinase activity.
If he has a problem metabolizing Phenylalanine, he would be presenting with the PKU
sx like intellectual disability, musty body odor, etc., in addition to his fair complexion
A 55-year-old - 2 weeks history of palpitations and anxiety. He has primary hypothyroidism and
treated with Levothyroxine. Pt is taking 2x dosage of Levothyroxine for 2 months. Which are
most likely set of findings? -Answer- Increased Free T4
Increased Free T3
Decreased Thyroid Iodine Uptake
the patient has been taking excess of his levothyroxine medication. Levothyroxine is the
exogenous form of T4. Therefore, free T4 must be elevated. T4 is converted to T3 at
most peripheral tissues, so T3 will also be elevated.
12-year-old boy - brought to the physician by his mother for a well-child exam. His mother has
outgrown the clothes that she bought him 6 months ago. 50th percentile for height and weight.
Pt shows growth of 2.3 in/year. PE shows mild acne. Pubic hair and testes development are
Tanner stage 2. Cause of onset of physical changes in this patient? -Answer- Nocturnal
Luteinizing Hormone Pulses
Gonadotropin pulses occur only during sleep, but as puberty progresses they can be detected
during the day. By the end of puberty, there is little day-night difference in the amplitude and
frequency of gonadotropin pulses.

A 23-year-old primgravid woman at 22 weeks gestation is brought to a small rural hospital by


her husband becase of intermittent bleeding during past week. Pt woke up in copius amounts
of blood and BP 90/60 mm/Hg. Her female obstetrician is a woman who practices 2 hours
away. Pt is a conservative muslin and dressed in a burka, and she doesn't want a male
physician. Only physician available is a male. Which of the
following is the most appropriate action by the physician? -Answer- Ask the patient if she would
allow the examination if her husband is present at all times
Patient needs medical attention immediately, which eliminates obtaining a court order, or
transferring her. A nurse does not have the same training and qualifications as a physician best
option among those given is to ask the patient if she will allow with her husband present.

A 43 year old woman - nonproductive cough for 3 weeks. She has 6.8 kg (15 lb) weight loss. A
CXR shows 3 nodules in right lung. FNA suggest a malignant neoplasm. A photograph in the
patient is shown. Which of the following is most likely diagnosis Answer- Metastatic
Carcinoma metastasis was the best option here because there are multiple malignant
neoplasms

A 62-year-old man - comes to physician for a follow-up exam. One month ago, he was
prescribed a proton pump inhibitor (PPI) for epigastric pain that relieved when he easts. Pt
states that the symptoms are still present and cause discomfort. Serum studies shows gastrin
4x reference range. Physician discontinues the PPI therapy in this patient. 3 weeks later, most
appropriate to measure the concentration of which? - Answer- Serum Gastrin
The use of any agent that inhibits gastric secretion of acid
rule out Zollinger-Ellison syndrome you need to stop the PPIs first, then measure
Gastrin to make that your intial measure of elevated Gastrin was not due to your drug
rather than the patients disease.

You might also like